2ch勢いランキング アーカイブ

■ちょっとした物理の質問はここに書いてね236■


ご冗談でしょう?名無しさん [] 2019/01/24(木) 13:14:11.19:t3Uj28I4
★荒らし厳禁、煽りは黙殺
★書き込む前に      の注意事項を読んでね
★数式の書き方(参考)はこちら      (予備リンク:       )
===質問者へ===
重要 【 丸 投 げ 禁 止 】

・質問する前に
1. 教科書や参考書をよく読む
2. ttp://http://www.google.com/
  などの検索サイトを利用し、各自で調べる
3. 学生は自分の学年、物理科目の履修具合を書く
4. 宿題を聞くときは、どこまでやってみてどこが分からないのかを書く
5. 投稿する前に、ちゃんと質問が意味の通る日本語か推敲する、曖昧な質問文には曖昧な回答しか返せない
・「力」「エネルギー」「仕事」のような単語は物理では意味がはっきり定義された言葉です、むやみに使うと混乱の元
・質問に対する回答には返答してね、感謝だけでなく「分からん」とかダメOK
・質問するときはage&ID表示推奨
・高度すぎる質問には住人は回答できないかもしれないけれど、了承の上での質問なら大歓迎

===回答者へ===
・丸投げは専用スレに誘導
・不快な質問は無視、構った方が負け
・質問者の理解度に応じた適切な回答をよろしく
・単発質問スレを発見したらこのスレッドへの誘導をよろしくね
・逆に議論が深まりそうなら新スレ立てて移動するのもあり
・板違いの質問は適切な板に誘導を
・不適切な回答は適宜訂正、名回答は素直に賞賛

■ちょっとした物理の質問はここに書いてね234■
ttps://rio2016.5ch.net/test/read.cgi/sci/1542896976/

※前スレ
ttps://rio2016.5ch.net/test/read.cgi/sci/1544530843/
ご冗談でしょう?名無しさん [sage] 2019/01/24(木) 13:14:30.74ID:???
高校物理すらわかってない人が多いですね
ご冗談でしょう?名無しさん [sage] 2019/01/24(木) 13:31:18.64ID:???
劣等感来ると伸びていいね
ご冗談でしょう?名無しさん [sage] 2019/01/24(木) 13:35:45.49ID:???
外力が加えられた後も静止してたんじゃないの?
そう読めるんだけど
ご冗談でしょう?名無しさん [sage] 2019/01/24(木) 16:38:01.21ID:???
この世界は11次元時空だという物理学者が多くいる
そして人間は4次元時空(3次元空間)に適応した生物で4次元時空と3次元時空を認識できて2次元以下と5次元以上の時空は認識できないそうだ
ここで4次元時空に生物がいるなら他の次元時空にも生物がいてもおかしくないのではないか
でも3次元時空は人間が認識できるがそこでは生物は発見できていない
これはなぜなのか?
ご冗談でしょう?名無しさん [sage] 2019/01/24(木) 17:19:55.28ID:???
ムニちゃんポポだのんのおじさんだい
ご冗談でしょう?名無しさん [sage] 2019/01/24(木) 18:01:26.18ID:???
Wikipediaの中性子の項目に制御できないって書いてあるけど
数年前に京大が交流磁場で制御に成功してたよね
ご冗談でしょう?名無しさん [sage] 2019/01/24(木) 19:53:02.05ID:???
3次元時空を認識できてってのが嘘だな
俺が騙されてた
3次元時空を認識できると言ってる人は例えば(t,x,y,z)のzが極めて小さい時の話であって3次元時空に似た4次元時空なんだろう
3次元時空は(t,x,y)でzが存在しない場合とかで(t,x,y,z)上では表現できないから4次元時空の住人は3次元時空を認識出来ないが正解だな
ご冗談でしょう?名無しさん [sage] 2019/01/24(木) 22:45:25.54ID:???
何故、恒星や惑星は自転して、回転して24時間周期と決められているのでしょうかね?

回転が重力、時空間を決めるならそれは、非線形的現象だ。アインシュタインの一般相対論は非対称的現象で書かれているのであって、量子電磁気方程式に従えば自ずと導かれる。

真空との間の流速で電荷が出てくる。
それが重力の正体だったりしてね。
ご冗談でしょう?名無しさん [sage] 2019/01/24(木) 22:53:31.10ID:???
電磁重力波(スカラー波)は存在する。
利権や安全保障、経済的、地政学的権力が絡むから、存在しないことになっている。

決して重力は解明することはないだろう。
ご冗談でしょう?名無しさん [sage] 2019/01/25(金) 02:01:58.60ID:???
削除依頼を出しました
ご冗談でしょう?名無しさん [] 2019/01/25(金) 10:29:59.94:DZHvKxU3
バルディッシュ7S
ビールと、ポカリスエットと白砂糖を混ぜて生命の水を入れて、上級剣聖神癒癒し回復食を入れて、貴世世世世世世世世世世世世世石を作ったのが、フェイトのバルデッシュ・ガンバー、ポカリスエットと白砂糖を入れてつくった生命の水に賢者の石とその応用でを化合して、
生命の石をシリコンマットで下に包み、JAVA++Cでプログラムして、光水をそばに置いて、空中浮遊石ができる。登記
marriageの外部ログインのプログラムを使うといい 登記
ご冗談でしょう?名無しさん [sage] 2019/01/25(金) 13:03:51.94ID:???
自然に直線はあるんですかね
いや球投げると放物線ですから
同じように・・、なんてw
ご冗談でしょう?名無しさん [sage] 2019/01/25(金) 13:04:28.70ID:???
勘違いかな?
ご冗談でしょう?名無しさん [sage] 2019/01/25(金) 13:05:52.76ID:???
物を投げて描かれる曲線は四次元時空上の直線だが
ご冗談でしょう?名無しさん [sage] 2019/01/25(金) 20:12:31.23ID:???
磁石が鉄とくっつく理由がわからない
磁石も鉄も原子核と電子でできている
原子核どうし電子どうしはくっつかないはず
ご冗談でしょう?名無しさん [sage] 2019/01/25(金) 21:17:30.45ID:???
クーパー対で調べろよ
電子同士ではくっつく
まったく話にならんわ
質問の前に調べんのか
ゆとり君はこれだから
二度と答えてはやらん
ご冗談でしょう?名無しさん [sage] 2019/01/25(金) 21:22:30.73ID:???

磁石と鉄が引き合う力にクーパー対はどういう役割があるの?
ご冗談でしょう?名無しさん [sage] 2019/01/25(金) 21:35:51.10ID:???

電子と陽子がくっつく理由は何?
ご冗談でしょう?名無しさん [sage] 2019/01/25(金) 21:43:35.49ID:???

クーパー対って、極低温の超電導状態で初めて現れる効果だと思っていたんだけど
磁石と鉄がくっつく理由はクーパー対で説明できるの?
ご冗談でしょう?名無しさん [sage] 2019/01/25(金) 22:11:51.70ID:???
縦読みもできない無能ばかり
ご冗談でしょう?名無しさん [sage] 2019/01/25(金) 22:18:07.37ID:???

で、クーパー対と強磁性との関連はどういうこと?
ご冗談でしょう?名無しさん [sage] 2019/01/25(金) 22:22:13.72ID:???
んなもんねーよ
半年ROMってろ
ご冗談でしょう?名無しさん [sage] 2019/01/25(金) 22:34:11.31ID:???
鉄の中に微量の磁石が不純物として含まれるから磁石とくっつくんだよ
完全に純粋な鉄は磁石にはくっつかない
ご冗談でしょう?名無しさん [sage] 2019/01/25(金) 23:09:03.31ID:???

かわいそう…
ご冗談でしょう?名無しさん [age] 2019/01/26(土) 11:26:40.63ID:???
リットルとキロってどっちがどっちに合わせたの?
ご冗談でしょう?名無しさん [sage] 2019/01/26(土) 12:38:45.76ID:???
梶野さんのこの発言↓、わけ分からん。弱い重力波が光速度で伝わることに何の疑問が有るんだ?

ttp://https://www.youtube.com/watch?v=mJvmj_nT5Cg&t=3h29m48s
梶野:重力波が光速度を持っていたら僕は驚きですね。そんな理論はきっと今のところ無いでしょうね。
ご冗談でしょう?名無しさん [sage] 2019/01/26(土) 13:39:51.88ID:???

反対スピンと対になってない電子は磁石なんだよ
鉄や酸素などの一部の物質は電子がスピン対になってないから磁石になるのさ
クーパー対は何の関係もないけど「対」と言う言葉で混同したんだろうね
ご冗談でしょう?名無しさん [sage] 2019/01/26(土) 13:47:28.52ID:???
ニュートリノは光速くらい自明
ご冗談でしょう?名無しさん [sage] 2019/01/26(土) 14:03:58.97ID:???

クーパー対で調べろ「よ」
電子同士ではくっつ「く」
まったく話にならん「わ」
質問の前に調べんの「か」
ゆとり君はこれだか「ら」
二度と答えてはやら「ん」
ご冗談でしょう?名無しさん [sage] 2019/01/26(土) 15:07:40.34ID:???
宇宙空間でのニュートリノは宇宙空間での光速よりちょっと速い
ご冗談でしょう?名無しさん [sage] 2019/01/26(土) 18:13:19.88ID:???

クーパー対は関係無いだろ
ご冗談でしょう?名無しさん [sage] 2019/01/26(土) 18:37:03.59ID:???
縦読みだって言ってんだろゴミが
ご冗談でしょう?名無しさん [sage] 2019/01/26(土) 19:36:29.81ID:???

スピンが揃う磁区構造で説明できるから
クーパー対は関係ない。
ご冗談でしょう?名無しさん [sage] 2019/01/26(土) 19:56:25.84ID:???
おれはクーパ対君じゃないがこいつがたて読みだっていってんのにいまだこれに構うやつは頭おかしいんか?
ご冗談でしょう?名無しさん [sage] 2019/01/26(土) 20:08:50.44ID:???
イジング模型とか非線形シグマモデルとかそっち?
ご冗談でしょう?名無しさん [sage] 2019/01/26(土) 20:50:10.88ID:???
縦読みに気付けなかった気恥ずかしさを誤魔化そうとしているのでは?
ご冗談でしょう?名無しさん [sage] 2019/01/26(土) 22:54:47.93ID:???
磁力の原因は交換相互作用という量子力学的効果
ご冗談でしょう?名無しさん [sage] 2019/01/26(土) 23:24:10.94ID:???
磁性理論は未完成
ご冗談でしょう?名無しさん [sage] 2019/01/27(日) 00:18:08.90ID:???
ありとあらゆる物理理論は未完成
ご冗談でしょう?名無しさん [sage] 2019/01/27(日) 11:07:43.90ID:???
熱力学は完成してる
ご冗談でしょう?名無しさん [sage] 2019/01/27(日) 13:32:39.43ID:???

縦読み縦読み言ってたのはそう言う意味か
全然無視してたわ
すぐ忘れるから無駄だけど
ご冗談でしょう?名無しさん [sage] 2019/01/27(日) 13:41:11.24ID:???
ここよ回答者って縦読みもわからないんですね
ご冗談でしょう?名無しさん [sage] 2019/01/27(日) 14:09:16.61ID:???
縦読み強調の理由を暴かないって
みんな優しいね
ご冗談でしょう?名無しさん [sage] 2019/01/27(日) 14:56:13.05ID:???
内部エネルギーは何故Uで表記するんですか?
ご冗談でしょう?名無しさん [sage] 2019/01/27(日) 14:58:29.30ID:???
その辺の謎は多いけど個人的に一番気になるのは
ユニタリ変換のユニタリってなんだよっていうこと
ご冗談でしょう?名無しさん [sage] 2019/01/27(日) 15:09:12.13ID:???
ありとあらゆる学問はとっくの昔に完成しているんだけど
女の子の秘密だから
教えてあけないよ♪じゃん♪

今後とも🍔をよろしくお願いいたします。
ご冗談でしょう?名無しさん [sage] 2019/01/27(日) 15:10:20.62ID:???
また文字がどうたらの議論するの?
ご冗談でしょう?名無しさん [sage] 2019/01/27(日) 15:39:27.98ID:???
わからないんですね
ご冗談でしょう?名無しさん [sage] 2019/01/27(日) 15:46:51.73ID:???
相対性理論がどんなものか教えてください
ご冗談でしょう?名無しさん [sage] 2019/01/27(日) 15:47:18.54ID:???
見る人が違えば見え方が違うということです
ご冗談でしょう?名無しさん [sage] 2019/01/27(日) 15:49:39.16ID:???

1ってこと
ご冗談でしょう?名無しさん [sage] 2019/01/27(日) 15:51:56.80ID:???

議論はしない
由来を知ってる人が来るのを待ってる
ご冗談でしょう?名無しさん [sage] 2019/01/27(日) 16:56:37.40ID:???
最近、くっくっくを見ないのは
こんなふうにスレが腐ってるからか。
ご冗談でしょう?名無しさん [sage] 2019/01/27(日) 17:00:21.61ID:???
コンデンサで直流がカットされるのはなんで?
ご冗談でしょう?名無しさん [sage] 2019/01/27(日) 17:01:45.03ID:???
直流だとどんどん充電されて電池と同じ電圧のところでストップします
ご冗談でしょう?名無しさん [sage] 2019/01/27(日) 21:05:34.19ID:???

光速度が一定だと時間や空間が伸び縮みするって話さ
ご冗談でしょう?名無しさん [sage] 2019/01/27(日) 21:41:39.24ID:???
3次元空間に2次元空間が含まれるというのはあり得ますか?
あるとすれば(x,y,z)とzが存在しない(x,y)をどのように結びつけるのでしょうか?
ご冗談でしょう?名無しさん [sage] 2019/01/27(日) 22:17:19.03ID:???

ttp://https://en.m.wikipedia.org/wiki/Isosurface
ご冗談でしょう?名無しさん [] 2019/01/27(日) 23:22:33.67:R0S46fx7
ぷっちーポポ ぷちプラーイ
ご冗談でしょう?名無しさん [sage] 2019/01/27(日) 23:55:28.16ID:???

zが任意の値や一定値ではなくzが存在してない(x,y)の話ですよ
ご冗談でしょう?名無しさん [sage] 2019/01/27(日) 23:57:32.72ID:???
きみ学部で線形代数すら習わなかったの?
ご冗談でしょう?名無しさん [sage] 2019/01/27(日) 23:58:16.78ID:???
紙を手に持ってみましょう
その紙は3次元空間に存在する2次元空間だと考えられます

式で表すとどうなりますか?
わかりませんね
あなたが紙の持ち方変えたり形を変えたらその都度変わりますから
59 [sage] 2019/01/28(月) 00:12:05.47ID:???
,63
厨坊か。馬鹿すぎて話にならんな。
63 [sage] 2019/01/28(月) 00:19:45.73ID:???
どうして馬鹿なんですか?
ご冗談でしょう?名無しさん [sage] 2019/01/28(月) 01:30:29.49ID:???
ゆらぎとは何ですか?
ご冗談でしょう?名無しさん [sage] 2019/01/28(月) 02:03:31.98ID:???

次元定理
ご冗談でしょう?名無しさん [sage] 2019/01/28(月) 03:31:02.02ID:???

紙には厚さがあるから3次元
ご冗談でしょう?名無しさん [sage] 2019/01/28(月) 06:25:53.27ID:???
半導体の接合で二次元電子系を作ることはできる
もちろんz軸方向のギャップが大きいだけの擬二次元系だけど
ご冗談でしょう?名無しさん [sage] 2019/01/28(月) 07:44:50.90ID:???
平面のパラメータ表示も出来ん厨坊に、説明したって無駄。
ご冗談でしょう?名無しさん [sage] 2019/01/28(月) 12:34:45.30ID:???
ここの人たちって普段何してる人なの?ニート?
ご冗談でしょう?名無しさん [sage] 2019/01/28(月) 12:43:48.99ID:???
そう言うお前は?
ご冗談でしょう?名無しさん [sage] 2019/01/28(月) 12:45:56.42ID:???

紙は繊維の集合だから一次元
ご冗談でしょう?名無しさん [sage] 2019/01/28(月) 12:55:44.85ID:???
質問返しきっつw
ご冗談でしょう?名無しさん [sage] 2019/01/28(月) 13:31:59.18ID:???
1kgの鉄と1kgの綿はどちらの方が重いですか?
ご冗談でしょう?名無しさん [sage] 2019/01/28(月) 13:57:02.64ID:???

あり得ませんし
ゴミ質問ですね
ご冗談でしょう?名無しさん [sage] 2019/01/28(月) 14:19:23.09ID:???

運動しない三次元は実在しない
二次元は三次元と独立していない
一次元は0次元と独立していない
0次元は実在していない
ご冗談でしょう?名無しさん [sage] 2019/01/28(月) 16:31:25.18ID:???

空気中で重力の影響がある場合は浮力が働くから綿の方が軽い
ご冗談でしょう?名無しさん [sage] 2019/01/28(月) 17:01:50.54ID:???

綿を鉄以上の密度まで圧縮してないとは誰も言ってないから鉄に働く浮力のほうが大きくなる可能性を信じて否定できない
ご冗談でしょう?名無しさん [sage] 2019/01/28(月) 17:40:18.53ID:???
はいはいお利口さんでちゅねー
ご冗談でしょう?名無しさん [sage] 2019/01/28(月) 18:58:35.55ID:???

埋め込みはめ込み言われて分かる?


埋め込みとはめ込み (岩波オンデマンドブックス)
足立 正久
ttp://https://www.amazon.co.jp/dp/4007307059/
ご冗談でしょう?名無しさん [] 2019/01/28(月) 22:44:09.58:/18D0xLT
統計力学やってて、温度って何なのか分からなくなってきた。何のために温度の次元なんて物を導入したんだ?kTの形にひとまとめにしてエネルギーの次元にしてはいけなかったのか?
ご冗談でしょう?名無しさん [sage] 2019/01/28(月) 22:45:30.55ID:???
ユニタリの意味が分からない人はよほど粗雑な本で線形代数を学んだんだろうけど、
そういう粗雑な本が名著扱いされてるのも問題だ
ご冗談でしょう?名無しさん [sage] 2019/01/28(月) 22:52:44.96ID:???
単語の意味というのは数学とは別の分野の話だ
ご冗談でしょう?名無しさん [sage] 2019/01/28(月) 23:17:42.56ID:???
お前は時間をメートルの次元で数えるのか?
ご冗談でしょう?名無しさん [sage] 2019/01/28(月) 23:23:32.43ID:???

それが出来るから
「光年は距離の単位」
ってツッコミ見るたびにどうよってなる
ご冗談でしょう?名無しさん [sage] 2019/01/28(月) 23:37:04.73ID:???
時間も質量も長さとして計算するでしょ、幾何学単位系とかで。
ご冗談でしょう?名無しさん [sage] 2019/01/28(月) 23:57:37.96ID:???

熱力学で考えれば温度とエネルギーは関係ないですよね
熱のやりとりがされるかどうかの基準となる示強量が温度です
ご冗談でしょう?名無しさん [] 2019/01/29(火) 00:10:55.42:xX10UxUP

もしエントロピーをボルツマン定数で割ったものを新しいエントロピーとして、温度にボルツマン定数を掛けたものを新しい温度としたら何か不具合が生じたりするのでしょうか?
ご冗談でしょう?名無しさん [sage] 2019/01/29(火) 00:45:14.22ID:???
自然単位系とかプランク温度とかでググると幸せになれると思うよ
ご冗談でしょう?名無しさん [sage] 2019/01/29(火) 00:46:36.73ID:???
そもそもボルツマン定数は統計力学でのエントロピーの単位を熱力学のそれと合わせるための定数
流れが逆なんだよね
統計力学が先ならボルツマン定数なんてわざわざ持ち出さずに1にするだろう
ご冗談でしょう?名無しさん [sage] 2019/01/29(火) 00:51:51.32ID:???
ボルツマン定数の正体は未だによく分からない

てか有名所の定数では一番何が何だか分からない定数に見える
ご冗談でしょう?名無しさん [sage] 2019/01/29(火) 01:13:06.71ID:???

プランク定数より?
ご冗談でしょう?名無しさん [sage] 2019/01/29(火) 01:44:41.26ID:???

粗雑じゃない本の具体例はよ
ご冗談でしょう?名無しさん [sage] 2019/01/29(火) 06:29:34.55ID:???
エントロピーは本質的にマクロな量だというだけ
菩薩@太子 [] 2019/01/29(火) 08:43:09.65:giq1cTT/
宇宙はビッグバンによって「無」から始まったといわれているが、
その「無」はいまもあるのか?
「無」が「宇宙」に変わってしまったからもうないのか?
それとも「無」から「宇宙」が沸いてきたから、いまもあるのか?

どっちなんだろう?
菩薩@太子 [] 2019/01/29(火) 09:05:23.19:giq1cTT/
水野裕子と菜々緒が喧嘩するとどっちが勝つのか?

この問いかけは、我にとって宇宙の果てを問うに等しい質問であるから、真面目に答えてくれ!!
ご冗談でしょう?名無しさん [sage] 2019/01/29(火) 09:07:53.69ID:???

死ね、キチガイ
ご冗談でしょう?名無しさん [sage] 2019/01/29(火) 12:11:47.59ID:???
不思議なのですが
屈折のとき、媒質によって波長も光速も変動するのに、
振動数だけはなぜ保存するのですか?
ご冗談でしょう?名無しさん [sage] 2019/01/29(火) 12:13:59.39ID:???

逆に訊こう。なぜ振動数が変わると思った?
ご冗談でしょう?名無しさん [sage] 2019/01/29(火) 12:43:23.11ID:???

振動数は境界で一致せんとあかんやん
ご冗談でしょう?名無しさん [sage] 2019/01/29(火) 13:18:00.76ID:???
めっちょ強い光だと振動数は変わる
これを非線形光学効果と言います
ご冗談でしょう?名無しさん [sage] 2019/01/29(火) 14:15:05.14ID:???
それは解釈が間違い
高調波が発生するだけ
ご冗談でしょう?名無しさん [sage] 2019/01/29(火) 14:28:16.28ID:???

それを非線形光学効果と言います
ご冗談でしょう?名無しさん [sage] 2019/01/29(火) 14:54:03.34ID:???

なるほど……波が1個入ったら1個出るからですね😅
ありがとうございます
ご冗談でしょう?名無しさん [sage] 2019/01/29(火) 17:41:48.54ID:???
いつから以降のテンプレが消えたん?
ご冗談でしょう?名無しさん [sage] 2019/01/29(火) 18:07:15.30ID:???
宇宙の話になると3次元空間ではなく4次元以上の空間が存在するかのような話になりますが
4次元以上の空間が存在する可能性は極めて高いのですか?
そのような証拠になりそうなものはたくさんあるのでしょうか?
ご冗談でしょう?名無しさん [sage] 2019/01/29(火) 18:21:21.65ID:???
あなたの言う次元とはなんですか?
ご冗談でしょう?名無しさん [sage] 2019/01/29(火) 18:26:12.14ID:???

次元大介
ご冗談でしょう?名無しさん [sage] 2019/01/29(火) 18:35:33.73ID:???
そういやトンデモ科学で
量子論の無限次元ヒルベルト空間と相対論における次元をごっちゃにした言説をみないのは何でなんだろう
ご冗談でしょう?名無しさん [sage] 2019/01/29(火) 18:40:39.08ID:???

限定していません
ご冗談でしょう?名無しさん [sage] 2019/01/29(火) 18:44:07.98ID:???

限定してください?
私はあなたがどう言う意味で次元と言ってるのかわかりませんから


トンデモさんはヒルベルト空間なんて知らないからです
トンデモさんにとって量子力学は猫を殺害する学問ですから
ご冗談でしょう?名無しさん [sage] 2019/01/29(火) 18:51:51.24ID:???

限定しないとわからないという理由は何でしょうか?
例えばよく言われる4次元空間とは限定されたものなのですか?
ご冗談でしょう?名無しさん [sage] 2019/01/29(火) 18:53:08.46ID:???
4次元空間は空間に時間を加えたものと説明されますね

でもあなたはこの説明で納得できますか?
納得できないなら、あなたの次元の定義は一般的な物理の次元の定義とは異なると言うことになるのですけど
ご冗談でしょう?名無しさん [sage] 2019/01/29(火) 18:58:42.43ID:???
4次元空間と4次元時空は違うだろ
ご冗談でしょう?名無しさん [sage] 2019/01/29(火) 19:04:03.50ID:???
数式の書き方の例 ※適切にスペースを入れると読みやすくなります
●括弧: (), [], {}を適切に入れ子にして分かりやすく書く
●スカラー: a,b,...,z, A,...,Z, α,β,...,ω, Α,Β,...,Ω,...(「ぎりしゃ」「あるふぁ〜おめが」で変換)
●ベクトル: V=(v1,v2,...), |V>,V↑, (混乱しないならスカラーの記号でいい。通常は縦ベクトル)
●テンソル: T^[i,j,k...]_[p,q,r,...], T[i,j,k,...; p,q,r,...] (上下付き1成分表示)
●行列: M[i,j], I[i,j]=δ_[i,j] M = [[M[1,1],M[2,1],...], [M[1,2],M[2,2],...],...], I = [[1,0,0,...],[0,1,0,...],...]
(右は全成分表示。行または列ごとに表示する。例:M=[[1,-1],[3,2]])
●対角行列: diag(a,b) = [[a,0],[0,b]]
●転置行列・随伴行列:M^T, M†("†"は「だがー」で変換可) ●行列式・トレース:|A|=det(A), tr(A)
●複号: a±b("±"は「きごう」で変換可)
●内積・外積: a・b, a×b
●関数・汎関数・数列: f(x), F[x(t)] {a_n}
●平方根: √(a+b) = (a+b)^(1/2) = sqrt(a+b) ("√"は「るーと」で変換可)
●指数関数・対数関数: exp(x+y)=e^(x+y) ln(x)=log_e(x) (底を省略して単にlogと書いたとき多くは自然対数)
括弧を省略しても意味が容易に分かるときは省略可: sin(x) = sin x
●三角関数、逆三角関数、双曲線関数: sin(a), cos(x+y), tan(x/2), asin(x)=sin^[-1](x), cosh(x)=[e^x+e^(-x)]/2
●絶対値:|x| ●ノルム:||x|| ●共役複素数:z^* = conj(z)
●階乗:n!=n*(n-1)*(n-2)*...*2*1, n!!=n*(n-2)*(n-4)*...
ご冗談でしょう?名無しさん [sage] 2019/01/29(火) 19:04:14.32ID:???
すいません
4次元空間という表現は4次元時空とは別物で時間を入れてないと思ってました
自分は時間を入れない4次元以上の空間が存在する可能性について知りたいのです
ご冗談でしょう?名無しさん [sage] 2019/01/29(火) 19:04:24.34ID:???
質問・回答に標準的に用いられる変数の例

a:加速度、昇降演算子 A:振幅、ベクトルポテンシャル B:磁束密度 c:光速 C:定数、熱・電気容量
d:次元、深さ D:領域、電束密度 e:自然対数の底、素電荷 E:エネルギー、電場
f:周波数 f,F:力 F:Helmholtzエネルギー g:重力加速度、伝導度
G:万有引力定数、Gibbsエネルギー、重心 h:高さ、Planck定数 H:エンタルピー、Hamiltonian、磁場
i:虚数単位 i,j,k,l,m:整数のインデックス I:電流、慣性モーメント j:電流密度・流束密度
J:グランドポテンシャル、一般の角運動量 k:バネ定数、波数、Boltzmann定数 K:運動エネルギー
l,L:長さ L:Lagrangian、角運動量、インダクタンス m,M:質量 n:物質量 N:個数、トルク
M:磁化 O:原点 p:双極子モーメント p,P:運動量、圧力 P:分極、仕事率、確率 q:波数
q,Q:一般化座標、電荷 Q:熱 r:距離 R:抵抗、気体定数 s:スピン S:エントロピー、面積 t,T:時間 T:温度
U:ポテンシャル、内部エネルギー v:速度 V:体積、ポテンシャル、電位
W:仕事、状態数 x,y,z:変数、位置 z:複素変数 Z:分配関数

β:逆温度 γ:抵抗係数 Γ:ガンマ関数 δ:微小変化 Δ:変化 ε:微小量、誘電率 θ:角度 κ:熱伝導率
λ:波長、固有値 μ:換算質量、化学ポテンシャル、透磁率 ν:周波数 Ξ:大分配関数 π:円周率 ρ:(電荷)密度、抵抗率
σ:スピン τ:固有時 φ:角度、ポテンシャル、波動関数 ψ:波動関数 ω:角振動数 Ω:状態密度
ご冗談でしょう?名無しさん [sage] 2019/01/29(火) 19:06:09.06ID:???

統計力学という分野ではガンマ空間というのがありますね

考えたい粒子の個数だけ次元が増えてって普通は10の何十乗という感じで次元が増えています

あなたのいう次元とは、そういうことですか?
ご冗談でしょう?名無しさん [sage] 2019/01/29(火) 19:14:34.70ID:???
何故時間の次元の1つなのでしょうか?
ご冗談でしょう?名無しさん [sage] 2019/01/29(火) 19:16:24.57ID:???
次元とは現象を表すのに必要な変数な数だからですね

場所と時間がわかれば十分だと考えれば4次元です
ご冗談でしょう?名無しさん [sage] 2019/01/29(火) 19:18:27.74ID:???

どう言った理由でその次元ではないかという発想に至ったのですか?
ご冗談でしょう?名無しさん [sage] 2019/01/29(火) 19:19:29.72ID:???

あなたは限定しないと言いましたよね

やはり、あなたの知りたい次元とはどういう類のものなのかを限定しないのではいけないのですか?
ご冗談でしょう?名無しさん [sage] 2019/01/29(火) 19:20:07.25ID:???
なんか日本語変ですね

あなたの知りたい次元の定義をちゃんと書いてくださいね
ご冗談でしょう?名無しさん [sage] 2019/01/29(火) 19:24:36.95ID:???
の文脈から3次元空間というのが時間を入れてるかもしれないという発想に至った経緯が知りたい
ご冗談でしょう?名無しさん [sage] 2019/01/29(火) 19:26:40.06ID:???
あなたは次元を限定しないと言いましたよね

でも実際は限定しているわけです

なら早くそれを書いてください?
ご冗談でしょう?名無しさん [sage] 2019/01/29(火) 19:33:10.05ID:???

こちらとしては限定しなくてもいいですよってこと
の文脈から整合性の取れる解答があれば
自分が期待してない答えでもそれはそれでためになるでしょうから
ご冗談でしょう?名無しさん [] 2019/01/29(火) 22:02:26.92:IRffm6Hy
潮汐力のとこがわからない。

月側の方はより月の引力に引かれるので
地球の引力をその分打ち消すわけだから
月がない側の地表が強い重力になるはずで、もし重力の勾配が無いと仮定したら月のない側がいつも海面上昇してると思う。

この場合だと一日に一回の干満になるはずが、現実には一日二回。それは月の引力の勾配で、地球表面の引力が月側と反対側で丁度同じくらいになるから、ではないかと考えたんですけど、ウィキの説明だと違うみたいで。

月側とその反対側に膨らみができるのは
その両方がそれ以外の地表より重力が強くなってるからじゃないんですか?そのため水がそちらに強く引き付けられて海面が膨らむ。

どうもウィキの説明が納得できないんで。
ご冗談でしょう?名無しさん [sage] 2019/01/29(火) 22:58:31.27ID:???
量子力学的には「過去」は安定したものなのでしょうか?
極端な事を言えば、朝、家を出る時に鍵を閉めたかどうか思い出せない場合、
家に戻って確かめるまでの間、鍵がかかっているかどうか確定しないのでしょうか?
ご冗談でしょう?名無しさん [sage] 2019/01/29(火) 23:01:01.27ID:???
特になにもなければ、鍵が閉まっている状態が固定されていますので、安心できるわけですね
ご冗談でしょう?名無しさん [sage] 2019/01/29(火) 23:04:15.57ID:???

ウィキの説明は読んでいないが……

重力は、相手に近いほど強く、遠いほど弱い。
遠心力は、相手に近いほど弱く、遠いほど強い。
従って、相手に近い側では、重力>遠心力となり、相手に近付くように盛り上がる。
一方、遠い側では、重力<遠心力となり、相手から遠ざかるように盛り上がる。
ご冗談でしょう?名無しさん [sage] 2019/01/29(火) 23:07:59.66ID:???

ニュートンがプリンキピアで幾何学的に説明した方法↓ p.30〜
ttp://http://www.phys.cs.is.nagoya-u.ac.jp/~tanimura/lectures/Tanimura_20161001.pdf
ご冗談でしょう?名無しさん [sage] 2019/01/30(水) 00:00:04.20ID:???

たぶん、基本的なところに誤解があると思うんだけど…
とりあえず、いろいろめんどくさい事を考えるのはやめて、
地球は均一でなめらかな真球で、表面には海があるものとしよう。
めんどくさいから地球は自転も公転もしていなくて、それどころか宇宙には地球以外には天体が一切存在していないものとする。
この状態で海がどうなるか考えると…、まあ当然まん丸だ。どこにも偏りはない。

少し状況を複雑にして、月を作って、そのままだと重力で地球に落ちてきてしまって困るのでつっかえ棒をして支える事にしよう。
これがで仮定された状態(重力のみを考えた状態)だが……
これは、どう考えても、月のある側に海水が多く集まる。
地球の重力は地球上にいる限りどこでも一定だけど、月の重力は月のある方に一方的に引っ張ってるんだから月のある方向に海水が集まるのは当然だ。
月の反対側に海水が集まるわけがない。これが誤解だ。

で、つっかえ棒をしたままおもむろに月を中心に地球をぶんぶん光速回転させるとする。
地球は固体なのでなかなか変形しないが、液体である海水は流れて月から遠い方に集まるだろう。
これが、公転による遠心力のみを考えた場合だ。

実際には月の引力と遠心力は釣り合っていて、月と地球は距離を一定に保ったまま月-地球系の重心の周りを公転しているから、
月の重力による影響と、月-地球系の公転による影響が同じだけ海水に表れるわけだ
ご冗談でしょう?名無しさん [sage] 2019/01/30(水) 00:00:54.46ID:???

これ読んで今すごい衝撃受けてる……
膨らむ方向が90度違うって
ええ……
でも満潮干潮時刻を見ると確かに
えええ……
ご冗談でしょう?名無しさん [sage] 2019/01/30(水) 00:28:59.12ID:???

更には、その位相差によって月はエネルギーを獲得し、地球から離れていっている。
ご冗談でしょう?名無しさん [] 2019/01/30(水) 00:33:46.56:7q3i6lz+

第二段落のとこがなぜ自明かわからない。

中空の球にウニのように柔らかい針を沢山付けたとして、それをある方向からバキュームする。そしたら、柔らかい針はバキュームの方に流れるように傾くと思う。

でも地球は中空の球ではないし、月より引力が大きい。この場合に、月の引力と地球の引力が作る重力の場を考えて、その等位面を考えると地球上では月と反対側の方が遥かに勾配がきついでしょ。。
物体はより勾配のあるとこでより力を受けるんだから、水はそっちに流れるはず。
ご冗談でしょう?名無しさん [sage] 2019/01/30(水) 04:07:51.67ID:???

確かに勾配の大きなところでは強い力を受けるけど
強い力を受けるような場所なんかに留まれないでしょ

水はポテンシャルが一番低いところに集まる
ご冗談でしょう?名無しさん [sage] 2019/01/30(水) 06:46:05.93ID:???
一番低い所...基底エネルギー状態...
ご冗談でしょう?名無しさん [] 2019/01/30(水) 10:58:44.79:/lFshz55
プラスとマイナスはくっつくのになんで原子核と電子は分裂したままの状態が安定なんですか?
ご冗談でしょう?名無しさん [sage] 2019/01/30(水) 11:00:44.34ID:???
月と地球は互いに引き合ってますけど、月が落ちくることはないですよね
遠心力で地球と反対側に引っ張られるから、万有引力に逆らうことができるのです

それと同じですね
ご冗談でしょう?名無しさん [sage] 2019/01/30(水) 11:24:49.80ID:???
運動する荷電粒子は電磁波を放出してエネルギーを失う
ご冗談でしょう?名無しさん [sage] 2019/01/30(水) 11:27:59.83ID:???
わからないんですね
ご冗談でしょう?名無しさん [] 2019/01/30(水) 12:01:56.98:L739MtLp
縮退圧力
ご冗談でしょう?名無しさん [sage] 2019/01/30(水) 12:12:38.65ID:???
測地線を通っているからだろうね
ご冗談でしょう?名無しさん [sage] 2019/01/30(水) 12:29:13.95ID:???
荷電粒子の“電荷”
たとえば原子の軌道電子の一つ、あるいはその原子核の陽子の一つは局所的な粒子とその周囲の真空との間の流束強度の差(ポテンシャル)を表すなら、それは重力ではないだろうか。
ご冗談でしょう?名無しさん [sage] 2019/01/30(水) 12:33:18.79ID:???

不確定性関係から
ご冗談でしょう?名無しさん [sage] 2019/01/30(水) 13:33:02.08ID:???

くっついてるやんか
反応しないから、くっついたままだし
くっついた状態で振動してるから広がってるのさ
ご冗談でしょう?名無しさん [] 2019/01/30(水) 14:33:21.54:p6H001Is
容器に入れた水が地表と水平で安定するのは地表と水平に重力場の等位面があるからで、じゃあ等位面がもし凸型になっていたらどうなるか。

凸に盛り上がった等位面の部分というのは、水にとって上に書いた水平な等位面と同じなんだから、水は同じ等位面の内側を満たすように動く。

このように考えるとき、月側が水で膨らむとすると地球の月側の引力が月のない側より強いことになる。地球と月の引力が作る場を考えると、それはおかしいと思うんだが。
ご冗談でしょう?名無しさん [sage] 2019/01/30(水) 16:06:50.12ID:???
何故反粒子の電荷は逆符号で、質量は同じなのでしょうか?
ご冗談でしょう?名無しさん [sage] 2019/01/30(水) 16:53:40.54ID:???
対称性の原理
対称性があると美しいだろ、物理学者はミクロ世界の対称性をこよなく愛する。
ご冗談でしょう?名無しさん [sage] 2019/01/30(水) 17:04:12.06ID:???

CPT定理
ご冗談でしょう?名無しさん [sage] 2019/01/30(水) 19:53:41.58ID:???
電子が原子核に落っこちない理由を明示的に書いてある量子力学の本って何?
ご冗談でしょう?名無しさん [sage] 2019/01/30(水) 20:07:09.59ID:???
どんなのにも書いてあるんじゃないですか?
ご冗談でしょう?名無しさん [sage] 2019/01/30(水) 20:17:21.88ID:???
水素原子内の電子の位置の確率密度は中心が一番高い、原子核内の確率は小さくないが
原子核内の電気力は弱く、電子とクォークは低エネルギーで反応しない。
さらに不確定性原理により、原子核の小さな領域に留まっていることができない。
ご冗談でしょう?名無しさん [sage] 2019/01/30(水) 20:55:09.42ID:???
>水素原子内の電子の位置の確率密度は中心が一番高い

俺が理解した量子力学ではそうではなかった。
動径方向の波動関数は原点以外に確率密度の最大値がある。
ご冗談でしょう?名無しさん [sage] 2019/01/30(水) 20:56:16.89ID:???

ミスプリが多いけど、不確定性関係で説明してる。

演習しよう量子力学
ttp://https://www.amazon.co.jp/dp/4864810400
演習問題 16.1 (2) 電子が陽子に落ち込まない理由について説明せよ。
ご冗談でしょう?名無しさん [sage] 2019/01/30(水) 21:12:55.15ID:???

原子核中心が確率密度最大でも体積は無限小だから、動径方向の体積の確率なら
ボーア半径の距離が最大になる。
ご冗談でしょう?名無しさん [sage] 2019/01/30(水) 21:14:12.36ID:???

明示的に書いてる本はそんなにない気がするぞ
ご冗談でしょう?名無しさん [sage] 2019/01/30(水) 21:15:23.40ID:???
動径関数と動径分布関数の違いだな
ご冗談でしょう?名無しさん [sage] 2019/01/30(水) 21:18:04.74ID:???
落ちるなんて考えが古典なんです
ご冗談でしょう?名無しさん [sage] 2019/01/30(水) 21:25:29.75ID:???

ディラック方程式より
ご冗談でしょう?名無しさん [sage] 2019/01/30(水) 21:26:46.05ID:???

負の質量がないからです
ご冗談でしょう?名無しさん [sage] 2019/01/30(水) 21:42:20.14ID:???
パウリのスピン行列ってなんであんな成分を持ってるの?
たまたま実験結果を説明しようとしたらあれが出てきたの?
ご冗談でしょう?名無しさん [sage] 2019/01/30(水) 22:11:03.36ID:???

なにそれ?
ご冗談でしょう?名無しさん [sage] 2019/01/31(木) 00:11:55.44ID:???
そういう数学的道具があれば説明するのに便利だよねっていうので出てきた
ハミルトンの四元数も似たような性質持ってる現代数学的なややこしい話につながるからあんまり興味はない
ご冗談でしょう?名無しさん [sage] 2019/01/31(木) 01:19:34.87ID:???
ディラック作用素の指数定理いいよね・・・
ご冗談でしょう?名無しさん [sage] 2019/01/31(木) 01:30:59.55ID:???

グラフの横軸が動径だから錯覚してるだけ
直交座標にすれば中心が最大と分かる
ご冗談でしょう?名無しさん [sage] 2019/01/31(木) 04:32:36.19ID:???
軌道電子とかだったら定常状態で安定しているだろうから、電気的に反応しない。ミクロ世界に干渉するために、マクロ的に定常状態を作り出して、内部パターンを変えればいい。

3次元的に定常状態を作り出して、周波数を弄れば、マクロ的に時空間に影響が出るらしいな。
ご冗談でしょう?名無しさん [sage] 2019/01/31(木) 05:38:50.85ID:???

su(2)またはso(3)の既約表現行列の
一番簡単なもの
ご冗談でしょう?名無しさん [sage] 2019/01/31(木) 06:43:44.39ID:???

だから動径関数と動径分布関数の違いだってば
ご冗談でしょう?名無しさん [sage] 2019/01/31(木) 06:45:41.08ID:???

単語並べるだけの厨二は帰れ
ご冗談でしょう?名無しさん [sage] 2019/01/31(木) 07:15:30.52ID:???

ばか
ご冗談でしょう?名無しさん [sage] 2019/01/31(木) 07:17:14.90ID:???
音響浮揚...
ご冗談でしょう?名無しさん [sage] 2019/01/31(木) 09:00:15.74ID:???

???
ご冗談でしょう?名無しさん [sage] 2019/01/31(木) 17:58:58.26ID:???
実は宇宙の場所によっては光の速さはマチマチである規則性にそって変化してたりする可能性もある?
光の速さの測定は地球よりかなり遠いところで出来ないから確定してないよね?
ご冗談でしょう?名無しさん [sage] 2019/01/31(木) 18:07:12.00ID:???
すべての物理法則についても同じことが言える
ご冗談でしょう?名無しさん [sage] 2019/01/31(木) 22:10:14.45ID:???
動径関数と動径分布関数の違い
ttp://https://keisan.casio.jp/exec/system/1176445481
ご冗談でしょう?名無しさん [sage] 2019/02/01(金) 01:13:32.41ID:???
そんなことは最初から知ってた
ご冗談でしょう?名無しさん [sage] 2019/02/01(金) 01:18:39.55ID:???
宇宙の観測と理論の整合性は
同じ法則でないと説明できない
ご冗談でしょう?名無しさん [sage] 2019/02/01(金) 09:16:29.04ID:???
シミュレーションですが、FDTDと有限要素法だと、FDTDのほうが過渡状態も計算できると聞いたのですが、正しいでしょうか?
ご冗談でしょう?名無しさん [sage] 2019/02/01(金) 17:55:52.25ID:???
何の略かすら知らんな
ご冗談でしょう?名無しさん [sage] 2019/02/01(金) 21:42:53.40ID:???
荷電スピンってなんなんだよ
なんで陽子と中性子が荷電スピンの違いだけで同じ核子って言えるんだよ
質量違うし
ご冗談でしょう?名無しさん [sage] 2019/02/01(金) 23:02:37.53ID:???
歴史的な概念ですね

陽子と中性子が素粒子だと考えられていた時のお話しです
二つはほとんど同じものだけど、荷電スピンだけが違うだけ
そういう風に考えて理論を組み立てたわけです

実際には陽子も中性子も素粒子ではなかったわけですから、今となっては意味のないものとなっています
ご冗談でしょう?名無しさん [sage] 2019/02/02(土) 01:46:28.86ID:???
超伝導とか超巨大磁気抵抗効果とか超流動とか超新星とかの他に超がつく現象を教えてください
ご冗談でしょう?名無しさん [sage] 2019/02/02(土) 01:47:07.17ID:???
超音波
ご冗談でしょう?名無しさん [sage] 2019/02/02(土) 01:51:26.80ID:???
超臨海流体
超対称性
超ひも理論

もっとあるんじゃないかと思ったが意外と思いつかないね
ご冗談でしょう?名無しさん [sage] 2019/02/02(土) 10:34:33.66ID:???
ウルトラヴァイオレット発散
ご冗談でしょう?名無しさん [sage] 2019/02/02(土) 10:59:35.10ID:???
超高速衝突近似
ご冗談でしょう?名無しさん [sage] 2019/02/02(土) 11:00:59.97ID:???
超時空要塞
ご冗談でしょう?名無しさん [sage] 2019/02/02(土) 12:01:28.46ID:???
超微細構造
ご冗談でしょう?名無しさん [sage] 2019/02/02(土) 13:22:38.81ID:???

u, d クォークの対称性さ
この2つのクォークは特に軽いから入れ替えても大差ないという
近似的対称性
ご冗談でしょう?名無しさん [sage] 2019/02/02(土) 17:31:15.06ID:???
正孔がスピン持ってるってホワイ
ご冗談でしょう?名無しさん [sage] 2019/02/02(土) 21:22:17.23ID:???
ドラゴンボールの手からエネルギー出す奴どうやったらできますか?
ご冗談でしょう?名無しさん [sage] 2019/02/02(土) 22:53:23.19ID:???
手からエネルギーくらい いつも出てる
ご冗談でしょう?名無しさん [sage] 2019/02/03(日) 07:25:33.01ID:???
定常波は基底状態でも産み出されている。物質とエネルギーを支えているのは基底状態(定常状態)であり、特別な電磁気回路を使えば時空間/時間で、高次エネルギーを取り出す。

1、定常波を生み出すには電磁波を破壊する干渉、またはゼロベクトルにする合成で真空に応力を生じる。(五次元ポテンシャル:カルツァクライン理論)

2、その一成分が重力ポテンシャルであり、その重力ポテンシャルは勾配として流れてくる。電磁気へと放出する経路をそれぞれ閉め出すと物質に与える時空間にも影響が出る。

3、低次の超空間では、物質の貫通が可能である。例えば、ワープしたいと思ったら多段ゼロ合成によって適切な流量を調節しなければならない。
ご冗談でしょう?名無しさん [sage] 2019/02/03(日) 11:31:46.95ID:???
基地外は基地外用のスレに書き込んでくれ
ご冗談でしょう?名無しさん [sage] 2019/02/03(日) 12:53:38.25ID:???
低次の超次元では
はぁ
ご冗談でしょう?名無しさん [sage] 2019/02/03(日) 12:58:35.68ID:???
原子核の中では、核子が絶えず陽子と中性子の間を行きつ戻りつ姿を変えているので、その電荷は核子全体に“行きわたり”、すべての核子により共有されている。

さらにまた各元素(実際には各同位体)は、それぞれ固有のフーリエ変換スカラー周波数、振幅などの“集合パターン”を持っている。
このパターンはもちろん、改良された電磁波送信機により、人工的に再現し送信することができる。
しかし,核子(仮想的な電荷の流れの交換により相互に行きつ戻りつ姿を変えている陽子と中性子)には、一種の“親鍵”となる電気重力パターンがある。
もしこのパターンを反対にして“充電を逆転”させると、“反対の重力荷で質量を充電する”ことになり、外部の観測者にとってその充電している質量は、どんどん軽くなり、その慣性もどんどん小さくなる。

最終的にそれは(観測者にとり)負の質量と負の慣性を持つに至り、加速して地球から彼方へと飛び去る。
物体は“下に落下”する代わりに“上に落下”するのである。

またある奇妙な時間効果も起きる; 物体は時間の中を実験室の観測者よりもゆっくりと移動することができるし、時間の中を逆行することさえできる。


実際には、第4カルーザ-クライン空間でその3次元物体を“迂回”している。

このような船体は輝いて見えることになるだろう。それらの表面の外観や機構もまた輝いたり回転したりする光といったものに見えるだろう。
ご冗談でしょう?名無しさん [sage] 2019/02/03(日) 13:24:47.38ID:???

ここは物理板の質問スレ。
キチガイ理論は他所逝って。
ご冗談でしょう?名無しさん [sage] 2019/02/03(日) 13:26:18.06ID:???
相手すると余計につけあがるぞ
ご冗談でしょう?名無しさん [] 2019/02/03(日) 14:05:57.21:HxTc9ft1
シュレディンガーの猫で箱を開けて中を観測するまで生きているか死んでいるか分からないという話だけど
猫自身が観測者だとすると、猫は自分の死を自分で観測する事が出来ないので、猫にとって猫自身は不死身なのですか?
箱を開けた外部の観測者が猫の死を確認したとしても
猫の魂は猫が死ななかった場合のパラレルワールドに行って生きた自分を観測し続けると言う意味で・・・
ご冗談でしょう?名無しさん [sage] 2019/02/03(日) 14:09:04.07ID:???

箱の中にいる猫は観測できるじゃん。
ご冗談でしょう?名無しさん [sage] 2019/02/03(日) 14:09:44.29ID:???
パラレル世界なんてものは物理の対象外なので
ご冗談でしょう?名無しさん [sage] 2019/02/03(日) 14:20:34.61ID:???
多世界解釈ならあるがな
ご冗談でしょう?名無しさん [sage] 2019/02/03(日) 14:22:52.98ID:???
解釈は物理ではないので
ご冗談でしょう?名無しさん [] 2019/02/03(日) 14:34:58.73:HxTc9ft1

確かに猫は、放射性元素の崩壊を観測できるけど自分の死は観測できない。

ちなみにこんな質問を思いついたのは人間原理のwikiに
「宇宙が人間に適しているのは、そうでなければ人間は宇宙を観測し得ないから」なんて論が書かれていたからだ。
人間原理の人間の部分をシュレディンガーの猫に変えている。
ご冗談でしょう?名無しさん [sage] 2019/02/03(日) 14:36:35.08ID:???
猫は9個の魂を持ってるし1個くらいよくわからんようになってもいいんじゃね?
ご冗談でしょう?名無しさん [sage] 2019/02/03(日) 14:37:49.93ID:???

放射性元素の崩壊を観測=ガスの放出=よって猫が死ぬということだぞ

猫はガスを吸って胸くそ悪くなった瞬間に放射性元素の崩壊を観測している

全然的外れな意見だ。
ご冗談でしょう?名無しさん [sage] 2019/02/03(日) 14:53:07.16ID:???
この宇宙の真空がすごく不安定でも我々は真空崩壊を観測することが出来ない
その意味では我々もシュレディンガーの猫と立場は同じ。
ご冗談でしょう?名無しさん [sage] 2019/02/03(日) 15:03:49.57ID:???
観測者が死んでいるのでそもそも観測できない
ご冗談でしょう?名無しさん [sage] 2019/02/03(日) 15:26:14.50ID:???
観測者が死ぬことを物理は想定していない
ご冗談でしょう?名無しさん [sage] 2019/02/03(日) 16:39:55.84ID:???
今年に入って
まともな書き込みなし。
モーメントの質問にも答えられず。
ご冗談でしょう?名無しさん [sage] 2019/02/03(日) 16:47:23.08ID:???
モーメントの質問なんてありましたっけ?
ご冗談でしょう?名無しさん [sage] 2019/02/03(日) 16:49:53.93ID:???
しかし多世界解釈が一番素直に観測理論を説明できる
波束の収縮みたいなメカニズムを必要としない。
ご冗談でしょう?名無しさん [sage] 2019/02/03(日) 16:50:39.41ID:???
コペンハーゲン解釈でも問題ありません
ご冗談でしょう?名無しさん [sage] 2019/02/03(日) 17:19:45.62ID:???

原点の電流か磁場を振動させれば電磁波が広がっていくのがシミュレーションで見れるよ
浮動小数点演算コアで並列計算することで計算時間を短縮できる。
ご冗談でしょう?名無しさん [sage] 2019/02/03(日) 18:14:12.63ID:???
ちょっと物理とは違う質問だけど、面積の数値が確定的に求まる円って作図可能?
何が言いたいかと言うと半径1の円だと面積はPiで無理数なので数値計算すると小数点以下が無限に続く
逆に半径1/sqrt(Pi)なら面積は1になって数値的に確定する

どっちに無理数を押し込むかの差でしか無いから難しそうだけど
ご冗談でしょう?名無しさん [sage] 2019/02/03(日) 18:15:01.39ID:???
コペンハーゲン解釈は波束の収縮というメカニズムが説明できない
ご冗談でしょう?名無しさん [sage] 2019/02/03(日) 18:19:30.22ID:???

一辺1cmの正方形は作図可能ですか?
誤差とかで正確に作図するのは難しそうですけど



エヴェレット解釈もどのようにして重なり合った世界が確定するか説明できませんよね
ご冗談でしょう?名無しさん [sage] 2019/02/03(日) 18:21:08.06ID:???

あー、基準が取れないのか。
じゃあ質問をある線分を1としたときにそれ基準で1/sqrt(Pi)の線分を引けるかに変更します
ご冗談でしょう?名無しさん [sage] 2019/02/03(日) 18:24:02.10ID:???

線分を引ける、とはどのような意味ですか?

数学における作図は定規とコンパスだけを用いて線を引くことですね
ご冗談でしょう?名無しさん [sage] 2019/02/03(日) 18:24:11.65ID:???

「円積問題」だな。
作図不可能なことが証明されてる。
ご冗談でしょう?名無しさん [sage] 2019/02/03(日) 18:31:07.08ID:???

「作図可能数」でググれ。
ご冗談でしょう?名無しさん [sage] 2019/02/03(日) 18:32:54.45ID:???
コンパスと定規では二次方程式しか解けない(ある数値の長さが与えられたとき、その三乗根は作図できない)

πは超越数といっていかなる代数的方程式の解にもならない
なので無理
ご冗談でしょう?名無しさん [sage] 2019/02/03(日) 18:58:28.52ID:???

なるほど…どうもありがとうございます
ご冗談でしょう?名無しさん [sage] 2019/02/03(日) 19:18:30.37ID:???
世界が確定する必要は無い
測定を行えば観測対象と観測者の間に量子もつれが生じて
測定を確定させた観測者の重ね合わせにユニタリー発展するにすぎない。

|→>をx軸方向スピン |↑↓O(?)>をz軸方向スピンを測定する観測装置と観測者とすれば

|→>|↑↓O(?)> ⇒ 1/√2|↑>|↑↓O(↑)> + 1/√2|↓>|↑↓O(↓)>

|→>をx軸方向スピン |←→O(?)>をx軸方向スピンを測定する観測装置と観測者とすれば

|→>|←→O(?)> ⇒ |→>|←→O(→)> = 1/√2|↑>|←→O(→)> + 1/√2|↓>|←→O(→)>

どの様に時間発展するかは基底表現に依存しない。
ご冗談でしょう?名無しさん [sage] 2019/02/03(日) 19:39:56.19ID:???
系と観測者をセットにした系を考えればそれはユニタリ発展である
ご冗談でしょう?名無しさん [sage] 2019/02/04(月) 10:25:03.24ID:???

適当に円書いて、この円の面積を1と定義する、としてしまえば可能。
ただしこのときは、長さ1の線分の作図ができなくなるだけ
ご冗談でしょう?名無しさん [sage] 2019/02/04(月) 17:42:36.41ID:???
超と言えばオーバードクター
ご冗談でしょう?名無しさん [] 2019/02/04(月) 20:02:39.52:8MZ48iHh
基本的疑問なんだけど、地表から初速vと100vで同じ質量の物体を投げ上げたとき、同じ高さに到達したときには同じ運動エネルギーを失いますか?
ご冗談でしょう?名無しさん [sage] 2019/02/04(月) 20:06:10.24ID:???
その通りですよ
ご冗談でしょう?名無しさん [sage] 2019/02/04(月) 20:40:30.68ID:???
のつづき

ブラジルの物理学者フラン・デ・アキノ(Fran De Aquino)教授は、2000 年に発表された幾つかの興味深い論文 112) の中で重力場の制御により、

次のことを可能にしたと述べている: 重力場から直接エネルギーを取り出すことができた;
極低周波放射を使って重さ77ポンド、直径2フィートのドーナツ形物体を浮揚させた。デ・アキノ教授は、ブラジル・マラニャン州の州都サン・ルイス市にあるマラニャン州立大学物理学部教授である。
デ・アキノ教授は、論文がノーベル賞受賞者を含む36人の科学者により査読されたことを報告している。

サール(Searl)電気重力円盤とロシア人の実験。

英国の技術者ジョン・サールにより40年以上も前に開発されたこの装置は、分割された一つの回転円盤で構成されており、その分割部分は円周軌道の内側を転がる一連の円筒永久磁石で支持されている。

これは完全な離昇(lift off)を達成したと言われている。この数年間に,ロシア国立科学アカデミー所属の二人のロシア人科学者,ロスチン(Roschin)とゴディン(Godin)は、単純化されたサール円盤を建造し、その特異な重量減少効果を裏付けた。

彼らは、直径1メートルの円盤を毎分600回転させ、35パーセントの重量減少を実現すると同時に、7キロワットの余剰電力を発生させた。

ポドクレトノフ(Podkletnov)の重力遮蔽体とグリーングロー計画。

ポドクレトノフ博士に率いられたフィンランドの研究チームが、回転超伝導円盤で実験を行なっていた。

それは一連の電磁石が発生した反発磁場により、空中に浮揚させられていた。彼らは1996 年に、円盤は部分的に地球の重力場を遮蔽し、円盤の上方にある物体の重量を2パーセント減少させることができたと報告した。

推進の他に、重力差生成による機械的電力発生という応用があるのは明らかである。

彼らはこの研究を、非既成技術の実現可能性を調査するために彼らが立ち上げた、グリーングロー計画のもとで進めている。
ご冗談でしょう?名無しさん [] 2019/02/04(月) 20:49:44.77:8MZ48iHh

しかし…

そうすると初速100vの物体は同じ高さに到達する時間は短くなりますよね。重力加速度はどちらもgとすると、速い物体はその高さに至る時間が短いので速さの減少量も少なくなる。-gt分速さが減るわけだから。

もし同じ高さに到達するとき、初速に関係なく同じ運動エネルギーを失ってるとすると、質量は同じだから同じ速さが減少してないといけないはずが、上記の通り初速が違うと速さの減少量が違ってしまう。

これだと、ポテンシャルや保存則は成り立ちますか?
ご冗談でしょう?名無しさん [sage] 2019/02/04(月) 20:57:39.06ID:???
宇宙は真空を含めて正の電荷と負
の電荷を持つ素粒子、すなわち電子、陽電子、陽子、反陽子の密な結晶体の集まりであると理解すべきと言っています。

Medel Sachsの理論を発展させた米国のMarcus S. Cohenは8個のスピノルからなる宇宙のセルの集合が宇宙全体を作り、そのセルがその中にエネルギーを保持する状態なると粒子化するとしています。

8個のスピノル、例えて言えば8個のクォークが基となって宇宙が
構成され、8個のクォークから成るセルが格子点となってネットワ
ークとしての宇宙が構成されているとしています。

セルの中や格子点を繋ぐ力の源泉が非線形の磁場であるスピン磁場です。スピン磁場は粒子のスピンと不可分に結びついています。
格子点を伝わる弾性波動的なもののうち、横波を電磁波と呼んでいます。スピン磁場自体は縦波であり、この弾性波動を構成する一要素です。
スピン磁場の縦波の波動がスピン磁気量子波動であり、原子核の周囲の電子が永久運動するように見えるものスピン磁気量子波動がエネルギーを与えているからです。

現代科学はそのことに気がついていません。
ご冗談でしょう?名無しさん [sage] 2019/02/04(月) 20:59:31.82ID:???
なんでキチガイって改行がキチガイなの?
なんかそういうキチガイ宗教でもあるの?
ご冗談でしょう?名無しさん [sage] 2019/02/04(月) 21:00:55.67ID:???
光を打ち消すとエネルギーはどうなるのでしょうか?
ご冗談でしょう?名無しさん [sage] 2019/02/04(月) 21:04:04.23ID:???
「光を打ち消す」というのを物理の言葉で定義すれば答えが得られるかもしれません。
ご冗談でしょう?名無しさん [sage] 2019/02/04(月) 21:28:00.03ID:???

>もし同じ高さに到達するとき、初速に関係なく同じ運動エネルギーを失ってるとすると、質量は同じだから同じ速さが減少してないといけないはず
(100v)^2 - (100v - Δv)^2 = v^2 - (v - Δv)^2
同じ速さが減少するならこれが任意のΔvで成立することになるが本当か?

同じ高さに到達するときの運動エネルギーの減少量は同じだが
初速によって速度の減少量は違う
ご冗談でしょう?名無しさん [] 2019/02/04(月) 22:52:32.55:8MZ48iHh

運動エネルギーをどう定義するのかわからなくなった。

FtとFxは本質的に同じものだと思っていたけど、どうも違うのかな。ポテンシャルを考える場合、Fxこそ本質的でFtは違うということなんだろうか。

見方が変わるなぁ。
ご冗談でしょう?名無しさん [sage] 2019/02/04(月) 23:02:02.92ID:???
見えもしないクォークの話して何が楽しいの?
ご冗談でしょう?名無しさん [] 2019/02/04(月) 23:05:40.92:8MZ48iHh
場が物体に仕事をすると見るときの力Fが一定時間t作用したとき。

静止物体を力Fで時間t押したとき。

これは同じ運動エネルギーを与えるか失わせるかするんじゃないの?
ご冗談でしょう?名無しさん [sage] 2019/02/04(月) 23:12:09.67ID:???
ポテンシャルと運動エネルギーは一体であって、
単独で考えるようなことするから混乱してるだけ。

運動エネルギーはどうやって定義されてるのか、ポテンシャルも合わせて
大学教養課程の物理本を読めば済む話で実にくだらない。

モーメントの問題は
F1L1>F2L2なら、どういう理由でF1側に回転するのかだったね。
こっちのほうがはるかに重要だね。答え知ってるけど、高校物理だね。

もっとも、教科書見てもはっきりと書いてないから
教育の欠陥だねこれ。だからまともに答えられない連中ばっか。
ご冗談でしょう?名無しさん [sage] 2019/02/04(月) 23:19:07.65ID:???
わからないんですね
ご冗談でしょう?名無しさん [] 2019/02/04(月) 23:29:24.29:8MZ48iHh
初速の違う物体が投げ上げられて同じ高さに至る時間は違う。質量が同じなら、加わる重力は同じだから、力積は違ってくる。

ここである速さの物体を静止させることを考えるとき、力Fが時間tかかって距離x進んだところで静止したとすると、Fx=1/2mv^2
これを運動エネルギーの定義とみてるんだけど、

最初の話に戻って初速が違うとFtが違うわけだから、運動エネルギーの得喪の量も違ってくるんじゃないの?

高さが同じなら運動エネルギーの量も同じとすると、本質的な運動エネルギーの意味がわからなくなる。
ご冗談でしょう?名無しさん [sage] 2019/02/04(月) 23:34:17.22ID:???
モーメントとわからないんですねは別人か
一体何人キチガイが住んでるんだ
ご冗談でしょう?名無しさん [sage] 2019/02/04(月) 23:37:32.77ID:???

(初速の自乗 ー 一定高度Hまで上昇した時点での速度の自乗)

これはHまで上昇できるだけの初速があれば、初速に関わらず一定。エネルギー保存則よりただちに導かれる

初速 ー 一定高度Hまで上昇した時点での速度

これは別に保存しない(保存則無いし)。速度に依存して変化するよ。
ご冗談でしょう?名無しさん [sage] 2019/02/04(月) 23:42:01.81ID:???

運動量とエネルギーを区別しましょうね

Ftの力積は運動量に関する話、Fxの仕事はエネルギーに関する話です
ご冗談でしょう?名無しさん [sage] 2019/02/04(月) 23:42:46.30ID:???
運動量保存が使えるのは外力が無いときだけ。
今回は高さ方向について運動量保存は成り立たない(重力があるからね)ので運動量保存則は使えない。

なのでエネルギー保存則しか使えない。
エネルギー保存則で解きたいなら速度は自乗(2乗)しないと意味がないぞい。

それが嫌なら運動方程式を立てることじゃ。

運動方程式、運動量保存則、エネルギー保存則

この3つしか解き方はなくて、これらが使える時と使えない時の区別を付けないといけないなりね。

身ににつくまでで難しいのは運動量保存則使える時はどういう時かかな。

あなたのレベルだと教科書とか高校レベル参考書でも勉強になるはずなので、そういうのをバカにせず読んでみることだね。
「わかりやすい高校物理の部屋」のサイトなんかおすすめ
ご冗談でしょう?名無しさん [sage] 2019/02/04(月) 23:47:49.92ID:???
運動量とエネルギーが別物っていうのも慣れるまでは難しいよね。

運動量に対応するのは力積。
エネルギーに対応するのは仕事。

運動量と力積は単位が一緒(N・s=重さ*速度、基本的に力学の問題でしか出てこない)

エネルギーと仕事は単位が一緒(J=N・m=力*距離
ジュールは電気E、位置E、核E、ありとあらゆるエネルギーの単位でもある)

これらはそもそも単位が違うので、例えば運動量と位置エネルギーを足し算するのは1リットルと1メートルを足すようなものでなんの意味もない。

この辺教科書で教えればいいのに教えないんだよね。
物理の先生はこういうとこで躓く学生いるってわからないのかな?
ご冗談でしょう?名無しさん [sage] 2019/02/04(月) 23:57:42.00ID:???
普通の教科書読む知能があればわかると思いますけど
ご冗談でしょう?名無しさん [] 2019/02/05(火) 00:01:27.85:DYVpJDdW
じゃあFxとは何ですか?
運動する物体を静止させることを基準にして定める運動に関するエネルギーじゃないんでしょうか。

そのときFtとFxは一定の関係を持つわけで、それが関係ないとなると、運動エネルギーとは何ですか?どうやって定めるものですか、力学的に?
ご冗談でしょう?名無しさん [sage] 2019/02/05(火) 00:03:17.05ID:???
これは、落ちこぼれだな。
ご冗談でしょう?名無しさん [sage] 2019/02/05(火) 00:21:04.64ID:???

Fxは仕事ですね

Ftは力積ですね

運動エネルギーはmv^2/2ですね

運動量はmvですね

与えられた仕事=エネルギー変化

与えられた力積=運動量変化


証明は大学入ったら勉強できます
高校の証明は、特定の場合に限って証明してるので、おそらくあなたはそれらの概念が限定的なものだと思ってしまっているのではないでしょうか

証明は一旦忘れて、上に書いたことが分かれば良いのですよ
ご冗談でしょう?名無しさん [sage] 2019/02/05(火) 01:34:56.13ID:???

を読んでみて、それから質問してくれ。

*の記号は掛けるという意味だ。×と同じ

Fxの単位はジュール=ニュートンメートル=重さ*加速度、
Ftの単位はニュートン秒=重さ*速さで、
単位が違うので足し引きでは結ばれない。

運動量pとエネルギーEとは、他の変数との掛け算や割り算によって結ばれる。

例:質量mの物体が速度vで運動する時
運動量p=mv(単位はニュートン*秒)
運動エネルギーE=mv^2/2(単位はニュートン*メートル)

この場合例えばE=p^2/2mなどと書けるが、これはEとpの間にいつも成り立つわけではない。

例えばEが電気エネルギー、位置エネルギーなどではこういう関係はもちろん成り立たない。
ご冗談でしょう?名無しさん [sage] 2019/02/05(火) 01:44:18.64ID:???
すまん、大事なとこ書き忘れた
エネルギー=ニュートン*メートル=重さ*加速度*距離

最後の*距離が抜け落ちてた
ご冗談でしょう?名無しさん [] 2019/02/05(火) 02:28:56.81:NzGx2lKC
ヒカキンの年収が10億超え!?明石家さんま・坂上忍も驚愕の総資産とは??
ttp://https://logtube.jp/variety/28439
【衝撃】ヒカキンの年収・月収を暴露!広告収入が15億円超え!?
ttp://https://nicotubers.com/yutuber/hikakin-nensyu-gessyu/
HIKAKIN(ヒカキン)の年収が14億円!?トップYouTuberになるまでの道のりは?
ttp://https://youtuberhyouron.com/hikakinnensyu/
ヒカキンの月収は1億円!読唇術でダウンタウンなうの坂上忍を検証!
ttp://https://mitarashi-highland.com/blog/fun/hikakin
なぜか観てしまう!!サバイバル系youtuberまとめ
ttp://http://tokyohitori.hatenablog.com/entry/2016/10/01/102830
あのPewDiePieがついに、初心YouTuber向けに「視聴回数」「チャンネル登録者数」を増やすコツを公開!
ttp://http://naototube.com/2017/08/14/for-new-youtubers/
27歳で年収8億円 女性ユーチューバー「リリー・シン」の生き方
ttp://https://headlines.yahoo.co.jp/article?a=20170802-00017174-forbes-bus_all
1年で何十億円も稼ぐ高収入ユーチューバー世界ランキングトップ10
ttp://https://gigazine.net/news/20151016-highest-paid-youtuber-2015/
おもちゃのレビューで年間12億円! 今、話題のYouTuberは6歳の男の子
ttp://https://www.businessinsider.jp/post-108355
彼女はいかにして750万人のファンがいるYouTubeスターとなったのか?
ttp://https://www.businessinsider.jp/post-242
1億円稼ぐ9歳のYouTuberがすごすぎる……アメリカで話題のEvanTubeHD
ttp://https://weekly.ascii.jp/elem/000/000/305/305548/
世界で最も稼ぐユーチューバー、2連覇の首位は年収17億円
ttp://https://forbesjapan.com/articles/detail/14474
ヒカルの収入が日収80万、月収2400万、年収3億と判明www
ttp://https://matomenewsxx.com/hikaru-income-8181.html
はじめしゃちょーの年収は6億?2017年は30億突破か?
ttp://https://2xmlabs.com/archives/1873
ご冗談でしょう?名無しさん [] 2019/02/05(火) 10:01:01.91:p625d/Zw
化学科の熱力学で熱機関のウエイト重すぎません?
化学ポテンシャルとか活量の方がよほど重要なのに最後にちょこっとやるだけのテキストや授業ばかりなのはなんででしょう?
まずは使えるようになってから背景としての基礎知識を仕入れた方が良い気がします
ご冗談でしょう?名無しさん [sage] 2019/02/05(火) 10:04:02.72ID:???
すみません板間違えました
ご冗談でしょう?名無しさん [sage] 2019/02/05(火) 12:06:17.34ID:???
物理と化学じゃ関心は違くて当たりまえ
物理ではヘルムホルツエネルギーの方をメインにやるが化学ではギブズエネルギーらしいし
ご冗談でしょう?名無しさん [] 2019/02/05(火) 12:18:38.61:P8/x5XCs
【30日、東海村で放射能漏れ事故】 鼻から出血ツイート多いな、こんな症状出てもまだ気がつかないのか
ttp://rosie.5ch.net/test/read.cgi/liveplus/1549162998/l50
ご冗談でしょう?名無しさん [] 2019/02/05(火) 17:48:36.52:2Ek3tYg5
体積を一定とした方が考えやすい人と圧力を一定とした方が考えやすい人の違い
ご冗談でしょう?名無しさん [sage] 2019/02/05(火) 17:59:52.29ID:???
物理ではエネルギーでしょ
ご冗談でしょう?名無しさん [sage] 2019/02/05(火) 18:02:35.43ID:???
わからないんですね
ご冗談でしょう?名無しさん [sage] 2019/02/05(火) 18:08:55.09ID:???
物理量って結局一体何なんだ?
冷静になると力Fだって何なのか正体がよく分からない
ご冗談でしょう?名無しさん [sage] 2019/02/05(火) 18:51:23.45ID:???
人間が測ることのできる量の組み合わせ
ご冗談でしょう?名無しさん [sage] 2019/02/05(火) 20:11:36.78ID:???
まったくその通り
ご冗談でしょう?名無しさん [sage] 2019/02/05(火) 20:50:31.98ID:???
測定できない物理量なんて山ほどあるわけだが
ご冗談でしょう?名無しさん [sage] 2019/02/05(火) 21:17:54.38ID:???
可測定と可制御の双対性
ご冗談でしょう?名無しさん [sage] 2019/02/05(火) 21:19:14.88ID:???

お外に出てきちゃいけないゴーストはBRSTコホモロジーで形式的に特定できる。
ご冗談でしょう?名無しさん [sage] 2019/02/05(火) 21:50:36.52ID:???
世界の本質が物理学なんですか?
物理世界が世界の近似表現なんですか?

〇〇という仮定のもとに予測される結果が、現実とよくマッチする
というところまでは良いとして、
そこから現実世界にも〇〇というものが存在している
というところまで物理学者は考えているのですか?
ご冗談でしょう?名無しさん [sage] 2019/02/05(火) 22:26:00.91ID:???

ある仮定がどうして成り立つか
例えばどうして運動方程式が成り立つのか
どうしてシュレディンガー方程式が成り立つのか
どうして相対性原理が成り立つのか
そういうのを考えるのはやはり不毛
だってそれを議論するにしたってまた新たな原理を考えなければならない

数学で言う1,2などの数字や、その演算法をはじめに定義しなければなんもできないのと一緒
ご冗談でしょう?名無しさん [sage] 2019/02/05(火) 22:26:23.19ID:???

視野狭すぎて笑う
ご冗談でしょう?名無しさん [sage] 2019/02/05(火) 22:35:35.86ID:???
重力という力を仮定するとリンゴの運動を精度よく再現できる時、重力は現実に存在するか否か
ご冗談でしょう?名無しさん [sage] 2019/02/05(火) 22:39:42.94ID:???

もっと具体的に突っ込んでこっちもお勉強になるような中身のあるレスしてよ
ご冗談でしょう?名無しさん [sage] 2019/02/05(火) 22:42:06.48ID:???

重力の存在と矛盾するような現象が観測されるまでは正しいと考えるのが普通
ご冗談でしょう?名無しさん [sage] 2019/02/06(水) 01:20:10.33ID:???

数学ではそういうのを公理と言うんだよ
ご冗談でしょう?名無しさん [sage] 2019/02/06(水) 01:22:14.83ID:???
物理ではそういうのを原理というんですよ
ご冗談でしょう?名無しさん [sage] 2019/02/06(水) 01:44:21.52ID:???

なわけねーよ
時空間でさえ本当は存在しないと言う予想が常識
ご冗談でしょう?名無しさん [sage] 2019/02/06(水) 01:45:46.44ID:???
時空間が存在する、もしくは存在しない、とはどのようなことですか?
ご冗談でしょう?名無しさん [sage] 2019/02/06(水) 05:12:10.46ID:???
「モーメントの問題は
F1L1>F2L2なら、どういう理由でF1側に回転するのかだったね。」

これってどうやって証明すんの?
今まで当たり前だと思っていたけど、そんなことないと気付いたはw
ご冗談でしょう?名無しさん [sage] 2019/02/06(水) 06:01:54.33ID:???
モーメント=角運動量変化ですから当たり前ですね
ご冗談でしょう?名無しさん [sage] 2019/02/06(水) 06:03:39.99ID:???
Idω/dt=Nからすぐ分かるでしょ
これの導出は大抵の力学の教科書に載ってるだろう
ご冗談でしょう?名無しさん [sage] 2019/02/06(水) 08:06:57.40ID:???


矛盾していますね
ご冗談でしょう?名無しさん [sage] 2019/02/06(水) 12:57:50.43ID:???
量子力学に隠れ変数は存在しないということになっているが
その前提となっている時空の存在が揺らぐと怪しくなる
ご冗談でしょう?名無しさん [sage] 2019/02/06(水) 13:05:38.38ID:???
角運動量の固有値ってどうして離散的なんですか?
角運動量演算子の交換関係とエルミート性のみから昇降演算子で固有値問題を解く奴、最初から固有値が離散的な事を仮定してませんか?
ご冗談でしょう?名無しさん [sage] 2019/02/06(水) 13:32:10.93ID:???
束縛されてれば離散的
空間が有限次元だからだね
ご冗談でしょう?名無しさん [sage] 2019/02/06(水) 13:41:38.72ID:???

時空間座標 (t,x,y,z) で表されるような物理的対象は存在しないと言うこと
量子群で群が存在せず群上の数学的構造の類似だけが存在するのと同様さ
それを何らかの近似を行えば時空間上の記述と見ることができるだけ
ご冗談でしょう?名無しさん [sage] 2019/02/06(水) 13:50:28.91ID:???

「正しい」は結果が正しいと言うこと
存在など関係ないし、今の物理学じゃ問題にもならない
ご冗談でしょう?名無しさん [sage] 2019/02/06(水) 16:29:45.18ID:???
ムニちゃーん ダンス
ご冗談でしょう?名無しさん [sage] 2019/02/06(水) 18:25:08.88ID:???

>時空間座標 (t,x,y,z) で表されるような物理的対象は存在しないと言うこと

それは、背景独立じゃない量子力学、場の量子論等の話。
相対性理論は時空そのものが研究対象。
ご冗談でしょう?名無しさん [sage] 2019/02/06(水) 19:11:36.40ID:???
電磁波が1/4λの波長で反射し電磁波のさざ波を起こさない状態は何が観測されるのでしょうか?
ご冗談でしょう?名無しさん [sage] 2019/02/06(水) 19:18:49.81ID:???
さざ波とは
ご冗談でしょう?名無しさん [sage] 2019/02/06(水) 19:19:39.47ID:???

角運動量が束縛されているとはどう言う意味ですか?
ご冗談でしょう?名無しさん [sage] 2019/02/06(水) 19:22:10.56ID:???

一般に「さざ波」とは液体表面の表面張力波のことだが、電磁波の表面張力波って何だろうね?
ご冗談でしょう?名無しさん [sage] 2019/02/06(水) 20:54:23.29ID:???

だから何だ
研究対象なら存在すると思ってんのか
ご冗談でしょう?名無しさん [sage] 2019/02/06(水) 21:00:21.65ID:???

角運動量自体が束縛されてるのではなく双対の角変位が有界
空間位置が有界かどうかで双対の運動量が離散か連続になるのと同様
ご冗談でしょう?名無しさん [sage] 2019/02/06(水) 21:19:14.81ID:???

なんだ、唯のアホか。
ご冗談でしょう?名無しさん [sage] 2019/02/06(水) 22:36:15.10ID:???
独自用語の羅列は創作板でやってほしい
ご冗談でしょう?名無しさん [sage] 2019/02/06(水) 23:49:44.34ID:???
正しい用語のcheckが面倒だから知ってるんなら書け
ご冗談でしょう?名無しさん [sage] 2019/02/07(木) 00:31:40.38ID:???
定在波の全体が0になる瞬間の話?
ご冗談でしょう?名無しさん [sage] 2019/02/07(木) 00:33:27.55ID:???
なーんか一部の統合失調症装った言葉のサラダの実際の目的は検索エンジン混乱させるためのチャフのような気がしてきた。
ご冗談でしょう?名無しさん [sage] 2019/02/07(木) 01:21:38.72ID:???

角変位が有界とはどういうことですか?
角度って0から2πで有界じゃないんですか?
ご冗談でしょう?名無しさん [sage] 2019/02/07(木) 01:48:28.11ID:???

角運動量を交換子で定義した際は、その量が何らかの回転操作に関係してほしいという気持ちがあって、それ故に離散的な固有値が望まれるって事ですかね
それで結果的に出て来た半整数の方はスピノル空間におけると回転だと
ご冗談でしょう?名無しさん [sage] 2019/02/07(木) 02:03:24.93ID:???
証明もう一度読んでみたらどうですか?
離散であることを最初から仮定してなんてないですよ?
角運動量は位置と運動量の積で、それらは互いに交換しないから離散になるとか変なこと起こってもいいですよね
不確定性原理です
ご冗談でしょう?名無しさん [sage] 2019/02/07(木) 02:25:05.49ID:???

エネルギーの双対は時間だと思いますが時間が有界でなくてもエネルギーは離散化しますね?
ご冗談でしょう?名無しさん [sage] 2019/02/07(木) 02:49:38.26ID:???

例えばJ^2の固有値をaとJ_zの固有値bに対して
a≧b^2を示すのに両辺をノルムで割る時に
固有値が連続(即ちノルムが無限大)の場合も上手く処理出来たとしても
この不等式からだとbに上下限が存在する事しか言えなくないですか?
離散だとそのまま最小値最大値の存在は言えると思いますが
ご冗談でしょう?名無しさん [sage] 2019/02/07(木) 02:55:55.99ID:???
連投すみません
確かにbの上限j'十分近いbの取り得る値をjと置けば以下同じ議論でjが離散である事は示せる気はして来ました
ご冗談でしょう?名無しさん [sage] 2019/02/07(木) 11:31:30.31ID:???
質量の単位kgのgと
重力加速度をgとするとかのgは全く別物と考えていいの?

それとも質量のkgってk(1000)*g(重力加速度)の意味が隠れてたりする?
グラムという全く別のモノ?
ご冗談でしょう?名無しさん [sage] 2019/02/07(木) 11:37:27.76ID:???
重力加速度のgはgravityのgだと思いますよ
ご冗談でしょう?名無しさん [sage] 2019/02/07(木) 11:46:08.65ID:???

重力加速度は文字通り加速度であり、その単位はm/s^2で、質量の単位とは全くの別物

グラムはギリシャ語で「わずかな重量」を意味するgrámmaから
重力加速度をgで表すのはgravity
ご冗談でしょう?名無しさん [sage] 2019/02/07(木) 11:46:57.30ID:???

別物
斜体と立体くらい完全に別物
ご冗談でしょう?名無しさん [sage] 2019/02/07(木) 12:08:10.29ID:???
質量は加速されにくさとも言えるので全く無関係ではないけど質量の単位であるgと重力加速度のgは別物
すでに紹介されているけどたまたま頭文字が同じだっただけ
ご冗談でしょう?名無しさん [sage] 2019/02/07(木) 12:37:36.72ID:???

単位が違う
重力加速度のgはm/ss(メートル毎秒毎秒)

質量のkgはkgという質量のSI基本単位
ご冗談でしょう?名無しさん [sage] 2019/02/07(木) 12:46:22.60ID:???
簡単な質問には面白いほど同じ回答が何個もつくんですね
ご冗談でしょう?名無しさん [sage] 2019/02/07(木) 12:46:59.69ID:???
電圧は量記号も単位も両方Vだからややこしい
ご冗談でしょう?名無しさん [sage] 2019/02/07(木) 13:06:49.15ID:???

充分な情報は出てるから自分で考えろよ
ご冗談でしょう?名無しさん [sage] 2019/02/07(木) 14:09:58.52ID:???
昇降演算子は演算子の交換関係しか使ってないんじゃないの
ご冗談でしょう?名無しさん [sage] 2019/02/07(木) 16:30:42.38ID:???



みんなありがとん
ご冗談でしょう?名無しさん [sage] 2019/02/07(木) 16:32:22.86ID:???


のお二方もありがとう
ご冗談でしょう?名無しさん [sage] 2019/02/07(木) 18:55:45.86ID:???
グラスマン数の積分とか意味分かんないのですが
ご冗談でしょう?名無しさん [sage] 2019/02/07(木) 21:34:19.82ID:???
わからないんですね
ご冗談でしょう?名無しさん [sage] 2019/02/07(木) 21:39:07.34ID:???
ぶっちゃけこのスレ見てると
気違いの演技をしてるだけの気違いについて詳しくなれるよね
ご冗談でしょう?名無しさん [sage] 2019/02/07(木) 22:46:25.71ID:???
わかんにゃい
ご冗談でしょう?名無しさん [sage] 2019/02/08(金) 14:03:45.83ID:???

基礎をおろそかにしちゃダメよ
ご冗談でしょう?名無しさん [] 2019/02/08(金) 16:51:00.30:XNLnGwfW
基礎は山の中
ご冗談でしょう?名無しさん [sage] 2019/02/08(金) 17:11:05.81ID:???
数学も物理も基礎が一番難しいけどな

とりあえず計算して応用するだけなら簡単とは言わないけど、そこまでじゃないし
ご冗談でしょう?名無しさん [sage] 2019/02/09(土) 09:49:51.88ID:???
宇宙は加速しながら膨張しているそうですが
そんなことが可能ってことは膨張する「スペース」が元々「宇宙の外側」にあるってこと??
菩薩@太子 [] 2019/02/09(土) 13:29:51.50:eGVKkbQa
昔つーか、20年ばかり前のことだが、ヤフー掲示板にスレ立てて「永久機関をつくってる。もうすぐできる」というやつがいたけど、その後永久機関が完成したという話はきかないし、どうなったの?
やつはいま何してるの?

誰か知ってたら教えてくれ。ふと思い出したものでね・・・我はしつこいぞ。何十年前のことでもほじくり返す・wwwww
ご冗談でしょう?名無しさん [sage] 2019/02/09(土) 13:30:42.42ID:???
当局に消されたよ
ご冗談でしょう?名無しさん [sage] 2019/02/09(土) 13:34:52.76ID:???
トンデモなんぞ消す必要もない
ご冗談でしょう?名無しさん [sage] 2019/02/09(土) 13:39:29.98ID:???
ふと自分が何故熱やエネルギーが保存するのかを知らないことに思い至り咽び泣く

直感的に明らかな法則ほど深く追求すると難しい
ご冗談でしょう?名無しさん [sage] 2019/02/09(土) 13:42:50.88ID:???
人類は,現在は秘密にされているエネルギー発生と反重力推進の装置を実際に所有しており、それらは,現在用いられているエネルギーと輸送システムのあらゆる形態を完全かつ永久に無用のものとすることができる。

それらの装置は空間中の電磁気と、いわゆるゼロポイント・エネルギーと呼ばれる状態に作用し、いかなる汚染物質をも発生させずに巨大なエネルギーを生み出す。

本質的にこのようなシステムは,遍在する量子真空エネルギー状態、つまり、あらゆるエネルギーと物質が生じる基底エネルギー状態を利用してエネルギーを発生する。

すべての物質とエネルギーを支えるのはこの基底エネルギー状態であり,特別な電磁気回路と仕掛けを使えば,我々を取り巻く周囲の空間/時間から巨大なエネルギーを引き出すことが可能なのである。

これらはいわゆる永久機関ではないし、熱力学の法則にも反しない。
ただ我々の周囲に遍在するエネルギー場に作用して、エネルギーを発生するのである。
ご冗談でしょう?名無しさん [sage] 2019/02/09(土) 14:07:34.13ID:???
さざ波はリップル、
時間変化せず、
全体的に0であることを示します。
ご冗談でしょう?名無しさん [sage] 2019/02/09(土) 14:14:44.99ID:???

ここ質問スレだから、オカルトはオカルト板逝って。
ご冗談でしょう?名無しさん [sage] 2019/02/09(土) 18:10:25.62ID:???
複数の物理の本を読むとき、
@1冊読んでからもう1冊読む
A同時に2冊を平行作業で読む
のどっちが効率が良いか?答えよ
ご冗談でしょう?名無しさん [sage] 2019/02/09(土) 18:11:16.66ID:???
本による。以上
ご冗談でしょう?名無しさん [sage] 2019/02/09(土) 18:13:38.81ID:???
普段はどうしてんるだよ?答えよ
ご冗談でしょう?名無しさん [sage] 2019/02/09(土) 20:39:32.10ID:???
場の理論みたいな数学的によく定式化されてなくて気持ち悪い理論ってどうやって勉強してる?
議論しようにもあちこち気になって脳みそが発散する
ご冗談でしょう?名無しさん [sage] 2019/02/09(土) 21:02:41.57ID:???

つ「東大読書」
ご冗談でしょう?名無しさん [sage] 2019/02/09(土) 21:14:34.20ID:???
ムニちゃーん しんぷ
ご冗談でしょう?名無しさん [] 2019/02/10(日) 01:48:39.40:o7TYWjWe
最近オカルトをありうると感じてるんだけど、物理学徒の方々はどうでしょうか。

科学は対象化可能な領域を扱うだけで、対象化不可能な領域は扱えず、かつ否定もできない。そんな気がしてきて、どうも人間が再現不可能な現象もありうるのではないか、と思うようになってるんですよね。
ご冗談でしょう?名無しさん [sage] 2019/02/10(日) 03:50:16.10ID:???
まあ、オカルトが科学になったら今までの習ったことが無駄になる。
ご冗談でしょう?名無しさん [sage] 2019/02/10(日) 03:51:52.43ID:???
もし、あれが本物だとすると間違いなく現在の科学技術は使われなくなる領域が出てくるだろうね。
ご冗談でしょう?名無しさん [sage] 2019/02/10(日) 07:54:50.12ID:???
心理学的な意味では「本物」だろう
ご冗談でしょう?名無しさん [sage] 2019/02/10(日) 07:58:48.75ID:???
再現性のない現象は科学の対象外です
寝言は寝て言ってね
ご冗談でしょう?名無しさん [sage] 2019/02/10(日) 08:38:46.62ID:???
元来、世界観を根本的に変えるようなのは批判され、理解されにくく、現状からは生まれることはない。

それはマクスウェルだったり、ニュートンだったり、ガリレオガリレイだったりする。

平賀源内は、天才だったが故に周囲の人に理解されにくく最後は獄死したけど、倉田大嗣もその人かな

以下の実験で明らかにしたらしいな

イ)瞬時に水を油に変えることが出来た。水の分解で水素を得て、空気中の窒素を炭素に変えて、これらを化合させて作る。

ロ)三重水素水を10年以上崩壊させなかった。

ハ)炭化水素を構成する原子に分解し、テンプレート触媒を使って任意の炭化水素分子に組み替えることができた。

ニ)磁気波動を使って144番目までの新しい元素を作った。

ホ)ダイヤモンドを144面体カットすると、全ての放射光が渦を作ることがわかった。

ヘ)380℃で水を波動分解し、水素酸素分解燃焼させることができた。

ト)重力の制御が小規模にできた。コップを浮かせることが出来る。

チ)5kWの永久磁石によるフリーエネルギーモータを10年以上走行させてきた。

その人はその後、行方不明になったらしいけど消されたんだろうなぁ
ご冗談でしょう?名無しさん [sage] 2019/02/10(日) 09:00:32.41ID:???
倉田大嗣による非線形電磁気学と波動科学
ttp://http://www.geocities.co.jp/Technopolis/3684/kurata.pdf
ご冗談でしょう?名無しさん [hage] 2019/02/10(日) 11:49:51.05ID:???
心理学的な再現性ならある
ご冗談でしょう?名無しさん [sage] 2019/02/10(日) 12:35:35.05ID:???
心理学が科学かどうかはまた別だけどね(個人的には科学だと思ってるけど)
それにレアな科学的事象を錯覚してる場合もあるし(鬼火とか)一概に科学と無関係とも言い切れん
ご冗談でしょう?名無しさん [] 2019/02/10(日) 12:45:57.93:mNnbHn0E
【アイドル恋愛禁止問題】 自由″主義は皮肉にも、営業効率のため、労働者に厳しい統制″を強いる
ttps://rosie.5ch.net/test/read.cgi/liveplus/1549768675/l50
ご冗談でしょう?名無しさん [sage] 2019/02/10(日) 13:30:00.17ID:???
宇宙論は現象の再現性ないけど観測の再現性あるから科学だぜ
そもそも現象の再現性なくて認識できるんかな
ご冗談でしょう?名無しさん [] 2019/02/10(日) 13:30:11.88:+s8IsG2Q
熱力学第零法則の本質とはなんですか?
ご冗談でしょう?名無しさん [sage] 2019/02/10(日) 17:39:30.34ID:???

ちゃんと関数解析をフォローし切って場以前第二量子化以前の量子論をまるっきり理解できてますって言い切れる自信があるなら大秀才だなオマエ。
ご冗談でしょう?名無しさん [sage] 2019/02/10(日) 18:16:30.53ID:???
むしろしてない人がいるというのが驚きですね
数学の基礎がわからずに適当に計算してるってことですよね
ご冗談でしょう?名無しさん [sage] 2019/02/10(日) 18:19:36.79ID:???
分かった気になって計算する
ご冗談でしょう?名無しさん [sage] 2019/02/10(日) 18:20:14.42ID:???
わからないんですね
ご冗談でしょう?名無しさん [sage] 2019/02/10(日) 18:36:47.61ID:???

ゲージ場も一応古典場っちゃあ古典場なんだ
馬鹿なババァは知らんかもしれんが
ご冗談でしょう?名無しさん [sage] 2019/02/10(日) 18:47:08.58ID:???
とういうことは、ゲージ場とかいうのもここの人たちはわかってないということですね
ご冗談でしょう?名無しさん [sage] 2019/02/10(日) 19:04:25.64ID:???
劣等感婆さんは今年再受験するんでしたっけ?
勉強しなくていいんですか?
ご冗談でしょう?名無しさん [sage] 2019/02/10(日) 19:13:58.00ID:???
センター受け忘れちゃったのでまた来年ですね
ご冗談でしょう?名無しさん [sage] 2019/02/10(日) 19:24:36.07ID:???
どこ受けるつもりだったんですか?
ご冗談でしょう?名無しさん [sage] 2019/02/10(日) 19:27:05.03ID:???
それでグラスマン数の方は…
ご冗談でしょう?名無しさん [sage] 2019/02/10(日) 19:53:13.60ID:???

本当に忘れたのなら認知症の検査をしてみては?
ご冗談でしょう?名無しさん [sage] 2019/02/10(日) 20:02:41.81ID:???

┐( ´〜`)┌
ご冗談でしょう?名無しさん [sage] 2019/02/10(日) 21:17:43.15ID:???

超対称性を前提にして第二量子化以前の問題で量子論教えた方が数学的には洗練されてるよね
数学嫌いの物理屋さんの中には激怒しそうな手合も居るけど。
ご冗談でしょう?名無しさん [sage] 2019/02/10(日) 22:27:04.89ID:???
めんどくせー
ご冗談でしょう?名無しさん [sage] 2019/02/10(日) 22:43:37.82ID:???

温度が一定になることです
ご冗談でしょう?名無しさん [sage] 2019/02/10(日) 22:44:51.42ID:???
平衡状態が存在することだぞ
ご冗談でしょう?名無しさん [sage] 2019/02/10(日) 23:08:03.79ID:???
わからないんですね
ご冗談でしょう?名無しさん [sage] 2019/02/11(月) 10:52:00.69ID:???

平衡状態が一意に定まることが本質だ
ご冗談でしょう?名無しさん [sage] 2019/02/11(月) 11:52:03.34ID:???

違います
ご冗談でしょう?名無しさん [sage] 2019/02/11(月) 13:00:26.99ID:???

基点を決めただけさ
ご冗談でしょう?名無しさん [sage] 2019/02/11(月) 15:12:58.30ID:???
基点とはなんですか?
ご冗談でしょう?名無しさん [sage] 2019/02/11(月) 15:27:44.43ID:???
温度という示強変数の存在証明が第0法則
ご冗談でしょう?名無しさん [sage] 2019/02/11(月) 15:41:17.71ID:???
いいえ
ご冗談でしょう?名無しさん [sage] 2019/02/11(月) 16:47:31.69ID:???
熱平衡が推移律を満たすという法則が熱力学第零法則
ご冗談でしょう?名無しさん [sage] 2019/02/11(月) 17:08:53.58ID:???

ググってんじゃねえよ
ご冗談でしょう?名無しさん [sage] 2019/02/11(月) 17:23:59.04ID:???
わからないんですね
ご冗談でしょう?名無しさん [sage] 2019/02/11(月) 18:46:22.67ID:???
量子の場合は出来ないんだけど
古典的な実体であるサイコロって全ての初期条件を入力すれば出目の予測って出来るんじゃないの?
量子における確率と古典的な現象レベルでの確率表現って本質的な差異がある気がするんだけど
ご冗談でしょう?名無しさん [sage] 2019/02/11(月) 18:57:27.04ID:???
ムーニーちゃんしんぷだい
ご冗談でしょう?名無しさん [sage] 2019/02/11(月) 18:58:51.38ID:???
AとBが熱平衡であるとはどういうことですか?
ご冗談でしょう?名無しさん [sage] 2019/02/11(月) 19:49:21.72ID:???

決定論的にはそうだけど
カオス理論的には人間がその予測をすることは出来ない(ことが多い)
ご冗談でしょう?名無しさん [sage] 2019/02/11(月) 22:26:17.61ID:???
熱的平衡状態が存在し、それを異なる系で比較できることを述べたのが第0法則
ご冗談でしょう?名無しさん [sage] 2019/02/12(火) 00:02:48.33ID:???

0点だな
ご冗談でしょう?名無しさん [] 2019/02/12(火) 05:14:26.66:5UB4k9Qx
ttp://https://juken-mikata.net/how-to/physics/equation-of-motion.html

まず、mg=2×9.8=19.6ですね。

そして、運動方程式の公式F=maを使いましょう。

F

= 2×0.2

= 0.4[N]

ですね。

0.4 = T – 19.6 より、

T = 20.0[N]・・・(答)

となります。


これの解説の意味がさっぱりわからない
F=mg で、下方向に働いてる力は19.6Nでしょ?
F=maで、上方向に働いてる力は0.4Nでしょ?
でなんで0.4 = T – 19.6 って式になるの?
意味がさっぱりわからんのだけど
ご冗談でしょう?名無しさん [sage] 2019/02/12(火) 05:28:21.04ID:???
ma=T-mgだから
ご冗談でしょう?名無しさん [sage] 2019/02/12(火) 07:58:31.15ID:???

どれもこれもFと書いて自分でわけがわからなくなっているのでは?

その物体に働く力は、張力T(上向き)と重力mg(下向き)なので合わせると上向きにT-mg=T-19.6……※1
一方、加速度から物体に働く力を考えると上向きにmaの力が働いていることになり問題の条件から0.4……※2
※1と※2が等しいのでその式になる

mgは物体に働く重力であってその解説におけるFではないよ
運動方程式におけるFは物体に働く全ての力の合力であり、その問題では重力と張力の合力
ご冗談でしょう?名無しさん [sage] 2019/02/12(火) 09:08:32.46ID:???
めこすじマンもういないんだな
ご冗談でしょう?名無しさん [sage] 2019/02/12(火) 09:11:00.93ID:???
パンパース王の御前であるぞ皆慎め!
ご冗談でしょう?名無しさん [] 2019/02/12(火) 14:19:05.09:X/85373f
曖昧な質問で申し訳ないんですが

直径1kmの小惑星を亜光速で地球に落としたとすると、目標から4000km離れた地点でさえ致命的な熱波とかなり危険な地震波、
すさまじい粉塵と致命的に近い威力の衝撃波が襲い掛かり、衝撃波だけでも地球上のほぼすべての木をなぎ倒すレベルである。

という空想科学的な話があるんですが、本当なんですか?
この場合の亜光速ってどの程度以上の速度なんですか?
ご冗談でしょう?名無しさん [] 2019/02/12(火) 14:22:39.95:+KMxPcA5
びょーそくさんじゅうまんめーとるくらい。
ご冗談でしょう?名無しさん [sage] 2019/02/12(火) 16:03:45.27ID:???
むしろその程度の威力で済むの???
ご冗談でしょう?名無しさん [sage] 2019/02/12(火) 16:04:13.04ID:???
そんなデカイ小惑星が亜光速まで加速されるはずがない。
超新星爆発ですら光速の10%程度
ご冗談でしょう?名無しさん [sage] 2019/02/12(火) 16:15:41.89ID:???
あ、本当なんですか?っていうのは、
直径1kmの小惑星を亜光速で地球に落とすとここまで威力が出るんですか?
ってことです


小惑星を兵器化して亜光速で地球上に落下させるという話です
ご冗談でしょう?名無しさん [sage] 2019/02/12(火) 16:36:49.10ID:???
あえて科学的に考えるなら大質量を亜光速まで加速できるだけのテクノロジーがあるのに
それをぶつけるだけの兵器しか作れないってのはおかしいな
ご冗談でしょう?名無しさん [sage] 2019/02/12(火) 16:47:23.55ID:???
運動エネルギー兵器の評価は高まってます
ご冗談でしょう?名無しさん [sage] 2019/02/12(火) 17:14:34.18ID:???

びょーそくさんじゅうまんめーとる=300000m/s=300km/s。亜光速というにはちと遠いかな。
直径1kmの小惑星がこのスピードだと、恐竜を絶滅させた小惑星(直径10-15km,20km/sくらい)より
運動エネルギーとしては劣るくらいだから、「衝撃波だけでも地球上のほぼすべての木をなぎ倒すレベル」
には至らない
ご冗談でしょう?名無しさん [sage] 2019/02/12(火) 17:16:37.71ID:???
10円玉ぐらいの質量を光速の2/3ぐらいまで加速したら広島原爆の熱量に相当する運動エネルギーになるけど
運動エネルギーが兵器的にどんな効率なのかわからない
ご冗談でしょう?名無しさん [sage] 2019/02/12(火) 17:29:42.71ID:???

そんな小さい威力で済むのかねぇ

≫諸説あるが、2010年にサイエンス誌に掲載された説では、衝突した小惑星(チクシュルーブ衝突体)の大きさは直径10-15km、
衝突速度は約20km/s、衝突時のエネルギーは広島型原子爆弾の約10億倍(ツァーリ・ボンバの300万倍)、
衝突地点付近で発生した地震の規模はマグニチュード11以上、生じた津波は高さ約300メートルと推定されている。

恐竜を絶滅させたクレーターでこれ
亜高速で直径1キロならば大きさ1/15(質量はこの3乗)、速度は1万倍だから
エネルギーは3万倍くらい

この衝突でできたクレーターが半径160kmらしいから、3万発落とすと地表全てを5周くらいはカバーできるはず

このエネルギーで木がなくなるとかそんなもんで済むのかね?
ご冗談でしょう?名無しさん [sage] 2019/02/12(火) 18:06:55.71ID:???
真に亜光速なら、光速に近づければ近づけるだけ、いくらでも運動エネルギーは大きくなるわな。
もっとも、地球を突き抜けるほどになっちゃったらそれ以上地球へ与えるエネルギーは大きくならない、
という上限はあるかもしれないけど
ご冗談でしょう?名無しさん [sage] 2019/02/12(火) 23:16:38.25ID:???
そもそもどうやって計算すればいいんです?
ご冗談でしょう?名無しさん [sage] 2019/02/13(水) 00:25:06.32ID:???
m'=m/√1-(v/c)^2 だっけ?
ご冗談でしょう?名無しさん [sage] 2019/02/13(水) 01:02:29.89ID:???
わからないんですね
ご冗談でしょう?名無しさん [sage] 2019/02/13(水) 02:23:57.43ID:???
地球が割れるか耐えるかというレベルのエネルギーはあるんじゃないかな
ご冗談でしょう?名無しさん [sage] 2019/02/13(水) 02:24:55.32ID:???
地球割るのってどの程度のエネルギー必要なの?
ご冗談でしょう?名無しさん [] 2019/02/13(水) 02:42:37.33:pMQfbAdJ
ttp://https://twitter.com/akkyi22

障害者キチガイネトウヨヒトモドキ増殖工作中
ttp://https://twitter.com/5chan_nel (5ch newer account)
ご冗談でしょう?名無しさん [sage] 2019/02/13(水) 11:18:27.97ID:???
村本大輔(ウーマンラッシュアワー)(@WRHMURAMOTO)

正方形は全ての角度に面がある。正論は「わたしの側の面」でしかなく、
人の数だけ正しい面が存在する。正しいを一つにしてる価値こそ間違いで正しいは存在しない。

--------------
これの

 正方形は全ての角度に面がある

の意味は何でしょうか?
ご冗談でしょう?名無しさん [] 2019/02/13(水) 12:06:10.09:MDh4Kruf
立方体の間違いじゃね?
無知なだけなので、暖かく見守ってあげませう。
ご冗談でしょう?名無しさん [sage] 2019/02/13(水) 12:21:43.59ID:???

本当は
立方体にはいろんな方向から違う面がみれる
くらいのことを言いたいんでしょう
ご冗談でしょう?名無しさん [sage] 2019/02/13(水) 15:11:21.53ID:???
でも立方体の面は全て等価だよね?
ご冗談でしょう?名無しさん [sage] 2019/02/13(水) 15:52:49.81ID:???
科学の文脈で語られた言葉でもないのに科学的に解釈するのはバカだよ
ご冗談でしょう?名無しさん [sage] 2019/02/13(水) 17:34:10.78ID:???
不確定性原理は量子力学の原理ですか?
ウィキペディアには定理とあるんですが
ご冗談でしょう?名無しさん [sage] 2019/02/13(水) 18:10:06.69ID:???
定理です
ご冗談でしょう?名無しさん [sage] 2019/02/13(水) 18:25:04.22ID:???
シュレディンガー方程式から導けるだけで定理だと思い込む馬鹿が多過ぎる。
ご冗談でしょう?名無しさん [sage] 2019/02/13(水) 18:40:37.46ID:???
わからないんですね(笑)

不確定性原理って、演算子の代数的な関係のみから出てくる式ですので、シュレーディンガー方程式や量子論ぶっちゃけ関係ないんですけど
ご冗談でしょう?名無しさん [sage] 2019/02/13(水) 19:29:36.03ID:???
プランク定数って、「演算子の代数的な関係のみから出てくる」の?
俺は交換関係は実験事実から「要請」される関係だと理解してるんだけど。
ご冗談でしょう?名無しさん [sage] 2019/02/13(水) 19:30:38.24ID:???
一般的な不確定性原理を知ってればそう言うお話は出てこないはずですね(笑)
ご冗談でしょう?名無しさん [sage] 2019/02/13(水) 19:32:21.38ID:???
フーリエ変換から導かれる「不確定性」は数学的原理だけど
量子力学の不確定性原理はそれとは違うはずだけど。
ご冗談でしょう?名無しさん [sage] 2019/02/13(水) 19:32:49.42ID:???
「はず」ってどういうことですか?
わからないんですか?
ご冗談でしょう?名無しさん [sage] 2019/02/13(水) 19:34:01.61ID:???
いや、わかってるのならあなたが説明してくださいよ。
量子力学は実験事実とは無関係に演繹される理論体系だと言いたいのでしょ?
ご冗談でしょう?名無しさん [sage] 2019/02/13(水) 19:36:28.81ID:???
そんなこと言ってないですよね
不確定性原理は、一般的には任意の演算子間で成り立っていて、それら代数的に求めることが可能だと言ってるんです
ご冗談でしょう?名無しさん [sage] 2019/02/13(水) 19:38:38.65ID:???
不確定性原理は任意の演算子間で成り立つというのはどうしてですか?
それを証明できますか?
ご冗談でしょう?名無しさん [sage] 2019/02/13(水) 19:39:29.40ID:???
ウィキペディアにでも載ってるんじゃないですか?
ご冗談でしょう?名無しさん [sage] 2019/02/13(水) 19:39:57.48ID:???
どこに載ってるんですか?
分からないんですか?
ご冗談でしょう?名無しさん [sage] 2019/02/13(水) 19:41:07.30ID:???
どこに載ってるかは正直どうでもよくて、
証明できるのなら自分で示せるはずですよね?
ご冗談でしょう?名無しさん [sage] 2019/02/13(水) 19:41:51.13ID:???
証明できないんですか?
分からないんですか?
ご冗談でしょう?名無しさん [sage] 2019/02/13(水) 19:42:51.95ID:???
まーた劣等感か
ご冗談でしょう?名無しさん [sage] 2019/02/13(水) 19:43:50.17ID:???
ttp://https://ja.wikipedia.org/wiki/%E4%B8%8D%E7%A2%BA%E5%AE%9A%E6%80%A7%E5%8E%9F%E7%90%86

ここのロバートソンの不等式のところですね
ご冗談でしょう?名無しさん [sage] 2019/02/13(水) 19:50:08.01ID:???
「量子力学において(純粋)量子状態は状態空間 {\displaystyle {\mathcal {H}}} \mathcal{H}という
複素内積ベクトル空間(ヒルベルト空間)における長さ1のベクトル(状態ベクトル)として記述され、
物理量(オブザーバブルと呼ぶ)は {\displaystyle {\mathcal {H}}} \mathcal{H}上の
自己共役作用素として定式化される。」

これは物理的な要請であって、数学的には何の根拠もない仮定ですよね?
任意の演算子という前提がすでにここで崩れてますよね?
ご冗談でしょう?名無しさん [sage] 2019/02/13(水) 19:51:17.31ID:???
めんどくさいですね
じゃー任意の自己共役作用素に対して、ならいいですか?
ご冗談でしょう?名無しさん [sage] 2019/02/13(水) 19:53:39.19ID:???
任意の自己共役作用素というのは、ヒルベルト空間でない空間に対しても定義できるものを指しているのですか?
ご冗談でしょう?名無しさん [] 2019/02/13(水) 19:53:47.25:Ao0+LKVh
ttp://http://mb2.jp/_grn/2250.html-11-14
はいかがでしょうか?
ご冗談でしょう?名無しさん [sage] 2019/02/13(水) 19:54:40.89ID:???

ヒルベルト空間の自己共役作用素に対して、ならいいですか?
ご冗談でしょう?名無しさん [] 2019/02/13(水) 19:55:06.89:Ao0+LKVh
は、エーレンフェストのパラドックスの解決方法についてです。
ご冗談でしょう?名無しさん [sage] 2019/02/13(水) 19:55:19.31ID:???

なぜヒルベルト空間の自己共役作用素に限定するのですか?
ご冗談でしょう?名無しさん [sage] 2019/02/13(水) 19:55:43.33ID:???

どれだけあなたが重箱の隅つつこうが、シュレーディンガー方程式から導けるものだと思ってた事実は変わりませんよ?
ご冗談でしょう?名無しさん [sage] 2019/02/13(水) 19:56:32.20ID:???
ヒルベルト空間上で定義されない自己共役作用素とは具体的に何ですか?
ご冗談でしょう?名無しさん [sage] 2019/02/13(水) 19:57:11.27ID:???
なぜヒルベルト空間の自己共役作用素に限定するのですか?
ご冗談でしょう?名無しさん [sage] 2019/02/13(水) 19:59:59.91ID:???

量子力学考えてるんだからあたり前ですよね
ご冗談でしょう?名無しさん [sage] 2019/02/13(水) 20:00:02.48ID:???

不確定性原理は、一般的には任意の演算子間で成り立っていて、それら代数的に求めることが可能だと言ってるんです

と書いておきながら、ヒルベルト空間という特殊な空間を勝手に選んでしまう理由を教えてください。
ご冗談でしょう?名無しさん [sage] 2019/02/13(水) 20:01:08.52ID:???
量子力学の不確定性原理は、量子力学が要請する数学的空間とは無関係である、ということを
で主張していたのではないですか?
ご冗談でしょう?名無しさん [sage] 2019/02/13(水) 20:01:41.62ID:???

あなたがシュレーディンガー方程式から導かれるものだということを否定したんですよ?
441 [sage] 2019/02/13(水) 20:02:29.93ID:???
その人と私は別人です
415 [sage] 2019/02/13(水) 20:03:50.43ID:???
あなたは誰ですか?
ご冗談でしょう?名無しさん [sage] 2019/02/13(水) 20:05:17.70ID:???
不確定性原理が任意の演算子間で成り立つ
→[Δx,Δx]≥hbar/2
ご冗談でしょう?名無しさん [sage] 2019/02/13(水) 20:09:00.96ID:???
は別人ですか?
ご冗談でしょう?名無しさん [sage] 2019/02/13(水) 20:10:16.70ID:???

その式なんすか?w
415 [sage] 2019/02/13(水) 20:11:08.34ID:???

はい、別人です
ご冗談でしょう?名無しさん [sage] 2019/02/13(水) 20:13:25.40ID:???

ああよかったw
系列とは少なくとも別人なんですね
415 [sage] 2019/02/13(水) 20:16:30.64ID:???
ΔxΔx≧1/2|[x,x]|=0
ですから確かに不確定性原理成り立ってますね
ご冗談でしょう?名無しさん [sage] 2019/02/13(水) 20:21:07.28ID:???
物理量一つだけ測定すれば誤差の最小値は0なのはあたりまえだろ
そんなもんは不確定性とはなんの関連もない。
415 [sage] 2019/02/13(水) 20:22:29.26ID:???
わからないんですね
ご冗談でしょう?名無しさん [sage] 2019/02/13(水) 20:25:02.98ID:???

これを不確定性原理と言ってるアホの頭の中に脳のかわりに何が詰まってるかは確かにわかりません
ご冗談でしょう?名無しさん [sage] 2019/02/13(水) 20:29:43.83ID:???

0も含まれちゃダメでしょww
415 [sage] 2019/02/13(水) 20:30:35.65ID:???
ΔAΔB≧1/2|[A,B]|

これが不確定性原理ですね

ΔxΔp≧1/2|[x,p]|=h/2

おなじみの式です
ご冗談でしょう?名無しさん [sage] 2019/02/13(水) 20:33:48.39ID:???
だからそのプランク定数はどこから導き出されたんだよw
415 [sage] 2019/02/13(水) 20:34:32.73ID:???
少なくともシュレーディンガー方程式ではないでしょうね
ご冗談でしょう?名無しさん [sage] 2019/02/13(水) 20:35:02.15ID:???
なら何から?それを教えてよ。
分からないんですか?
415 [sage] 2019/02/13(水) 20:36:32.30ID:???
ΔAΔB≧1/2|[A,B]|

これはシュレーディンガー方程式から導かれるものではありません
残念でしたね
ご冗談でしょう?名無しさん [sage] 2019/02/13(水) 20:38:18.89ID:???
で、プランク定数はどここらやってくるの?
415 [sage] 2019/02/13(水) 20:38:53.52ID:???
量子力学の仮定ですね
ご冗談でしょう?名無しさん [sage] 2019/02/13(水) 20:39:10.41ID:???
もしかして日本語読めない?
ご冗談でしょう?名無しさん [sage] 2019/02/13(水) 20:40:57.95ID:???
劣等感婆ってマジでやってんの? それとも釣りなの?
ご冗談でしょう?名無しさん [sage] 2019/02/13(水) 20:43:04.17ID:???
風呂場で釣り竿を持って釣りをしてる精神病患者に、
意思が「どうですか釣れますか?」って訪ねたら
「風呂場て釣れるわけないだろ
ご冗談でしょう?名無しさん [sage] 2019/02/13(水) 20:52:43.42ID:???
劣等感婆さんが何故物理やってるか気になります
学生じゃないんですよね?
ご冗談でしょう?名無しさん [sage] 2019/02/13(水) 20:53:01.00ID:???
その量子力学の仮定からシュレディンガー方程式やハイゼンベルグ方程式となっているじゃないんですかね?
415 [sage] 2019/02/13(水) 21:02:22.30ID:???
でもそれらの方程式から導出されるわけではないですよ?
ご冗談でしょう?名無しさん [sage] 2019/02/13(水) 21:10:49.37ID:???

それは交換関係であって不確定性原理とは別物
交換関係と不確定性原理の区別くらいつけよう!
ご冗談でしょう?名無しさん [sage] 2019/02/13(水) 21:13:43.31ID:???

だから量子力学の仮定=シュレディンガー方程式なんだって
415 [sage] 2019/02/13(水) 21:14:50.57ID:???
414 名前:ご冗談でしょう?名無しさん [sage] :2019/02/13(水) 18:25:04.22 ID:???
シュレディンガー方程式から導けるだけで定理だと思い込む馬鹿が多過ぎる。


例えそうだとしても、どう考えても、こういう解釈は間違えですよね
ご冗談でしょう?名無しさん [sage] 2019/02/13(水) 21:18:41.41ID:???
なぜですか?
415 [sage] 2019/02/13(水) 21:23:12.41ID:???
ttp://https://ja.wikipedia.org/wiki/%E4%B8%8D%E7%A2%BA%E5%AE%9A%E6%80%A7%E5%8E%9F%E7%90%86


シュレーディンガー方程式なんて出てきてませんよ?
ご冗談でしょう?名無しさん [sage] 2019/02/13(水) 21:27:17.96ID:???

だからシュレディンガー方程式に等価なものから導いてるってだけだろ
415 [sage] 2019/02/13(水) 21:29:50.70ID:???
何と何が等価だと言ってるんですか?
ご冗談でしょう?名無しさん [sage] 2019/02/13(水) 21:36:34.20ID:???
もう一度聞きますが不確定性原理は原理ですか?定理ですか?
415 [sage] 2019/02/13(水) 21:36:59.44ID:???
定理ですよ
証明可能です
ご冗談でしょう?名無しさん [sage] 2019/02/13(水) 21:49:53.50ID:???
では証明して下さい
415 [sage] 2019/02/13(水) 21:51:36.03ID:???
ttp://https://ja.wikipedia.org/wiki/%E4%B8%8D%E7%A2%BA%E5%AE%9A%E6%80%A7%E5%8E%9F%E7%90%86

何度も貼りましたね
ご冗談でしょう?名無しさん [sage] 2019/02/13(水) 21:59:12.45ID:???
そのページに、任意の演算子に対して成り立つって書いてありますか?
ご冗談でしょう?名無しさん [sage] 2019/02/13(水) 22:01:28.70ID:???

具体的に説明してください
415 [sage] 2019/02/13(水) 22:01:42.33ID:???
不確定性原理の証明じゃなかったんですか?
ご冗談でしょう?名無しさん [sage] 2019/02/13(水) 22:08:08.90ID:???
演算子の代数的な関係のみから出てくる式と不確定性原理は同じじゃねえだろ
いくら演算子の代数的な関係のみから出てくる式の証明をしても不確定性原理の証明にはならねえよ
ご冗談でしょう?名無しさん [sage] 2019/02/13(水) 22:15:07.41ID:???
不確定性原理と演算子のブラケットの関係もわからず上から目線のやつ学部何回生君かな?
ご冗談でしょう?名無しさん [sage] 2019/02/13(水) 22:15:37.18ID:???
一回生だが何か?
ご冗談でしょう?名無しさん [sage] 2019/02/13(水) 22:16:12.77ID:???
不確定性原理は原理ですか?
ご冗談でしょう?名無しさん [sage] 2019/02/13(水) 22:17:55.62ID:???
量子力学は不確定性原理を満たすような定式化をしているに過ぎない
原理は原理だ
ご冗談でしょう?名無しさん [sage] 2019/02/13(水) 22:18:23.42ID:???

関係とはなんですか?
415 [sage] 2019/02/13(水) 22:27:15.03ID:???
いい加減認めた方が楽だと思いますけどねぇ
素直にわかりませんでした、と
415 [sage] 2019/02/13(水) 22:30:11.71ID:???
>不確定性原理と演算子のブラケットの関係もわからず上から目線のやつ学部何回生君かな?

演算子のブラケットの関係ってなんなんでしょうね
交換関係のことなんですかね
ご冗談でしょう?名無しさん [sage] 2019/02/13(水) 22:37:50.27ID:???

何がわからないんですか?
ご冗談でしょう?名無しさん [sage] 2019/02/13(水) 22:37:54.78ID:???
劣等感さんが何故物理をやってるか気になります
学生じゃないんですよね?
ご冗談でしょう?名無しさん [sage] 2019/02/13(水) 23:29:59.18ID:???

同意
物理的にはプランク定数h[J・s]が h>0の普遍定数と認めれば、不確定性原理になる。
ご冗談でしょう?名無しさん [sage] 2019/02/13(水) 23:40:57.23ID:???
劣等感婆は以前誘導起電力の問題でトンチンカン解答して
質問者からバカにされて煽られ、ガチギレして「死ね」を連投してたの見て
マジの精神障害者やんけと思ったわ
ご冗談でしょう?名無しさん [sage] 2019/02/13(水) 23:48:33.71ID:???
演算子の関係から不確定性原理が導かれてしまうなら、演算子を使わない定式化をするだけで不確定性原理を打破できるってことか
そんなことは不可能だからこその原理というわけね
ご冗談でしょう?名無しさん [sage] 2019/02/14(木) 00:00:40.73ID:???
というか数学畑的にはなんでもかんでも量子化したくて堪らないぐらいだ
ご冗談でしょう?名無しさん [sage] 2019/02/14(木) 00:14:13.84ID:???

どういう意味?
415 [sage] 2019/02/14(木) 00:15:27.45ID:???
不確定性”原理”てあるから証明不可能だとか言ってる人って、証明できませんよね多分

どうしてh/2より大きくなければならないのか、説明できるんでしょうかね
h/2はどこから来たのか
ご冗談でしょう?名無しさん [sage] 2019/02/14(木) 00:36:16.20ID:???
だいぶ前に
クーロンの法則の誘電率=波動方程式の誘電率
は本当かって話あったけど結論はどうなったの?
ご冗談でしょう?名無しさん [sage] 2019/02/14(木) 00:44:52.35ID:???
数学的に位置と波数の不確定性が1/2より大きいことが導ける
波数と運動量の関係がディラック定数倍だから位置と運動量の不確定性もh/2より大きい
415 [sage] 2019/02/14(木) 00:57:41.53ID:???
証明はどうやるんですか?
ご冗談でしょう?名無しさん [sage] 2019/02/14(木) 01:07:01.37ID:???

馬鹿が騒いでただけ。
ご冗談でしょう?名無しさん [sage] 2019/02/14(木) 01:19:01.09ID:???

教科書読めば?
前者はシュワルツの不等式から出るし後者は定義だ
415 [sage] 2019/02/14(木) 01:20:30.18ID:???
わからないんですか?
ご冗談でしょう?名無しさん [sage] 2019/02/14(木) 01:30:18.70ID:???
どこの証明が知りたいの?
415 [sage] 2019/02/14(木) 01:42:19.56ID:???
前者ですね
ご冗談でしょう?名無しさん [sage] 2019/02/14(木) 01:57:41.25ID:???
手元にあったのがだいぶ前に作った短時間フーリエ変換の不確定性原理の雑な証明だったけど
t→x, ω→kと読み替えれば位置と波数の不確定性原理になる
ttp://imgur.com/v5aNFrW.png
ご冗談でしょう?名無しさん [sage] 2019/02/14(木) 02:06:47.53ID:???
不確定性原理が原理だと思ってる人がこんなところにもいたとはな…
415 [sage] 2019/02/14(木) 02:17:20.60ID:???

標準偏差の定義は何故それなんですか?
ご冗談でしょう?名無しさん [sage] 2019/02/14(木) 02:28:02.52ID:???

gを分布関数と見たら期待値の二乗の積分値の平方根になってる
415 [sage] 2019/02/14(木) 02:37:25.47ID:???
V(x)=E(x^2)-E(x)^2ですよね?
ご冗談でしょう?名無しさん [sage] 2019/02/14(木) 02:54:36.79ID:???

今回計算したのはその式の右辺第一項
第二項は適当な並進変換で0にできる
ご冗談でしょう?名無しさん [殺す] 2019/02/14(木) 03:02:31.58ID:???
ありがとうございます
ご冗談でしょう?名無しさん [sage] 2019/02/14(木) 03:20:55.66ID:???
ぷっちーポポ ぷちプラーイ
ご冗談でしょう?名無しさん [sage] 2019/02/14(木) 04:38:10.82ID:???

本当に雑でイライラする
人名はカタカナかアルファベットか統一しろ
式2,4にピリオド付けろ
式3にコンマ付けろ
dtなどの後ろ空けすぎ¥,使え
コロン使いすぎ
ご冗談でしょう?名無しさん [sage] 2019/02/14(木) 07:47:27.68ID:???

原理を証明とは??
ご冗談でしょう?名無しさん [sage] 2019/02/14(木) 07:50:27.52ID:???

すごい早口で言ってそう
ご冗談でしょう?名無しさん [sage] 2019/02/14(木) 07:59:06.68ID:???


とかに証明が載ってますね
ご冗談でしょう?名無しさん [sage] 2019/02/14(木) 08:01:29.83ID:???
定理なのに反例があるみたいですね
読んでないですが
ご冗談でしょう?名無しさん [sage] 2019/02/14(木) 08:10:22.41ID:???
関係ないですよね
どんな原理にも定理にも例外はつきものです
ご冗談でしょう?名無しさん [sage] 2019/02/14(木) 08:22:33.22ID:???
例外と反例は違いますよね
ご冗談でしょう?名無しさん [sage] 2019/02/14(木) 08:27:49.01ID:???
どう違うんですか?
ご冗談でしょう?名無しさん [sage] 2019/02/14(木) 08:52:14.97ID:???
え、そんなこともわからないんですか?
ご冗談でしょう?名無しさん [sage] 2019/02/14(木) 12:38:20.41ID:???
原理と基本定理は逆でも定義できるだけ
特殊相対論  光速不変の原理(定理)<-> ローレンツ変換(原理)
量子力学   不確定性原理(定理)<-> シュレディンガー方程式(原理)
ご冗談でしょう?名無しさん [sage] 2019/02/14(木) 12:41:22.43ID:???
不確定性原理からシュレーディンガー方程式導いてみてください
ご冗談でしょう?名無しさん [sage] 2019/02/14(木) 12:47:03.62ID:???
逆でもいいですよ

シュレーディンガー方程式から不確定性原理でも
ご冗談でしょう?名無しさん [sage] 2019/02/14(木) 12:52:43.70ID:???
ドブローイ波(h)に不確定性関係が成り立つように構成するとシュレーディンガー方程式がでる
自分でググレ
ご冗談でしょう?名無しさん [sage] 2019/02/14(木) 12:56:36.38ID:???
hがゼロならシュレーディンガー方程式が成り立たない
ご冗談でしょう?名無しさん [sage] 2019/02/14(木) 12:57:44.92ID:???

わからないんですね
ご冗談でしょう?名無しさん [sage] 2019/02/14(木) 13:01:14.43ID:???

レスで厳密にできるわけないだろ知りたければググレ、キチガイ婆
ご冗談でしょう?名無しさん [sage] 2019/02/14(木) 13:02:25.89ID:???
具体的に示せないなら、わからないということですね
ご冗談でしょう?名無しさん [sage] 2019/02/14(木) 13:03:59.17ID:???
婆の相手すんなよw
ご冗談でしょう?名無しさん [sage] 2019/02/14(木) 13:04:40.02ID:???
量子力学の詳しい専門書に書いてあるだろ
ご冗談でしょう?名無しさん [sage] 2019/02/14(木) 13:07:06.22ID:???
見たことないですね

不確定性原理関連なら、ハイゼンベルク方程式からシュレーディンガー方程式導出するやり方ならあると思いますけど
ご冗談でしょう?名無しさん [sage] 2019/02/14(木) 13:08:35.78ID:???
お金がなくて専門書は買えないって言ってましたね
ご冗談でしょう?名無しさん [sage] 2019/02/14(木) 13:09:38.84ID:???
じゃ例えばどんな専門書になら書いてあるんですか?
図書館で見てきますよ
ご冗談でしょう?名無しさん [sage] 2019/02/14(木) 13:10:34.16ID:???
平日の昼間から図書館に通えるんですか?
ご冗談でしょう?名無しさん [sage] 2019/02/14(木) 13:10:38.20ID:???
桜井
ご冗談でしょう?名無しさん [sage] 2019/02/14(木) 13:12:48.19ID:???
図書館にはないみたいですね
その本にはどのような証明が乗ってるんですか?
ご冗談でしょう?名無しさん [sage] 2019/02/14(木) 13:14:57.10ID:???
ハイゼンベルク方程式とシュレディンガー方程式の比較から運動量演算子を定義してその交換関係から不確定性原理を導くのはできるけど
別にそんな回りくどいことしなくても運動量演算子は定義できるよね
ご冗談でしょう?名無しさん [sage] 2019/02/14(木) 13:16:02.76ID:???
私は、不確定性原理からシュレーディンガー方程式を導く方法を教えてくださいと言ってるんですけど
ご冗談でしょう?名無しさん [sage] 2019/02/14(木) 13:20:44.61ID:???
簡単には
ドブローイ波の不確定性関係から波数と運動量、エネルギーと振動数の式が確定して
シュレーディンガー方程式になる。
ご冗談でしょう?名無しさん [sage] 2019/02/14(木) 13:21:07.83ID:???
複雑にはどうなるんですか?
ご冗談でしょう?名無しさん [sage] 2019/02/14(木) 13:23:06.49ID:???
アインシュタインの関係式も不確定性原理から導かれる者だったなんて初耳ですね
ご冗談でしょう?名無しさん [sage] 2019/02/14(木) 13:23:30.13ID:???
不確定性原理だけからシュレーディンガー方程式が出てくるわけがない
光速不変の原理だけからローレンツ変換が出て来ないのと同じ。
ご冗談でしょう?名無しさん [sage] 2019/02/14(木) 13:24:09.99ID:???
まぁ、もうちょっと待ちましょうよ
ご冗談でしょう?名無しさん [sage] 2019/02/14(木) 13:25:29.98ID:???

不確定性原理はエネルギーと時間の関係にもなってるのも知らんのか
ご冗談でしょう?名無しさん [sage] 2019/02/14(木) 13:30:28.70ID:???
不等式からどうやって等式が出てくるんでしょうね
ご冗談でしょう?名無しさん [sage] 2019/02/14(木) 13:31:50.73ID:???
不確定性原理から波動関数やその時間微分というものが出てくるの?
ご冗談でしょう?名無しさん [sage] 2019/02/14(木) 13:34:06.20ID:???

不確定性原理を原理に立てた場合は波動の力学が必要になるという意味だ
ご冗談でしょう?名無しさん [sage] 2019/02/14(木) 13:35:00.93ID:???
ですから、どうやってその波動の力学を構築するのか教えてください?
ご冗談でしょう?名無しさん [sage] 2019/02/14(木) 13:38:29.97ID:???
シュレディンガー方程式と不確定性原理が等価と示してるpdfくらいあるだろ?
ご冗談でしょう?名無しさん [sage] 2019/02/14(木) 13:40:45.84ID:???
慣性の法則だけから運動方程式が出ないが、原理に替わりない

自分で量子力学の資料を調べればいいだろ、構成方法は幾つもある
ご冗談でしょう?名無しさん [sage] 2019/02/14(木) 13:41:34.89ID:???

どこにあるんですか?
あとシュレーディンガー方程式ばかり持ち出してハイゼンベルク方程式を出さないのは何故ですか?
わからないんですか?
ご冗談でしょう?名無しさん [sage] 2019/02/14(木) 13:42:44.82ID:???
ちょっとした書き込みでできる訳ないだろ、少しは考えろキチガイ
ご冗談でしょう?名無しさん [sage] 2019/02/14(木) 13:46:22.35ID:???
pdfあるんですよね?
url貼るだけですよ?
ご冗談でしょう?名無しさん [sage] 2019/02/14(木) 13:55:03.81ID:???
シュレディンガー方程式さんは交換関係と不確定性原理を区別できてない
ご冗談でしょう?名無しさん [sage] 2019/02/14(木) 13:59:44.91ID:???
数学でも公理と定理が等価的なら逆の論理構成ができる
それだけのことに粘着するのはキチガイだけだ。
ご冗談でしょう?名無しさん [sage] 2019/02/14(木) 14:02:31.59ID:???
実際にやれと言ってもやらないということは、やれないということですよね

pdfあるから読めと言いながら、urlすら貼らない
なんなんですかね
ご冗談でしょう?名無しさん [sage] 2019/02/14(木) 14:08:31.97ID:???
ttp://https://oshiete.goo.ne.jp/qa/6282007.html

言葉の混乱による誤解ですね。専門家の物理学者の間では、不確定性関係と言う言葉はありますが、不確定性原理という言葉は便宜的に使われることがあっても、本質的な意味では使いません。

シュレディンガー方程式は量子力学の力学原理を表す方程式ですので、これを物理学の基本の方程式とか原理と呼びます。実はそれと本質的に等価の方程式があり、それをハイゼンベルグ方程式と言います。
厳密には、シュレディンガー方程式は力学の状態を記述する方程式で、ハイゼンベルグ方程式は状態ではなく物理量を表す方程式ですので、その間には微妙に違いがあるのですが、それは細かい話しであり、共に物理学の基本原理を表した方程式です。

一方、不確定性関係式はこの基本法則であるハイゼンベルグ方程式から演繹的に導き出される一つの特殊な定理あるいは関係式のことすので、これを基本原理と呼ぶわかにはいかないのです。
別な言い方をすると、ハイゼンベルグ方程式やシュレディンガー
方程式から不確定性関係式を導き出すことは出来ますが、その反対に、不確定性関係式からはハイゼンベルグ方程式やシュレディンガー方程式を導き出すことは出来ません。だから、不確定性関係は原理ではないのです。
ご冗談でしょう?名無しさん [sage] 2019/02/14(木) 14:09:03.89ID:???
こんな記事ありましたよ?
不確定性原理からシュレーディンガー方程式を導き出すpdfは見つかりませんでしたけど
ご冗談でしょう?名無しさん [sage] 2019/02/14(木) 14:13:14.65ID:???
物理歴史的な構成方法は、理論が完成すれば使わなくなるだけ
ご冗談でしょう?名無しさん [sage] 2019/02/14(木) 14:14:05.00ID:???
ならその構成法はやくおしえてください?
ご冗談でしょう?名無しさん [sage] 2019/02/14(木) 14:15:21.52ID:???
ちなみに、シュレーディンガーによるシュレーディンガー方程式の導出は、不確定性原理ではなく、解析力学のハミルトンヤコビ方程式から得られたものですけど
ご冗談でしょう?名無しさん [sage] 2019/02/14(木) 14:35:44.74ID:???
みたいな面倒なのは無視して叩けるところだけ叩く戦法は賢い
レスバ強い
ご冗談でしょう?名無しさん [sage] 2019/02/14(木) 14:37:41.91ID:???

お前には解らんだろが古典力学方程式からシュレーディンガー方程式に転換するには
必然的に不確定性関係が必要なんだよ。
ご冗談でしょう?名無しさん [sage] 2019/02/14(木) 14:38:29.25ID:???

必然性を示してくださいねー

不確定性原理からどうやってアインシュタインの関係式導くんですか?
ご冗談でしょう?名無しさん [sage] 2019/02/14(木) 14:40:24.61ID:???

だからそれは不確定性関係じゃなくて交換関係
ご冗談でしょう?名無しさん [sage] 2019/02/14(木) 14:43:20.87ID:???
シュレーディンガー方程式から不確定性関係は位置と運動量のフーリエ変換で出すんだろうが
ハイゼンベルグ方程式から出すのはどうするんだっけ?
行列が非可換だと同時対角化が出来ないことが理由だったはずだが分かんねー
ご冗談でしょう?名無しさん [sage] 2019/02/14(木) 14:48:35.69ID:???
それシュレーディンガー方程式から導いてるわけじゃないですよね
波動関数の性質だけを使ってますよね

何かの方程式から不確定性が出ると言ってる人はそもそもおかしいんです
不確定性と時間発展の方程式は無関係ですから
ご冗談でしょう?名無しさん [sage] 2019/02/14(木) 14:49:45.90ID:???

不確定性関係とは波動の関係の意味だ
ご冗談でしょう?名無しさん [sage] 2019/02/14(木) 14:50:22.47ID:???
波動の関係とはどのような意味ですか?
ご冗談でしょう?名無しさん [sage] 2019/02/14(木) 14:53:55.88ID:???
シュレディンガー方程式くんとかいう新たなキチガイを生み出してしまった
ご冗談でしょう?名無しさん [sage] 2019/02/14(木) 14:55:17.50ID:???


チョコットのスレレスでは詳細説明ができないことがわからんのか、キチガイ喚きは止めろ
ご冗談でしょう?名無しさん [sage] 2019/02/14(木) 14:56:03.85ID:???
あなたpdfに載ってると言いましたよね?
そのpdfのurl貼ればいいだけなのに何故それをしないんですか?
ご冗談でしょう?名無しさん [sage] 2019/02/14(木) 15:10:25.31ID:???

喚く前に運動量演算子の導出と交換関係を復習してこい
ご冗談でしょう?名無しさん [sage] 2019/02/14(木) 15:44:27.77ID:???
貼るまでもない
ご冗談でしょう?名無しさん [sage] 2019/02/14(木) 15:54:22.10ID:???
いい加減わかりませんでしたと認めたらどうですか?
ご冗談でしょう?名無しさん [sage] 2019/02/14(木) 18:42:29.36ID:???

お前は丸暗記バカだな、シュレディンガー方程式ありきの話じゃないんだよ、流れがわからんか

古典粒子の運動(方程式)をドブローイ波の波束に対応させた時点で不確定性関係になるということだ。
ご冗談でしょう?名無しさん [sage] 2019/02/14(木) 18:51:58.25ID:???
ムニちゃーんポポ
ご冗談でしょう?名無しさん [sage] 2019/02/14(木) 18:55:55.70ID:???

ファインマンも述べてる「量子力学など知らなくても波動を調べたことがある人なら誰でも知っていること」
ご冗談でしょう?名無しさん [sage] 2019/02/14(木) 18:56:33.61ID:???

しつこくレスする余裕があるんですから、はやく不確定性原理からシュレーディンガー方程式導いてください?
ご冗談でしょう?名無しさん [sage] 2019/02/14(木) 19:07:04.40ID:???
シュレディンガーとは
ご冗談でしょう?名無しさん [sage] 2019/02/14(木) 19:08:16.83ID:???
ここまで不確定性原理とシュレディンガー方程式が等価である証明が文献含め提示されていないのは凄いことだと思う
ご冗談でしょう?名無しさん [sage] 2019/02/14(木) 19:14:00.89ID:???

わからんのか後は本の手順どおりだろが、本の手順では不確定性について何も書いてないだけだ。
ご冗談でしょう?名無しさん [sage] 2019/02/14(木) 19:20:49.28ID:???
どの本ですか?
ご冗談でしょう?名無しさん [sage] 2019/02/14(木) 19:21:07.94ID:???
皆さんが見れるようにpdfだと尚良いですね
ご冗談でしょう?名無しさん [sage] 2019/02/14(木) 19:22:17.55ID:???
不確定性原理とシュレディンガー方程式が等価なら、電子もディラック方程式ではなく
シュレーディンガー方程式に従うことになるんだろうな。
不確定性原理とシュレディンガー方程式が等価である証明、楽しみだなー
ご冗談でしょう?名無しさん [sage] 2019/02/14(木) 19:25:27.29ID:???

古典物理から出してる本なら幾らでもあるどれでも同じ
ご冗談でしょう?名無しさん [sage] 2019/02/14(木) 19:27:06.71ID:???

なんだそれ、キチガイの仲間か?
ご冗談でしょう?名無しさん [sage] 2019/02/14(木) 19:50:58.92ID:???

ここまで理解できれば,不確定性原理はシュレディンガー方程式流の量子力学に限らず
他の形式や相対論的な形式でも成り立つことがわかるだろう。
ご冗談でしょう?名無しさん [sage] 2019/02/14(木) 19:52:46.64ID:???
不等式と等式が等価になる時点でおかしいと思わないのかな
ご冗談でしょう?名無しさん [sage] 2019/02/14(木) 19:54:37.75ID:???
スクイージングの限界ってどんなもんなの?
理論上は無限の精度出せそうに見えるけど
ご冗談でしょう?名無しさん [sage] 2019/02/14(木) 20:00:28.93ID:???

アホか、波束の関数が不明だから等式になるわけないだろ
ご冗談でしょう?名無しさん [sage] 2019/02/14(木) 20:14:38.09ID:???

シュレディンガー方程式と不確定性原理が等価って話じゃないの?
運動方程式をドブロイ波の波束に対応させるというのが意味不明なんだけど
運動量演算子が -ih d/dx って書けるってこと?
ご冗談でしょう?名無しさん [sage] 2019/02/14(木) 20:30:48.55ID:???
シュレディンガー方程式はihbar∂Ψ/∂t=HΨですよ
これは不確定性原理と等価です
ご冗談でしょう?名無しさん [sage] 2019/02/14(木) 21:38:04.33ID:???

あなたの持ってる本でいいですよ
引用してください
ご冗談でしょう?名無しさん [sage] 2019/02/14(木) 22:57:36.24ID:???
ムニちゃーん しんぷ
ご冗談でしょう?名無しさん [] 2019/02/14(木) 23:59:49.60:qSq8yGfR
よろしくお願いします。等加速度についての質問です。

>>ttp://https://www.youtube.com/watch?v=Z--iayAq2do&list=PLiU32qLvWeyEGFFq4xHxZsDxALtbq005p&index=7

の1分22秒で、一秒後の距離は4メートルと言われてますが
どうして4メートルがでるのかわかりません。

図形や公式は理解できますが、なぜでしょうか。
ご冗談でしょう?名無しさん [sage] 2019/02/15(金) 00:15:02.30ID:???
x=vt+at^2/2
v=3,a=2,t=1
ですね
ご冗談でしょう?名無しさん [sage] 2019/02/15(金) 01:34:54.76ID:???
シュレーディンガー方程式と不確定性原理の同等性の照明がいつまでたっても出てきませんね
ご冗談でしょう?名無しさん [sage] 2019/02/15(金) 05:51:01.57ID:???
同等性の照明とは何でしょうか
ご冗談でしょう?名無しさん [sage] 2019/02/15(金) 06:26:44.66ID:???
不確定性原理を使ってシュレディンガー方程式を始めに作ったのだから、シュレディンガー
方程式から不確定性原理が証明できるのがあたりまえだ。
ご冗談でしょう?名無しさん [sage] 2019/02/15(金) 07:01:52.18ID:???
ttp://http://eman-physics.net/quantum/schrodinger.html
有名なサイトのシュレディンガー方程式の導出だけど
君の考えではこれでいうとどこで不確定性原理を使ってることになるのかな?
ご冗談でしょう?名無しさん [sage] 2019/02/15(金) 07:06:18.58ID:???

ぷっ
ご冗談でしょう?名無しさん [sage] 2019/02/15(金) 07:54:59.83ID:???

その先に説明あるじゃん
ご冗談でしょう?名無しさん [sage] 2019/02/15(金) 08:15:41.43ID:???

方程式の操作で古典粒子の運動でなくなること位はわかるだろ、それで完成しましたのたぐい。
操作したとき位置と運動量の関係がどうなるか調べるんだ、波動光学と解析力学を使え。
ご冗談でしょう?名無しさん [sage] 2019/02/15(金) 08:35:10.91ID:???

もう古レスになってしまうけど、某学術系youtuberによると
地球という天体を破壊する(すべての破片を第二脱出速度にする)必要なエネルギーが
2x10^32Jとのこと。
ツァーリ・ボンバが2x10^17J
の計算に従えば、この小惑星の衝突時のエネルギーはツァーリ・ボンバの300万x3万倍で
約10^11倍だから、2x10^28J

なかなかどうしてまだ地球は木端微塵にはならないな。
でも地球は地球じゃなくなるよね
ご冗談でしょう?名無しさん [sage] 2019/02/15(金) 09:06:00.86ID:???

>操作したとき位置と運動量の関係がどうなるか調べるんだ、波動光学と解析力学を使え。

それで出てくるんだったら、原理として使ってないってことだぞ。
ご冗談でしょう?名無しさん [sage] 2019/02/15(金) 09:39:09.49ID:???

シュレディンガー方程式を作る段階で古典力学では矛盾する関係が仕込まれており
作られたシュレディンガー方程式は古典力学でないから矛盾しない「原理」といえる。
ご冗談でしょう?名無しさん [sage] 2019/02/15(金) 09:40:54.07ID:???
んで、不確定性原理からシュレーディンガー方程式を導出する方法はまだですか?
ご冗談でしょう?名無しさん [sage] 2019/02/15(金) 09:41:55.78ID:???
それはシュレーディンガー方程式と不確定性原理の等価性を示したことになっていない
ご冗談でしょう?名無しさん [sage] 2019/02/15(金) 09:46:10.33ID:???

病院逝け、馬鹿。
ご冗談でしょう?名無しさん [sage] 2019/02/15(金) 09:48:18.33ID:???

不確定性関係のない関係式からはシュレディンガー方程式が作れない。
「原理」と言うしかないだろ。
ご冗談でしょう?名無しさん [sage] 2019/02/15(金) 09:49:27.14ID:???
ですから、その作り方を教えてください?
ご冗談でしょう?名無しさん [sage] 2019/02/15(金) 09:49:28.90ID:???
哀れだな
ご冗談でしょう?名無しさん [sage] 2019/02/15(金) 09:49:30.40ID:???

脳無し
ご冗談でしょう?名無しさん [sage] 2019/02/15(金) 09:50:25.44ID:???

火星サイズの原始惑星が衝突したとされるジャイアントインパクトでさえ
(単純に大きさの比の3乗のエネルギーとして、チクシュルーブのざっと10億倍のエネルギー)
地球を木端微塵にするには至らず、破片の一部が月になった程度ですんでるからな
ご冗談でしょう?名無しさん [sage] 2019/02/15(金) 09:55:36.79ID:???
劣等感が合ってることもあるから困る
ご冗談でしょう?名無しさん [sage] 2019/02/15(金) 10:01:39.81ID:???

あと誰かが言ってましたけど、不確定性原理は量子論一般の話ですね
非相対論的なシュレーディンガー方程式だけでなく、相対論的なクラインゴルドン方程式やディラック方程式でも成り立つはずです

ということは、不確定性原理から、クラインゴルドン方程式やディラック方程式も出てくるのでしょうか?
もしそうなら、不確定性原理とシュレーディンガー方程式が等価であるとするならば、シュレーディンガー方程式もクラインゴルドン方程式もディラック方程式も全て等価だということですか?

(i∂0+1/2m ∇^2)ψ=0
(◻︎+m^2)ψ=0
(iγ^μ∂_μ-m)ψ=0

同じに見えないですけどねー
ご冗談でしょう?名無しさん [sage] 2019/02/15(金) 10:09:23.47ID:???

流れが読めんのか
古典力学方程式に波動の関係式をぶち込んで作ったのがシュレーディンガー方程式だ
誰でも知ってる波動の関係式を古典力学で調べると古典力学では矛盾する不確定性関係がある。
ご冗談でしょう?名無しさん [sage] 2019/02/15(金) 10:10:18.03ID:???
ですから、波動の関係式とはなんですか?
不確定性原理とは別ですか?
前も聞きましたよね
ご冗談でしょう?名無しさん [sage] 2019/02/15(金) 10:16:23.42ID:???

>波動の関係式とはなんですか?
馬鹿
ご冗談でしょう?名無しさん [sage] 2019/02/15(金) 10:17:37.14ID:???
光子が従うマクスウェル方程式∂^νF_μν=μoJ_μも不確定性原理と等価なはずだな。
証明楽しみだなー。早く出せよシュレーディンガー方程式くん?
ご冗談でしょう?名無しさん [sage] 2019/02/15(金) 10:19:50.19ID:???

馬鹿ではなく、なんなのですかと聞いてますね
ご冗談でしょう?名無しさん [sage] 2019/02/15(金) 10:23:12.36ID:???
シュレーディンガー方程式くんでは名前が長すぎる
ご冗談でしょう?名無しさん [sage] 2019/02/15(金) 10:28:43.78ID:???
そもそも古典物理の方程式から、量子力学の式を作るのは定理の導出とは全く違う
論理矛盾する新たな仮説をつかうことになる。
ご冗談でしょう?名無しさん [sage] 2019/02/15(金) 10:31:30.27ID:???
あやふやな言葉ばかりで、数式が出てきませんね
数式じゃなくても、ちゃんとした物理用語を用いて話せないんですか?
ご冗談でしょう?名無しさん [sage] 2019/02/15(金) 10:35:35.14ID:???
イーマンとかの詳しい作り方を見ればいいだけだろ
ご冗談でしょう?名無しさん [sage] 2019/02/15(金) 10:36:04.62ID:???

EMANはシュレーディンガー方程式の導出に不確定性原理使ってないですよ?
ご冗談でしょう?名無しさん [sage] 2019/02/15(金) 10:45:38.27ID:???
根拠あるならまるごと引用でいいから出せよ
ないなら作れ
ご冗談でしょう?名無しさん [sage] 2019/02/15(金) 10:50:44.52ID:???

どの本も、波動の式で置き換えてるだけだ。 置き換えて良いという根拠を詳しく説明してる本は少ない。
キミは理由を知ってるのか
ご冗談でしょう?名無しさん [sage] 2019/02/15(金) 10:52:00.85ID:???

波動の式とはなんですか?
ご冗談でしょう?名無しさん [sage] 2019/02/15(金) 10:52:41.15ID:???
あとEMANのやり方とは違った構成法あるの知らないんですかね
ご冗談でしょう?名無しさん [sage] 2019/02/15(金) 10:53:25.97ID:???

誰でも知ってる2つの式だろ、わからんのか
ご冗談でしょう?名無しさん [sage] 2019/02/15(金) 10:54:04.16ID:???
それを書けばいいのでは?
ご冗談でしょう?名無しさん [sage] 2019/02/15(金) 10:54:51.41ID:???

アインシュタインの関係式のことですか??
E=hν
p=h/λ
これを波動の式なんて言ってる人聞いたことないですね

んで、ハイゼンベルク方程式を経由してシュレーディンガー方程式を導出する流儀は知らないということでいいですか?
ご冗談でしょう?名無しさん [sage] 2019/02/15(金) 11:03:14.82ID:???

その式の何処に不確定性関係があるのか、自分で調べるんだな、ちょっとで説明するのは無理
bye-bye.
ご冗談でしょう?名無しさん [sage] 2019/02/15(金) 11:05:12.81ID:???

わからないんですね(笑)
逃げるんですね(笑)
ご冗談でしょう?名無しさん [sage] 2019/02/15(金) 11:10:27.62ID:???
書いてある本の名前を挙げるか、資料となるurlを示せばよいのでは?
ご冗談でしょう?名無しさん [sage] 2019/02/15(金) 11:37:39.66ID:???
E → i hbar ∂/∂t
p → -i hbar ∇
の置き換えのことかと思ったらのことかよ>「波動の式」

まぁ何でもいいけど
「波動の式」から(古典論の式へ適用することにより?)シュレーディンガー方程式が出て、
同じ「波動の式」から不確定性関係が出るから
シュレーディンガー方程式と不確定性関係は同値だ、なんて主張じゃないことを祈る。
もしそういう主張ならそう言ってくれ、シュレくん。臍の上に茶を準備して待ってるから
ご冗談でしょう?名無しさん [sage] 2019/02/15(金) 11:41:38.50ID:???
そういう意味だとしても、「波動の式」からは不確定性原理はでませんよ
不確定性原理は波動一般的な問題であり、非可換な演算子に対する条件でもあります
ご冗談でしょう?名無しさん [sage] 2019/02/15(金) 12:39:54.42ID:???

知障が逃げた!(^^)/~~
ご冗談でしょう?名無しさん [sage] 2019/02/15(金) 14:11:52.42ID:???
馬鹿いじりを見るのは飽きた
ご冗談でしょう?名無しさん [sage] 2019/02/15(金) 16:31:21.90ID:???
劣等感に負けるとか恥ずかしくないの
ご冗談でしょう?名無しさん [sage] 2019/02/15(金) 18:15:57.05ID:???
ムニちゃんダンシング
ご冗談でしょう?名無しさん [sage] 2019/02/15(金) 20:32:15.56ID:???
シュレ君、恥ずかしくて首吊って死んでじゃね?
ご冗談でしょう?名無しさん [sage] 2019/02/15(金) 20:46:15.22ID:???
たぶん途中で自分でも間違えたと薄々気づいてたんだと思うよ。だからいくら要求されても
具体的に説明もしなければ、「貼るまでもない」などとわけのわからない理由をつけて
pdfの提示さえしない。「波動の関係」などという、どうとでもとれそうな定義の曖昧な用語で
のらりくらりと逃げ回るだけ
ご冗談でしょう?名無しさん [sage] 2019/02/15(金) 20:51:35.82ID:???
半古典近似
ご冗談でしょう?名無しさん [sage] 2019/02/15(金) 21:03:36.41ID:???
古典と量子の間〈量子力学 3〉 (岩波講座 物理の世界)
首藤 啓
ttp://https://www.amazon.co.jp/dp/4000111116/



これ買おうかと思うんだけどどうよ?
ご冗談でしょう?名無しさん [sage] 2019/02/16(土) 03:19:25.92ID:???
何の役にも立たない自演厨
ご冗談でしょう?名無しさん [sage] 2019/02/16(土) 12:59:09.28ID:???
たくさんの粒子が、ある不動の板に、
単位時間あたりに一定の数
ぶつかり続けるとして

1粒子が板に与える力積をp、毎秒n個ぶつかるとすると

板に働く力がpnで与えられるのは何故ですか?
ご冗談でしょう?名無しさん [sage] 2019/02/16(土) 13:02:04.68ID:???
力積ってなんだかわかりますか?
ご冗談でしょう?名無しさん [sage] 2019/02/16(土) 13:03:33.01ID:???
分かります。
kg*m/sに/sを掛けると単位がNになるのは分かりますが、単位が揃ったことしかいえませんよね?

なぜそれで力が出るんでしょう?
ご冗談でしょう?名無しさん [sage] 2019/02/16(土) 13:05:26.51ID:???
力積とはなんですか?
説明してみてください
ご冗談でしょう?名無しさん [sage] 2019/02/16(土) 13:17:46.08ID:???
なんで逆質問されるんでしょう(^_^;)
もし分かってるのなら普通に回答してほしいんですが
ご冗談でしょう?名無しさん [sage] 2019/02/16(土) 13:19:36.98ID:???
力積は力×時間ですよね
力をF,時間をtとすると、t秒間に与えられる力積は
Ft=npt
F=npですね

わかりましたか?
ご冗談でしょう?名無しさん [sage] 2019/02/16(土) 13:22:56.40ID:???

f:一粒子による力
冲:一粒子が板に力を加えてる微小時間
N:冲間にぶつかる粒子数
p=f冲
N=n冲
F=fN=pn
ご冗談でしょう?名無しさん [sage] 2019/02/16(土) 14:23:46.44ID:???

の回答から力積を逆質問された理由がわかるはず
ご冗談でしょう?名無しさん [sage] 2019/02/16(土) 14:59:20.67ID:???
全く違うモデルから同じ結果が得られたらどうなるの?
ご冗談でしょう?名無しさん [sage] 2019/02/16(土) 15:01:10.65ID:???

他の実験をする
ご冗談でしょう?名無しさん [sage] 2019/02/16(土) 17:54:19.89ID:???
悪意ない人間に逆質問はやめとけ
喧嘩売る相手は選べ
ご冗談でしょう?名無しさん [sage] 2019/02/16(土) 18:18:01.23ID:???
なんで逆質問したら喧嘩なんですか?
理解を深めるためには問答は重要ですよ
ソクラテスですね
ご冗談でしょう?名無しさん [sage] 2019/02/16(土) 19:12:36.56ID:???
一般相対性理論の問題点について
ttp://http://sg2.phys.cst.nihon-u.ac.jp/~fffujita/EinsteinGR.pdf

ここで指摘されていることは本当なんですか?
ご冗談でしょう?名無しさん [sage] 2019/02/16(土) 19:34:22.70ID:???

たとえば、3ページ目に単位が合わない云々言ってますよね、その人

ttp://http://eman-physics.net/relativity/newton.html

ここの記事にもある通り、普通の単位系を用いれば/c^2という係数が入るためにちゃんとポテンシャルに相当する部分の次元も無次元になるようになってるんです

おそらく、この人が読んだ本ではc=G=1とした単位系を用いていたのでしょうね

ですから、普通のSI単位系に慣れた我々が一見すると単位が揃ってないように見えてしまっているのです

この人は単位というものすらわかってないということですね
読む価値はないと思います
ご冗談でしょう?名無しさん [sage] 2019/02/16(土) 19:48:33.50ID:???

喧嘩売るために掲示板やってる奴に言っても無駄だと思う
ご冗談でしょう?名無しさん [sage] 2019/02/16(土) 20:45:37.32ID:???

この人は無次元量であることは理解しているみたいです
ご冗談でしょう?名無しさん [sage] 2019/02/16(土) 21:11:35.25ID:???
分からないところが分かる
ってこれ実は凄いことで、分かる1歩手前の状態なんだよね。

今回の逆質問は力積とはなんなのかというのが分かれば解決出来るんだよということを促したかったんじゃないの
ご冗談でしょう?名無しさん [sage] 2019/02/16(土) 23:28:11.96ID:???
ムニちゃーんポポ
ご冗談でしょう?名無しさん [] 2019/02/17(日) 00:22:43.27:IOfBFw8a
春から物理学科なのですが、読んでおくべき本とかありますか?
高校と大学では全く違うそうなので、不安です
ご冗談でしょう?名無しさん [sage] 2019/02/17(日) 00:24:42.76ID:???
ランダウ力学くらい中学のうちに読んでおかないと入学後マウント取られて死ぬぞ
ご冗談でしょう?名無しさん [sage] 2019/02/17(日) 00:25:06.36ID:???

数学の本読んどけば
微分方程式とかベクトル解析とか
ご冗談でしょう?名無しさん [sage] 2019/02/17(日) 00:47:23.75ID:???

パウリの相対性理論。
大学入学3年後にこんな本書いたのか、ってやる気が起きるか、絶望する。
ご冗談でしょう?名無しさん [sage] 2019/02/17(日) 08:35:06.62ID:???
背伸びした本ばっか挙がってるなw
なんでだろうねw

微分方程式もベクトル解析も大学の微積やってからでいいし、パウリも2-4年で相対論やるだろうからそのときでいい

入学まで1ヶ月半しかないから、月並みだけど山本の物理入門や、杉山の理論物理学の道標で微積に慣れておくのがいいと思う
ご冗談でしょう?名無しさん [sage] 2019/02/17(日) 09:07:27.29ID:???
放送大学の無料放送(OCW)の動画で「物理の世界」が全部見れるから、教科書買って全部見てみたら?
放送で説明している分はともかく教科書での説明は結構難易度高いけど、文字通り大学レベルの基礎の
物理を一通り説明してくれる。
ご冗談でしょう?名無しさん [] 2019/02/17(日) 12:14:01.88:Psch+x0q
みなさんありがとうございます
とりあえず新物理入門でも読んでみようかと思います
ご冗談でしょう?名無しさん [sage] 2019/02/17(日) 12:44:35.50ID:???

繰り込み不能以外の問題点など相間にすぎん
ご冗談でしょう?名無しさん [sage] 2019/02/17(日) 12:48:56.82ID:???
こんな教授の講義取って単位とらなきゃならない日大生が哀れすぎる。
ご冗談でしょう?名無しさん [sage] 2019/02/17(日) 12:53:36.64ID:???

日大なんかいって物理を学ぼうとするのがおこがましいだろ
ご冗談でしょう?名無しさん [sage] 2019/02/17(日) 14:16:48.45ID:???
(;∀; )
ご冗談でしょう?名無しさん [sage] 2019/02/17(日) 14:19:45.38ID:???

ttp://https://togetter.com/li/1277376

更にレベルを落とすとこうなるから、大学って現状かなり終わってるよ
ご冗談でしょう?名無しさん [sage] 2019/02/17(日) 15:23:09.55ID:???
おこがましいなんて言葉が出るのが臭い
ご冗談でしょう?名無しさん [sage] 2019/02/17(日) 15:25:56.63ID:???

近日点に関する話などはどうですか
ご冗談でしょう?名無しさん [sage] 2019/02/17(日) 15:42:34.35ID:???

日大生乙
ご冗談でしょう?名無しさん [sage] 2019/02/17(日) 17:03:24.92ID:???
ムニちゃんポポだのんのおじさんだい
ご冗談でしょう?名無しさん [] 2019/02/18(月) 00:28:55.56:lkgiGvzl

グラフをイメージして数学的に理解するなら難しくないよ。np各成分を縦軸、時間を横軸、力は運動量の時間微分だから
面積で表わされる運動量を微分すれば力になる。図を書けば速い。
ご冗談でしょう?名無しさん [sage] 2019/02/18(月) 00:34:56.69ID:???
こんな簡単な問題でそんな面倒なこと考える人はいませんよ
ご冗談でしょう?名無しさん [sage] 2019/02/18(月) 00:42:35.00ID:???

それが面倒なことだと思うんだったら物理向いてない
ご冗談でしょう?名無しさん [sage] 2019/02/18(月) 01:17:18.85ID:???
整数の足し算議論するのに、圏論の議論から始める必要はないですよね?
そういうことです
ご冗談でしょう?名無しさん [sage] 2019/02/18(月) 03:42:43.52ID:???
これが圏論に見えるのか足し算に見えるのかは人によるだろう
圏論に見えるようじゃあ不向きだ
ご冗談でしょう?名無しさん [sage] 2019/02/18(月) 06:59:14.29ID:???
あなたは不向きだということですね
足し算は足し算でしかありませんから
ご冗談でしょう?名無しさん [sage] 2019/02/18(月) 07:49:24.67ID:???

定義から明らかなのに、何で態々そんなことすんの?
ご冗談でしょう?名無しさん [sage] 2019/02/18(月) 08:43:42.86ID:???
レベルが低いからですね
ご冗談でしょう?名無しさん [sage] 2019/02/18(月) 10:11:50.81ID:???

自然数から整数を定義するために負の数を定義するのにいきなりグロタンディーク構成使うのを掴みに加群一般論と圏論を勉強するのはそんなに悪い方法じゃないとおもうんだよね。

俺も小学生の頃に読んだマシン語の入門書の最後の方に載ってた補数表現で見かけ上無限大の向こう側彼岸にマイナスの数を定義するのとディラックの海の海面位の上昇下降としてのアノマリーの意味付けとの関係性に勘付く遠回りのようなモノを否定はしないが。
ご冗談でしょう?名無しさん [sage] 2019/02/18(月) 10:15:24.52ID:???
フォック空間フォック代数も二進数という名のガロア体も超対称的に海水面位のレベルの問題に帰着させるレベルにまで持っていかないと。
ご冗談でしょう?名無しさん [sage] 2019/02/18(月) 13:20:02.71ID:???
だって今回の質問は力積の本質となる所だからね。
なんで力積というものを習うかって言うと今までの力のつりあいでは扱えないような撃力が扱えるようになるということ。さらに今回は気体分子運動論の基礎にも繋がる超重要部分であり、
そこを詳しくやるのがまわりくどいと感じているようなら後々物理分からなくなるのは想像に難くないだろう?
ご冗談でしょう?名無しさん [sage] 2019/02/18(月) 13:23:24.23ID:???
2×3計算するときにペアノシステムの公理系から考えようとするのは、バカって言うんですよ?
ご冗談でしょう?名無しさん [sage] 2019/02/18(月) 13:28:19.84ID:???
物理の話しようぜ
ご冗談でしょう?名無しさん [sage] 2019/02/18(月) 13:29:35.00ID:???

vがpの関数として表せないとき、ハミルトニアンはどうやって構成するんですか?

そんな問題よりこちらの方がよっぽど大事ですよね
ご冗談でしょう?名無しさん [sage] 2019/02/18(月) 13:38:31.68ID:???

大事だと思った理由はなんですか
相手は高校物理の力積習いたて(?)なんだよ
ご冗談でしょう?名無しさん [sage] 2019/02/18(月) 13:42:07.64ID:???
自慢したい奴って醜悪だね
ご冗談でしょう?名無しさん [sage] 2019/02/18(月) 13:50:27.38ID:???

力積習いたてなら、微分云々教えるのは間違えですよね
ご冗談でしょう?名無しさん [sage] 2019/02/18(月) 14:03:40.24ID:???

(高校生とは言ってない)
ご冗談でしょう?名無しさん [sage] 2019/02/18(月) 14:10:17.02ID:???
不毛な議論やめれ
ここはちよっとした物理の質問に答える場だから

・定義から明らか(だからそんな質問するやつはバカだ)
・どう考えてもadvancedな数学の理論まくしたてる

とかは別の所行ってくださいって話
ご冗談でしょう?名無しさん [sage] 2019/02/18(月) 14:14:55.98ID:???
ニールセン以外に量子情報やるのにいい本ある?
ご冗談でしょう?名無しさん [sage] 2019/02/18(月) 14:29:30.08ID:???
物理は出来ても人に教えるのは苦手な人は無理して質問スレで回答しなくても良いんですよ
むしろ黙っていてくれた方がありがたい
ご冗談でしょう?名無しさん [sage] 2019/02/18(月) 14:38:51.05ID:???

こういう回答者のことですね
ご冗談でしょう?名無しさん [sage] 2019/02/18(月) 15:17:44.59ID:???

いやの事だろ
話聞いてた?
ご冗談でしょう?名無しさん [sage] 2019/02/18(月) 15:21:23.11ID:???
聞いてましたよ?
ご冗談でしょう?名無しさん [sage] 2019/02/18(月) 15:28:39.02ID:???
>・どう考えてもadvancedな数学の理論まくしたてる

>693>694のことでしょうか。
ご冗談でしょう?名無しさん [sage] 2019/02/18(月) 17:38:03.80ID:???

それはただの厨房
691 [sage] 2019/02/18(月) 18:14:46.82ID:???

括弧の中↓の主張なんて書いとらんぞ

「・定義から明らか(だからそんな質問するやつはバカだ)」
ご冗談でしょう?名無しさん [] 2019/02/18(月) 19:12:41.91:PO98IxDn
準静的過程と可逆過程の違いはなんですか
ご冗談でしょう?名無しさん [sage] 2019/02/18(月) 19:38:57.24ID:???
熱力学状態が可逆な過程は結果だけ見れば準静的過程と同じ
準静的過程は必ず可逆過程
ご冗談でしょう?名無しさん [sage] 2019/02/18(月) 19:39:22.46ID:???
可逆⊃準静的
ご冗談でしょう?名無しさん [sage] 2019/02/18(月) 19:41:15.68ID:???
可逆は元に戻ればなんでも許される
準静的は辿ってきたルートをキッチリ戻らなくてはならない
ただし平行熱力学においてこれらは実は等価
ご冗談でしょう?名無しさん [sage] 2019/02/18(月) 19:41:32.79ID:???

読む人次第だから余計なお世話
お前だけスルーしとけ
ご冗談でしょう?名無しさん [sage] 2019/02/18(月) 19:46:58.82ID:???



0点
ご冗談でしょう?名無しさん [sage] 2019/02/18(月) 20:25:24.36ID:???
何が0点なんですか?
ご冗談でしょう?名無しさん [sage] 2019/02/18(月) 20:27:30.69ID:???
こっちにも0点くんがw
ご冗談でしょう?名無しさん [] 2019/02/18(月) 20:53:42.24:2EVfPim3
12歳で量子力学の本を書いた日本人天才少年がいるらしいね。本屋に置いてた。
ご冗談でしょう?名無しさん [sage] 2019/02/19(火) 00:34:56.37ID:???
彼は年間3000冊の本を読める天才ですからね
ご冗談でしょう?名無しさん [sage] 2019/02/19(火) 00:45:35.01ID:???
ヒキルナの両親とは別の自己陶酔タイプだったわ
うちの親は

フジコヘミングに感動するタイプ。
ご冗談でしょう?名無しさん [sage] 2019/02/19(火) 14:39:57.26ID:???
0点くんは内容が読解できないから
一字一句同じでないとそうなる
ご冗談でしょう?名無しさん [] 2019/02/19(火) 15:56:35.59:xVCIQcTt
 
人殺しの殺人鬼の池田大作の創価の公明が政治活動

キチガイの集まりのいやがらせキチガイ集団の創価の公明が政治活動

創価学校・創価大学・キチガイ養成学校の創価の公明が政治活動
 
 
ご冗談でしょう?名無しさん [sage] 2019/02/19(火) 23:51:02.40ID:???

ありがとうございます
熱力学以外だとどうなるのですか?
ご冗談でしょう?名無しさん [sage] 2019/02/19(火) 23:53:48.11ID:???
両者は等しくならない。摩擦のない真空中の振り子などは可逆だが準静的ではない。
ご冗談でしょう?名無しさん [sage] 2019/02/20(水) 00:25:34.31ID:???

等価ではない
ご冗談でしょう?名無しさん [sage] 2019/02/20(水) 01:35:10.73ID:???
等価だろ
ご冗談でしょう?名無しさん [sage] 2019/02/20(水) 13:53:34.48ID:???
等価だと思っている人は、0点ですね。
ご冗談でしょう?名無しさん [sage] 2019/02/20(水) 13:57:43.77ID:???
0点くん
ご冗談でしょう?名無しさん [sage] 2019/02/20(水) 14:02:09.59ID:???
等価ではない
ご冗談でしょう?名無しさん [sage] 2019/02/20(水) 14:04:21.10ID:???
>>715
こいつらは馬鹿

は正しいね
ご冗談でしょう?名無しさん [sage] 2019/02/20(水) 14:13:30.05ID:???
熱力学においては同じ意味だろ
ご冗談でしょう?名無しさん [sage] 2019/02/20(水) 14:15:49.54ID:???
いいえ
ご冗談でしょう?名無しさん [sage] 2019/02/20(水) 17:56:00.29ID:???
等価です
ご冗談でしょう?名無しさん [sage] 2019/02/20(水) 20:12:38.19ID:???
平衡熱力学において準静的過程ではない可逆過程の具体例を示してくれよ
そうしたら等価じゃないと納得してやるからさ
ご冗談でしょう?名無しさん [sage] 2019/02/20(水) 20:52:07.93ID:???
0点くんはなんにも分かってないから0点連呼するしかできないんだよw
ご冗談でしょう?名無しさん [sage] 2019/02/20(水) 21:50:18.52ID:???

等価というのならそれを証明すればいいだろう?
そっちの主張は証明無しで納得しろってか?
ご冗談でしょう?名無しさん [sage] 2019/02/20(水) 22:02:31.94ID:???
ムーニーちゃんしんぷだい
ご冗談でしょう?名無しさん [sage] 2019/02/21(木) 00:40:58.01ID:???

詭弁もここまで行くと感心してしまう
ご冗談でしょう?名無しさん [sage] 2019/02/21(木) 00:52:11.51ID:???

嘘だから証明できないよなぁ
証明要求を詭弁なんて言うような奴には熱力学は難しかったか…
ご冗談でしょう?名無しさん [sage] 2019/02/21(木) 01:00:18.85ID:???
基地外かよ
ご冗談でしょう?名無しさん [] 2019/02/21(木) 01:11:02.99:UBLlAL+T
可逆過程であるためには、変化の途中において、系内および系と周囲との間で熱平衡、力学的平衡、化学的平衡が保たれていることが必要であり、このような理想化した状態変化を準静的過程と言う。
可逆過程は常に準静的だが、準静的過程であっても可逆でないものは存在する.[1]。
たとえばピストンとシリンダーの間に摩擦が存在する状況下で気体を準静的に圧縮する過程は準静的だが可逆ではない[2]。
他の形のエネルギーが摩擦や抵抗により熱エネルギーに変わる現象は、常に非可逆となる。
ただし文献によって用語の混乱があり、可逆過程と準静的過程を同義に使う文献[3]もある。
ご冗談でしょう?名無しさん [sage] 2019/02/21(木) 01:11:31.04ID:???

つ第二法則
ご冗談でしょう?名無しさん [] 2019/02/21(木) 01:17:16.20:UBLlAL+T
準静的過程の定義について
ttp://https://home.hiroshima-u.ac.jp/atoda/Thermodynamics/r09junseikatei.pdf

ttp://http://www.cmp.sanken.osaka-u.ac.jp/~koun/therm/dsf/discuss-ch4.html
ttp://http://www.cmp.sanken.osaka-u.ac.jp/~koun/therm/material/thermplus.pdf
ご冗談でしょう?名無しさん [sage] 2019/02/21(木) 01:22:09.83ID:???

これが等価派の証明なのか…
第二法則から導いてくれよ
ご冗談でしょう?名無しさん [sage] 2019/02/21(木) 01:29:09.33ID:???
ある状態変化において、

準静的過程であること、
粘性、摩擦、非弾性、電気抵抗、磁気ヒステリシス、等によるエネルギーの散逸が生じないこと。

が満たされれば、この変化は可逆過程となる [4]。 ただし文献によって用語の定義に相違や曖昧さがあり、 可逆過程と準静的過程を同義に使う文献[5]もある。


ってウィキペディアにあるけど熱力学の準静的過程にエネルギー散逸がないことは含まれるのでは?

シリンダとピストン間の間の摩擦を、熱力学で扱って良いものか? この摩擦は力学的な考察の対象では?
ご冗談でしょう?名無しさん [sage] 2019/02/21(木) 01:44:41.39ID:???
は準静的過程は可逆過程の一部分だと思っているようだが、本当は逆で、可逆過程が準静的過程の一部なのである!
ご冗談でしょう?名無しさん [sage] 2019/02/21(木) 01:47:06.72ID:???
言葉の使い方が各人微妙に異なっているせいで議論がめちゃくちゃになる
ご冗談でしょう?名無しさん [sage] 2019/02/21(木) 02:09:01.14ID:???

準静的な場合に散逸が許されない理由は?
力学量として分けるってのも意味不明
力学的仕事も熱力学第一法則に入ってるじゃん
ご冗談でしょう?名無しさん [sage] 2019/02/21(木) 02:16:11.16ID:???
摩擦は状態量
ご冗談でしょう?名無しさん [sage] 2019/02/21(木) 02:18:18.08ID:???

はじめの質問は

0712 ご冗談でしょう?名無しさん 2019/02/18 19:12:41
準静的過程と可逆過程の違いはなんですか
返信 ID:PO98IxDn

なんだからこれにお前が答えれば済む話
ご冗談でしょう?名無しさん [sage] 2019/02/21(木) 02:31:37.99ID:???

それに対して嘘を答えたクズ野郎を注意したかっただけだよ
もう上に色々レスついてるし違うのはわかるよね?
ご冗談でしょう?名無しさん [sage] 2019/02/21(木) 02:37:14.79ID:???

正しくないじゃんw
ご冗談でしょう?名無しさん [sage] 2019/02/21(木) 02:37:40.30ID:???

わかりませーん
ご冗談でしょう?名無しさん [sage] 2019/02/21(木) 02:42:05.53ID:???

そうか
まあこんな奴すら回答者になれるのが5chの魅力だよな
ご冗談でしょう?名無しさん [sage] 2019/02/21(木) 02:42:57.76ID:???

で、回答まだ?
ご冗談でしょう?名無しさん [sage] 2019/02/21(木) 02:44:09.37ID:???

なんで回答しないといけないの?
質問者はもう納得したみたいだけど?
ご冗談でしょう?名無しさん [sage] 2019/02/21(木) 02:46:35.30ID:???
結局、誰も理解していないということで良いですか?
ご冗談でしょう?名無しさん [sage] 2019/02/21(木) 02:48:14.17ID:???

どうして納得したと判断できるんですか?
ご冗談でしょう?名無しさん [sage] 2019/02/21(木) 02:50:11.13ID:???

747ではないが、準静的過程って熱力学的平衡を保つ変化なのに、その過程で不可逆なエネルギー散逸を伴いうるっていうのがよくわからん
どういうことなの?
ご冗談でしょう?名無しさん [sage] 2019/02/21(木) 02:50:44.91ID:???

もうお礼もして熱力学以外の話に移ったみたいだからね
ご冗談でしょう?名無しさん [sage] 2019/02/21(木) 02:55:45.00ID:???

それが相反してるという考えになるのがわからない
別に矛盾しないでしょ
ご冗談でしょう?名無しさん [sage] 2019/02/21(木) 02:59:56.15ID:???

だから第二法則で論破してやるから準静的でない可逆過程の例を示せよ
ご冗談でしょう?名無しさん [sage] 2019/02/21(木) 03:01:33.96ID:???

そもそもの質問への答えは間違ってるんでしょ?
正しい答えを教えてあげなきゃダメじゃん
ご冗談でしょう?名無しさん [sage] 2019/02/21(木) 03:02:23.50ID:???

等価である証明をすれば論破できるよ?
ご冗談でしょう?名無しさん [sage] 2019/02/21(木) 03:07:14.83ID:???

それはボランティアだよね?
質問者があれで納得するようなら今更何を教えてもね…
ご冗談でしょう?名無しさん [sage] 2019/02/21(木) 03:09:11.38ID:???
5chがボランティアでなければなんなのか
八つ当たりの場?
ご冗談でしょう?名無しさん [sage] 2019/02/21(木) 03:09:43.31ID:???
エネルギー散逸がある場合は等価ではないってよく知られてると思ってたけどそうではないのかもね…
それともたまたまここのレベルが低かったのかな
ご冗談でしょう?名無しさん [sage] 2019/02/21(木) 03:10:23.87ID:???

ボランティアは強制するものではない
嘘を嘘だと言うのもボランティアだからね?
ご冗談でしょう?名無しさん [sage] 2019/02/21(木) 03:13:16.72ID:???
系をどこまで系とみなすか
http://eman-physics.net/thermo/jt_effect.html [sage] 2019/02/21(木) 03:16:29.48ID:???
準静的でありながら不可逆過程。前に「準静的過程は可逆過程だ」と説明したが、それを全く覆すような実験である。
このような、系の全体では平衡ではないが部分的に平衡状態が保たれた状況を「広義の準静的過程」だと見なすことがある。
「広義の準静的過程」を含める場合、可逆過程だとは限らないということだ。
http://eman-physics.net/thermo/jt_effect.html [sage] 2019/02/21(木) 03:17:19.42ID:???
準静的でありながら不可逆過程。前に「準静的過程は可逆過程だ」と説明したが、それを全く覆すような実験である。
このような、系の全体では平衡ではないが部分的に平衡状態が保たれた状況を「広義の準静的過程」だと見なすことがある。
「広義の準静的過程」を含める場合、可逆過程だとは限らないということだ。
ご冗談でしょう?名無しさん [sage] 2019/02/21(木) 03:17:38.36ID:???
平衡熱力学において準静的過程ではない可逆過程の具体例を教えて下さい
ご冗談でしょう?名無しさん [sage] 2019/02/21(木) 03:31:26.80ID:???
準静的過程とは、正方向で受け取る仕事量と熱量は、逆方向で受け取る仕事量と熱量と、値は同じで正負だけが異なる過程を指す。
したがって準静的過程は可逆過程である。
また準静的でない過程は、熱力学第二法則から不可逆であると証明できる現象を伴うので、不可逆過程である。
従って準静的過程と可逆過程は等価である。
ご冗談でしょう?名無しさん [sage] 2019/02/21(木) 03:38:36.04ID:???

それが証明ですか
正方向とか逆方向って何ですかね 
ご冗談でしょう?名無しさん [sage] 2019/02/21(木) 03:58:16.87ID:???
そもそもエネルギー散逸とはなんですか?
ご冗談でしょう?名無しさん [sage] 2019/02/21(木) 04:34:54.21ID:???
準静的過程とは状態図上で全て表現できる過程ではなかったのですか?
ご冗談でしょう?名無しさん [sage] 2019/02/21(木) 05:19:14.94ID:???
変化には非平衡が必要だから平衡を保ちつつ変化するという準静的過程などあり得ない自己矛盾
だから人により定義が曖昧にならざるを得ない
ご冗談でしょう?名無しさん [sage] 2019/02/21(木) 08:50:54.29ID:???
他人の解答にケチつけるだけのやつは一体なんなんだ?
正しい解答を加えれば済む話なのに深夜まで張り付いて…
どうしてそこまで頑ななんだろう
ご冗談でしょう?名無しさん [sage] 2019/02/21(木) 09:28:47.36ID:???
構ってちゃんだからだろ
712 [] 2019/02/21(木) 09:59:03.98:6yVJle3l
すみません
レスの流れを読んでいると、またわからなくなってしまいました
準静的過程と可逆過程は何が違うのでしょうか?
また包含関係はどうなっているのでしょうか?
ご冗談でしょう?名無しさん [sage] 2019/02/21(木) 11:52:09.53ID:???

摩擦などのエネルギー散逸は内部エネルギ変化でなく運動エネルギー変化であるから熱力学の埒外
棚橋「変形と流れの基礎」p.106
ご冗談でしょう?名無しさん [] 2019/02/21(木) 12:20:27.52:k67NXYjS
アインシュタイン多様体だと、重力方程式が求められるらしいけど、Ricも、実際の式もしらない素人です。
証明をみれば理解できるかとおもって、ネットだとどこにありますか。
ご冗談でしょう?名無しさん [sage] 2019/02/21(木) 13:06:52.74ID:???

断熱圧縮
ご冗談でしょう?名無しさん [sage] 2019/02/21(木) 13:08:48.03ID:???

場の古典論ならnetで見れるんじゃない?
ご冗談でしょう?名無しさん [sage] 2019/02/21(木) 13:47:08.50ID:???

その本を持っていないから内容は知らないが内部エネルギー散逸と運動エネルギー散逸の違いは?
摩擦によるエネルギー変化は儷=冫+儻に含まれない理由がわからない
他にも無限小の温度差のある物体の接触は運動エネルギー散逸なのか?
ちなみにピストンシリンダ系で摩擦があると摩擦で内部エネルギーは変化するけどね
ご冗談でしょう?名無しさん [sage] 2019/02/21(木) 13:48:26.29ID:???

意味不明
ご冗談でしょう?名無しさん [sage] 2019/02/21(木) 13:54:37.21ID:???
言葉通りに意味を捉えると運動エネルギーの変化は内部エネルギーの変化に繋がるんだけどね
冫や儻になにか制限をかけるということなんだろうけど
ご冗談でしょう?名無しさん [sage] 2019/02/21(木) 14:07:38.83ID:???
準静的過程は可逆過程の部分集合だよ
ご冗談でしょう?名無しさん [sage] 2019/02/21(木) 14:10:04.80ID:???
ピストンをゆっくり動かす系の話なら、ピストンにはたらく力のつり合いから、圧力に摩擦の効果が現れるよね?
ご冗談でしょう?名無しさん [] 2019/02/21(木) 14:56:20.89:k67NXYjS
アインシュタイン多様体と、重力方程式の証明がちゃんと書いてあるURLがわからない
ご冗談でしょう?名無しさん [sage] 2019/02/21(木) 15:07:20.43ID:???

包含関係逆じゃね?
可逆の方が制限が強い
ご冗談でしょう?名無しさん [sage] 2019/02/21(木) 18:07:45.11ID:???
いいえ
ご冗談でしょう?名無しさん [sage] 2019/02/21(木) 18:32:02.70ID:???
いいえじゃねえよタコ
ご冗談でしょう?名無しさん [sage] 2019/02/21(木) 20:41:40.30ID:???
ムニちゃーんポポ
ご冗談でしょう?名無しさん [sage] 2019/02/21(木) 21:15:45.02ID:???
中国内の借金総額が日本円で約1京円、天文学的数で京スパコンもビックリ
巨大な借金バブルが崩壊したら世界はどうなる?
ご冗談でしょう?名無しさん [sage] 2019/02/22(金) 10:14:46.58ID:???

合ってるよ。
例えば、振り子の運動(空気抵抗や熱の移動は考慮しない)は可逆変化だけど準静的過程では無いよ。
ご冗談でしょう?名無しさん [sage] 2019/02/22(金) 10:26:55.53ID:???
散逸のある準静的過程は可逆過程じゃないから包含関係にはならない
ご冗談でしょう?名無しさん [sage] 2019/02/22(金) 10:53:00.64ID:???

振り子の例は力学現象では
ご冗談でしょう?名無しさん [sage] 2019/02/22(金) 11:55:58.09ID:???
ベン図で書け、ベン図で
言葉だけでやり取りしても手間かかるだけだぞ
ご冗談でしょう?名無しさん [] 2019/02/22(金) 13:42:20.13:oMaHPw1E
これらは類似物か一般化らしいですが、方程式が成り立つ一般論はないですか?
量子論と相対論では数学自体別物らしいですがどちらか一方に統一か、別の一般論はできないですか。


場の方程式
万有引力の法則
マクスウェルの方程式
ナビエ-ストークス方程式
アインシュタイン方程式
クライン-ゴルドン方程式
ディラック方程式
サイバーグ-ウィッテン方程式
バーグマン-ウィグナー方程式
ご冗談でしょう?名無しさん [sage] 2019/02/22(金) 14:12:57.30ID:???
並べるだけで意味分かってねーな
ご冗談でしょう?名無しさん [sage] 2019/02/22(金) 15:52:16.86ID:???
方程式の成り立つ群ならあるだろ
ご冗談でしょう?名無しさん [] 2019/02/22(金) 18:13:48.06:oMaHPw1E
方程式が類似物か一般化とわかってないので、できるだけ簡単に一般的に導出する方法ないですか?


ttps://static.minne.com/productimages/8869835/large/0dd950ba020d726caaa9d91f5b36fb9bbd7006cd.jpg
ttp://https://minne.com/items/2501022
ご冗談でしょう?名無しさん [sage] 2019/02/22(金) 18:42:16.17ID:???
各々の導出は入門書でも書いてあるけどそれじゃない情報がほしいのか?
何が知りたいのかわからない
ご冗談でしょう?名無しさん [] 2019/02/22(金) 18:44:28.98:oMaHPw1E
個別に全部やるのが大変で、一括してまとめられないですか。一般化された方程式の導出法が一個あればいいんですが。
ご冗談でしょう?名無しさん [sage] 2019/02/22(金) 18:48:32.11ID:???
>>ID:oMaHPw1E
キチガイ
ご冗談でしょう?名無しさん [sage] 2019/02/22(金) 20:08:14.61ID:???
最小作用の原理から全部出せる。以上
ご冗談でしょう?名無しさん [] 2019/02/22(金) 21:12:24.54:oMaHPw1E
サンクス
ご冗談でしょう?名無しさん [sage] 2019/02/22(金) 21:55:09.30ID:???
ムーニーちゃんしんぷのおじさんだい
ご冗談でしょう?名無しさん [sage] 2019/02/23(土) 00:54:03.63ID:???

変分原理と(相対論まで含んだ一般の)ゲージ原理の二者になんか付け足す原理ってある?。
物理学の根本原理として挙げられそうなのものの中で。
ご冗談でしょう?名無しさん [sage] 2019/02/23(土) 01:00:19.97ID:???
最小作用の原理でハイゼンベルグ模型出すのは無理
ご冗談でしょう?名無しさん [sage] 2019/02/23(土) 01:02:06.28ID:???
相対論の前提として光速度不変の原理は必須じゃなかろうか
ご冗談でしょう?名無しさん [sage] 2019/02/23(土) 01:03:27.69ID:???

ローレンツスカラーで書いてる時点で自明に含まれてるだろ
ご冗談でしょう?名無しさん [sage] 2019/02/23(土) 01:04:00.65ID:???
なぜ数学を使うといいのか、使えるのかを示す必要がある
数学を使うとは、連続時空があることをすでに組み込むこと
数論、実数をそのまま物理に使える根拠がよくわからない
ご冗談でしょう?名無しさん [sage] 2019/02/23(土) 01:13:31.21ID:???

関数解析の公理系はなんか別枠みたいな。
超対称性絡みが幾何学と関数解析絡み間の橋渡しって感じ?。

膀胱全摘オズラさん(巨泉の腰ぎんちゃ:「パソコンサンデーじゃなくてCCRカーグランプリ!
ご冗談でしょう?名無しさん [sage] 2019/02/23(土) 01:40:26.26ID:???

それをがんばってやるのが物理学の目標のひとつだよ

もし出来たら君の名前のついた法則が生まれる
ご冗談でしょう?名無しさん [hage] 2019/02/23(土) 02:30:07.95ID:???
未来人目線で無双したいのか
ご冗談でしょう?名無しさん [sage] 2019/02/23(土) 08:01:17.15ID:???
一番安定している原子って何?
水素とヘリウムではどっちが原子崩壊しないの?
全ての原子の中で一番崩壊しにくいものってなに?
ご冗談でしょう?名無しさん [sage] 2019/02/23(土) 08:18:18.46ID:???
鉄じゃないの
ご冗談でしょう?名無しさん [sage] 2019/02/23(土) 11:00:25.42ID:???
最近EMANも言ってるけど、化学って何であのあやふやな理論でやっていけるの?
どう見ても「理論はこうだ! これとこれを混ぜれば新物質が出来る!」じゃなくて
「(大量の組み合わせや触媒を試して)これとこれを混ぜたら出来た! 後からこじつけると理屈はこうだ!」
ばっかじゃねえか
ご冗談でしょう?名無しさん [sage] 2019/02/23(土) 11:16:51.90ID:???

幼稚な考えだな、物理学の理論範囲でも複雑系は手に負えない
複雑系の化学は物理学に還元できないから個々のケース理論で対応する。
分子生物学も同様
ご冗談でしょう?名無しさん [sage] 2019/02/23(土) 12:09:43.40ID:???

計算機パワーが足りなかったから。

最近はGPUとか高性能な計算機が出てきて
>「理論はこうだ! これとこれを混ぜれば新物質が出来る!」
が実践できるようになってきた。
ご冗談でしょう?名無しさん [sage] 2019/02/23(土) 12:56:59.12ID:???

それは対象の原子の数が少数の時だけ、原子の数が増えると指数関数的に計算量が増える。
量子論の基本方程式から解くのは京スパコンでも無理だ。
ご冗談でしょう?名無しさん [sage] 2019/02/23(土) 13:10:23.79ID:???
以前に炭化水素の燃焼さえ計算できないって聞いたけど
今はどうなんだろ?
ご冗談でしょう?名無しさん [sage] 2019/02/23(土) 13:11:44.21ID:???
高温超伝導だって圧倒的に実験先行だし
ご冗談でしょう?名無しさん [sage] 2019/02/23(土) 13:41:26.72ID:???

最近の第一原理計算の進展も知らない雑魚が知ったかするなよ
ご冗談でしょう?名無しさん [sage] 2019/02/23(土) 14:03:29.94ID:???
揚げ足かもしれませんが、計算量は指数関数では増えないですよね?
ご冗談でしょう?名無しさん [sage] 2019/02/23(土) 14:05:23.81ID:???

シッタカ馬鹿だろ
>最近の第一原理計算
サンプルソフトさえ今性能のPCで実行しても遅くて使い物にならない、100台でも無理だろ
所詮シミュレーションだから設定パラメータだらけで結局、実験データを持ってこないとダメなんだよ。
ご冗談でしょう?名無しさん [sage] 2019/02/23(土) 14:08:42.17ID:???

脊髄反射で煽り返しても無知晒すだけだぞ
固体物理2017年11月号読んでこい
ご冗談でしょう?名無しさん [sage] 2019/02/23(土) 14:08:46.33ID:???

量子論の方程式そのままなら指数関数的に増える、シミュレーション計算ではハナから近似計算。
ご冗談でしょう?名無しさん [sage] 2019/02/23(土) 14:27:01.71ID:???

どういう計算すると計算量がO(α^N)になるんですか?
αがいくつかも知りたいです
ご冗談でしょう?名無しさん [sage] 2019/02/23(土) 14:27:19.25ID:???

実験データと組み合わせた近似計算が実用レベルに近づいたのは確かだろう
量子論の方程式から直接計算できるとかの夢のような話ではない。
ご冗談でしょう?名無しさん [sage] 2019/02/23(土) 14:27:28.54ID:???
ダンシング ムーニー
ご冗談でしょう?名無しさん [sage] 2019/02/23(土) 14:31:09.26ID:???

誰もそんな話してない
コーンシャムも知らない雑魚が勝手に「QEDレベルからの物性予測物質予測は不可能!」とか喚き散らしても滑稽でしかないわけ
ご冗談でしょう?名無しさん [sage] 2019/02/23(土) 14:33:37.17ID:???
硫化水素なんかの高温超伝導が理論先行で発見されたことも知らないんだろうな
ご冗談でしょう?名無しさん [sage] 2019/02/23(土) 14:35:40.34ID:???

n個の粒子の波動関数を求めるには3n次元の偏微分方程式を解くのと同じになる
ご冗談でしょう?名無しさん [sage] 2019/02/23(土) 14:37:27.74ID:???

それはO(3n)ですよね?
ご冗談でしょう?名無しさん [sage] 2019/02/23(土) 14:39:52.24ID:???
全然違う
ご冗談でしょう?名無しさん [sage] 2019/02/23(土) 14:42:35.45ID:???
じゃあどうしたらO(α^N)になるんですか?
ご冗談でしょう?名無しさん [] 2019/02/23(土) 14:44:56.64:Xa7ShRGq
【ほらみろ人工地震じゃないか!】 鳩山ポッポ″由紀夫「CSSが原因、中止せよ」 ⇒ 直後に地震
ttps://rosie.5ch.net/test/read.cgi/liveplus/1550888671/l50
ご冗談でしょう?名無しさん [sage] 2019/02/23(土) 14:48:39.33ID:???
あ、何となくわかりました
ご冗談でしょう?名無しさん [sage] 2019/02/23(土) 15:22:27.29ID:???
シミュレーション計算の時間は四則演算の時間だと思ってるバカが多いが、とんでもない
偏微分方程式の計算時間などは殆どが遅いメモリーデータとの転送回数とその転送時間で決まる。
ご冗談でしょう?名無しさん [sage] 2019/02/23(土) 15:48:59.87ID:???
どれだけ多いんですか?
ご冗談でしょう?名無しさん [sage] 2019/02/23(土) 15:55:30.78ID:???

その「転送時間」が構成要素の数に対してどう依存するかという問題だろうが
句読点使う奴は馬鹿しかいないのか
ご冗談でしょう?名無しさん [sage] 2019/02/23(土) 15:56:09.59ID:???
その転送も計算量が増えれば増えるんだから、結局計算量って重要でしょって話になる
ご冗談でしょう?名無しさん [sage] 2019/02/23(土) 16:07:44.63ID:???
すこしは考えろ 3n次元で1000の格子辺モデルなら1000^3nの格子がある
1つの格子データには2x3nの周りの格子のデータによる計算が必要になる。
ご冗談でしょう?名無しさん [sage] 2019/02/23(土) 16:43:03.55ID:???
格子QCDは、すさまじい計算量
ttp://http://bridge.kek.jp/lecture/02-aoki/LectureNote.pdf
p.22
「例えば、 10⁴ の格子を考てみよう。ゲージ場の独立成分は 8(SU(3) の生成子の数) ×4 (方向 μ の数)あるので、各成分をメッシュ 10 で区分求積すると必要な浮動小数点演算の数 (Flop) は 10^(3.2×10⁵) になる。
現在の世界最速である 10 Peta Flops クラスの計算機で計算しても、10³¹⁹⁹⁷⁵年の計算時間が必要になり、(宇宙年齢より遥かに長いので)実行不可能である事が分かる。」
ご冗談でしょう?名無しさん [sage] 2019/02/23(土) 16:50:07.89ID:???

それってBCSの延長じゃん
ご冗談でしょう?名無しさん [sage] 2019/02/23(土) 16:50:40.18ID:???
物質設計にそんなん要らんがな
ご冗談でしょう?名無しさん [sage] 2019/02/23(土) 16:51:34.24ID:???

だから何だよ
BCSの延長上にない超伝導なんてねえぞ
ご冗談でしょう?名無しさん [sage] 2019/02/23(土) 17:31:47.32ID:???
高温超伝導ってなかなか実用化されないのはどうして?
ご冗談でしょう?名無しさん [sage] 2019/02/23(土) 17:42:30.86ID:???
高温超伝導もBCSの延長なんだね
知らなかったよ
ご冗談でしょう?名無しさん [sage] 2019/02/23(土) 17:44:35.59ID:???
結局は計算機という工業製品の性能アップの恩恵であって、物理学の理論の勝利というような感じはしないなあ
ご冗談でしょう?名無しさん [sage] 2019/02/23(土) 18:04:46.50ID:???

皮肉ってるつもりなんだろうが今知られている超伝導は全てBCSの延長にある
ご冗談でしょう?名無しさん [sage] 2019/02/23(土) 18:05:58.08ID:???
電子の質量消滅すれば超伝導になる。
ご冗談でしょう?名無しさん [sage] 2019/02/23(土) 20:41:57.56ID:???
BCSの延長とはなんですか?
ご冗談でしょう?名無しさん [sage] 2019/02/23(土) 20:50:54.42ID:???

すでに軍産複合体の中で大統一理論は出来上がっているさ。
それから見れば相対性理論は子供騙し

公開することは、社会全体総てを変えてしまうからな。決して公開されることはない。

君がそれを成し遂げたとしてもだ。
彼らはそれを許さない。

例え、地球変動や社会崩壊へと進もうがそれは現状維持を保とうとするだろう。
ご冗談でしょう?名無しさん [] 2019/02/24(日) 02:39:05.68:l9KzOXew
位相多様体、代数多様体、リーマン多様体(微分幾何)はあるのに、
関数解析、関数空間の多様体は聞かないとおもった。
量子論では多様体が出てこないような。
物理理論として量子論が優れていてすでに多様体相当を内在してる?
量子論は特殊相対性理論相当ってない?
ご冗談でしょう?名無しさん [] 2019/02/24(日) 03:09:24.55:l9KzOXew
これかな


アラン・コンヌ - Wikipedia
アラン・コンヌ(1947年4月1日 - )はフランスの数学者。IHES、コレージュ・ド・フランスおよびオハイオ州立大学教授。
作用素環論や非可換幾何の研究で知られる。
作用素環論の幾何学への応用を通じ、積の交換法則が成り立たない作用素環によって表されるような「非可換空間」を扱う非可換幾何のパラダイムを提唱した。
1990年代には他の数学者とともに量子ホール効果、超弦理論、ループ量子重力理論、格子ゲージ理論など様々な量子力学的概念に対し非可換幾何の手法が有効であることを示している。
また、同じ時期に数論的な構成物に対しても非可換空間の構成が可能であることを示し、有数体 Qのアデール類の空間 A/Qxに対する自然な力学系からリーマンゼータ関数の零点のスペクトル実現を得ている。


作用素環論 - Wikipedia
作用素環論とは、作用素環とよばれるクラスの位相線型環を主に研究する数学の分野である。
研究対象の直接的な定義からは複素数体上無限次元の線型代数学と言え、普通関数解析学に分類されている。
しかし、その手法や応用はいわゆる代数学・幾何学・解析学の諸分野に幅広くわたり、アラン・コンヌが提唱する非可換幾何の枠組みを与えていることでも特筆される。
作用素環論の主な目標として、作用素環によって「非可換」化・量子化された幾何的対象を表現し、通常の図形と位相群などとを統一的に理解することや、
それらに対するホモロジー・コホモロジー的な理論(K理論)の構成と理解などが挙げられる。
ご冗談でしょう?名無しさん [sage] 2019/02/24(日) 03:55:18.26ID:???

多様体の定義を調べよ。
線型空間も多様体。
ご冗談でしょう?名無しさん [] 2019/02/24(日) 04:24:05.19:l9KzOXew
多様体はしってるけど。
しかし可換環 ≒ アフィン代数多様体 ≒ アフィンスキームで、可換環も多様体とみれるけど一般の代数多様体もある。
それからして非可換、関数解析の一般の多様体を考えるのは自然では。



アラン・コンヌ博士の非可換幾何学とは?
非可換幾何学入門:アラン・コンヌ
代数幾何学によって、幾何学的な空間と可換環論との関係はあきらかになった。
この本の目的は実解析学の範疇で可換を越えたところでの幾何学的な空間と関数解析との同じような対応を示すことである。

この理論を支えるのは本質的な三本の柱である。
1)自然に現れて、古典的な解析学の手法を適用することはできないが、非常に自然に非可換代数を対応させることができる数多くの例。
たとえば、ペンローズの宇宙の空間、葉層多様体の葉全体の空間、離散群の既約表現全体の空間など。

2)測度論、位相、微分演算、計量などの古典的な解析学の手法の、代数やヒルベルト空間を用いた再定式化。

3)物理学者が考察する空間の多くが非可換として捉えられるという意味での物理学との関係。
まず、ハイゼンベルクによって行列力学という形で発見された量子力学は、古典力学の相空間上の関数全体の可換環を非可換なものに置き換えた。
次にベリサールの固体物理学における仕事によって、そのエネルギーと運動量の空間は、非可換なものとなった。
最後に、ワインバーグ-サラムモデルによる素粒子物理学は時空間を支配する幾何学をあきらかにしたが、
その幾何学は非常に微妙なものであって、われわれがそこに非可換幾何学を考える余地を残している。
ttp://https://blog.goo.ne.jp/ktonegaw/e/5f5fc6fd565dbd789d1129c23986c849
ご冗談でしょう?名無しさん [sage] 2019/02/24(日) 05:07:34.72ID:???

>多様体はしってるけど。
知ってたら、こんな↓意味不明の文は書けん。

「関数解析、関数空間の多様体は聞かないとおもった。 」
ご冗談でしょう?名無しさん [] 2019/02/24(日) 05:29:45.54:l9KzOXew
位相、代数、微分については多様体、幾何学版があるのに、
量子論でつかう関数解析には多様体版が無いとおもった。
一般相対性理論のように多様体へ拡張できないかと。
ご冗談でしょう?名無しさん [sage] 2019/02/24(日) 05:39:48.45ID:???

非線形への拡張という意味なら、重ね合わせの原理が成り立たないので無理。
ご冗談でしょう?名無しさん [sage] 2019/02/24(日) 11:15:00.22ID:???
量子論≒関数解析は本質的に無限次元化した線形代数。

多様体論は局所的に(有限次元の)線形代数で表現できるフレームやらバンドルやらを張り合わせた概念。
ご冗談でしょう?名無しさん [sage] 2019/02/24(日) 13:18:23.20ID:???
そもそも非可換幾何じゃ空間さえ存在しないからな
ご冗談でしょう?名無しさん [sage] 2019/02/24(日) 18:40:01.31ID:???
超伝導理論はBCSで完成したの?
超伝導の工学的進展が無いのはどうして?
ご冗談でしょう?名無しさん [sage] 2019/02/24(日) 18:46:19.66ID:???
ありますよ?
ご冗談でしょう?名無しさん [sage] 2019/02/24(日) 18:52:15.04ID:???

JRマグレブがあるでしょ。

イットリウム系高温超伝導体も、永久電流スイッチ等に使われている。
ご冗談でしょう?名無しさん [sage] 2019/02/24(日) 21:30:26.66ID:???
高温超伝導はいまだ発現機構さえ微妙なんだよなぁ
ご冗談でしょう?名無しさん [sage] 2019/02/24(日) 22:19:07.20ID:???
固体物性のコンピュータシミュレーションで発見した高温超伝導物質を実際作ったとか聞いたことないな。
ご冗談でしょう?名無しさん [sage] 2019/02/24(日) 22:38:34.11ID:???
高温超伝導はTcが高いというだけであって従来型超伝導も含まれる


高温と言ってるあたり銅酸化物(や鉄系)の話してるんだろうが、機構が微妙っていつの時代の話をしてるんだよ


その貴重な例が上で挙げた硫化水素
ご冗談でしょう?名無しさん [sage] 2019/02/24(日) 22:42:56.82ID:???

微妙ってのは断定できないってことですよ
従来型BCSだったら格子振動の質量依存性やフォノンのピークとか完全にフォノンを介しているとわかるけど

銅酸化物は言い切れる証拠はあるんですか?高温になる理由もはっきりとわからんらしいし
ご冗談でしょう?名無しさん [sage] 2019/02/24(日) 22:48:04.94ID:???

いつの時代の解説読んだのか知らんが銅酸化物はスピン揺らぎで決着付いてる
未だにやってんのは擬ギャップの起源だとかCDWだとかそういう話
ご冗談でしょう?名無しさん [sage] 2019/02/24(日) 23:04:02.73ID:???
まあでも専門家的には全然満足できないようだからその人達が微妙だと言うのは別にどうでもいい
ご冗談でしょう?名無しさん [sage] 2019/02/24(日) 23:12:26.14ID:???
>銅酸化物はスピン揺らぎで決着付いてる

馬鹿丸出し
ご冗談でしょう?名無しさん [sage] 2019/02/24(日) 23:14:29.44ID:???

どのレベルで異論あるわけ?
ご冗談でしょう?名無しさん [sage] 2019/02/25(月) 01:26:03.57ID:???
ムニちゃーんポポ
ご冗談でしょう?名無しさん [sage] 2019/02/25(月) 13:03:10.93ID:???
銅酸化物超伝導とスピンは前から分かってるとは言え
決着付いてるとは言わんだろう
ご冗談でしょう?名無しさん [sage] 2019/02/25(月) 19:19:12.04ID:???
レッツ ムーニー ダンシング
ご冗談でしょう?名無しさん [sage] 2019/02/25(月) 20:13:12.27ID:???
スクイーズド光の発生に理論限界って無いの?
式の上では片方を犠牲にすればいくらでも精度上がりそうだけど
ご冗談でしょう?名無しさん [] 2019/02/25(月) 22:18:28.91:/+9orEOY
ニュートン2018年12号で
「原始ブラックホールが無ければ初期宇宙に超巨大ブラックホールが存在する事を説明できない」みたいな話になっているんだけど、
これは、恒星質量ブラックホールが超巨大ブラックホールに成長するには時間がかかるという事だね?
恐らくエディントン限界によって質量の降着率に上限が有るという事だと解釈していたんだけど、
降着物質に角運動量が無くて降着円盤ができずに直線的にブラックホールに落下した場合、
降着物質は光を発しないからエディントン限界とか関係ないのじゃないのか?

降着円盤が出来ずに直線的に質量を吸収出来たらブラックホールが成長する速度に限界は無くなるんじゃないの?
ご冗談でしょう?名無しさん [sage] 2019/02/25(月) 22:52:45.29ID:???
当該記事が手元にないけど、原始ブラックホールにしても恒星質量ブラックホールにしても
超巨大ブラックホールに成長するのはブラックホールの合体じゃないの?エディントン限界関係ある?
少なくとも、原始ブラックホールだったら妥当な時間で超巨大ブラックホールに成長できて、
恒星質量ブラックホールだと時間がかかりすぎるという論理がよくわからない
ご冗談でしょう?名無しさん [sage] 2019/02/25(月) 23:04:28.08ID:???

馬鹿すぎ
ご冗談でしょう?名無しさん [] 2019/02/25(月) 23:04:40.38:/+9orEOY

>超巨大ブラックホールに成長するのはブラックホールの合体じゃないの?

それ昔、宇宙板とかで質問していたんだけど、
「複数のブラックホールの中で合体が発生すると合体できたブラックホール以外は弾かれて合体にブレーキがかかるから限界が有る」みたいな話になっていた。
ご冗談でしょう?名無しさん [sage] 2019/02/25(月) 23:16:29.38ID:???

どの解説に決着がついたって載ってるんですか?
ご冗談でしょう?名無しさん [sage] 2019/02/25(月) 23:22:03.01ID:???

こうかな?
ご冗談でしょう?名無しさん [] 2019/02/25(月) 23:25:40.63:/+9orEOY

>「原始ブラックホールが無ければ初期宇宙に超巨大ブラックホールが存在する事を説明できない」

              ↓訂正

「初期宇宙に超巨大ブラックホールが形成される過程が謎である」
「巨大ブラックホールに成長する為の時間が足りないから・・・」


ごめん、原始ブラックホールはあまり関係ない。
ご冗談でしょう?名無しさん [sage] 2019/02/25(月) 23:26:10.06ID:???

合体ではなく周りの物質の降着で成長するんだとすると、ますますエディントン限界によって
降着率に上限ができる、という説の論理がわからない。それが正しければ、原始BHだって
成長に時間がかかるはずでは?というか、原始BHが成長して恒星質量BHになったら
あとは同じなので、原始BHが成長して恒星質量BHになるまでの時間が余計にかかりそうだけど
ご冗談でしょう?名無しさん [sage] 2019/02/25(月) 23:27:15.20ID:???

入れ違いになっちゃった。それなら了解
a4 ◆L1L.Ef50zuAv [] 2019/02/25(月) 23:59:37.87:4aoQm3Ou
英語でMITやCalTechのような人たちと量子計算やタイムマシンなどの物理について議論
できる掲示板とかのグループを探しているのですが、お勧めのサイトとかご存知の方
はいらっしゃるでしょうか?
ご冗談でしょう?名無しさん [sage] 2019/02/26(火) 00:03:26.16ID:???
stackexchange
a4 ◆L1L.Ef50zuAv [] 2019/02/26(火) 00:10:21.29:iTAeKIEa

Thank you for your information. I will try to use it.
ご冗談でしょう?名無しさん [] 2019/02/26(火) 00:49:09.82:vjaNCGJ/
S-双対、T-双対、U-双対とはなんですか?
違い不明です
ご冗談でしょう?名無しさん [] 2019/02/26(火) 01:20:26.49:vjaNCGJ/
ここに書いてありました。S、T


超弦理論の双対性
素粒子が粒子でなく弦でできているとすると、時空は10次元でないといけないことが80年代に明らかになりましたが、
詳細にみるとIIA 型、IIB 型、I 型、混成 E8×E8、混成 SO(32)と呼ばれる五種類の弦理論が許されることが判明し、理論家を悩ませていました。
数年経って、二種の II 型弦及び二種の混成弦は、それぞれ小さな円周上に於いて観察すると実は同じ弦理論であることが認識されます(T双対)。
90年代半ばには、I 型弦と混成 SO(32)弦は結合定数が互いの逆数である関係にあること(S双対)、また11次元に膜が基本構成要素であるM理論があり、
それを円周上及び線分上に於いて調べると、それぞれ IIA型及び混成 E8×E8 弦となっていることが判明し、五種の弦理論は全て結局同じ物理系であったことが理解されました。
これら「双対性」は今世紀の超弦理論研究の基礎になっています。
ttp://https://www.ipmu.jp/sites/default/files/webfm/pdfs/News17/J_YujiTachikawa.pdf
a4 ◆L1L.Ef50zuAv [] 2019/02/26(火) 01:49:02.77:iTAeKIEa

ttp://https://physics.stackexchange.com/questions/462897/is-global-optimization-of-quantum-computation-possible-i-want-to-study-in-mit

早速投稿したんですが、votesがマイナスになっていく一方ですね。それで逆に
目立ってはいるのですが。多世界解釈の話はタブーなのかな。初めて使うのでまだ
よくわからないです。
ご冗談でしょう?名無しさん [sage] 2019/02/26(火) 10:23:04.26ID:???

Page not found
This question was removed from Physics Stack Exchange for reasons of moderation.

早っ
ご冗談でしょう?名無しさん [sage] 2019/02/26(火) 12:41:28.24ID:???
www
ご冗談でしょう?名無しさん [sage] 2019/02/26(火) 13:22:01.90ID:???
マイナスになる理由は詳しく出ないのか
ご冗談でしょう?名無しさん [sage] 2019/02/26(火) 13:47:21.94ID:???

多世界解釈の話だったからじゃないと思うぞ、たぶん
a4 ◆L1L.Ef50zuAv [] 2019/02/26(火) 17:04:45.92:iTAeKIEa

光速より速いものがあるか?について議論しているものも同じくvotesがマイナスに
増えていってます。読者数すなわちviewsは一気に50人近くに上がっていったのですが。
404になる前に削除の理由が書いてあったのを見たのですが、物理のmainstreamなどに
合っていなく、既存の理論を逸脱しているというようなものでした。それらはoffの話題
として削除されるように、規定があると。2^100以上の世界を繋げることにあたかも
成功したような文体で書いたので、核兵器が飛ぶ話になってしまったためだと思って
います。でも、こう書くと、現代では、僕が統合失調症になり、嘘ではなく、お薬を
飲みましょう、ということになるようです。
ご冗談でしょう?名無しさん [sage] 2019/02/26(火) 17:11:14.45ID:???
SFはスレチってことでしょ
ご冗談でしょう?名無しさん [sage] 2019/02/26(火) 17:21:31.26ID:???
電位係数と電位容量係数と違いというか関係性がよくわかりません
教えて下さい
ご冗談でしょう?名無しさん [sage] 2019/02/26(火) 17:21:57.31ID:???

電気容量係数です
ご冗談でしょう?名無しさん [sage] 2019/02/26(火) 17:50:01.49ID:???

SFなのかどうかわからんが、唐突に「核兵器が飛ぶ話になってしまった」などというのが出てくるあたりで
俺も「お薬を飲みましょうね」と言いたくなるぞ
ご冗談でしょう?名無しさん [sage] 2019/02/26(火) 18:02:52.40ID:???
でも多世界解釈が正しいと仮定すると脈絡なく核兵器が飛び交って滅ぶ世界もあるはずって指摘は興味深いと思うぞ
a4 ◆L1L.Ef50zuAv [] 2019/02/26(火) 18:07:20.15:iTAeKIEa

僕は量子コンピュータがイオンがn個あると、2^n通り同時に多世界解釈で計算できる。
と、聞いてから、大域最適解を探索する方法で、自動定理証明や作曲などを考えるよう
になったんですよ。でも、Groverのアルゴリズムが、その計算には速度が最適で、
たかだか2^100通りくらいしかノイズをクリアしても数学的に計算できないんじゃ
ないかって。これなら全ての世界で核は飛んでいなさそうです。ところが、ブラック
ホール近傍の量子の複製という非ユニタリな法則を使うと、2^10000通りくらいでも
その中から望みのものを1つ持ってくることができると論文を見ながら思いつきました。
僕は議論を発展させて、ナマズが地震予知できるなら、ナマズに生化学的な特異点が
入っているのではないか?と考え、LHCではなくても、日本式でできるのでは
ないかと。この理論が合ってるか議論して、量子コンピュータを創るための投資が
欲しいと書いたら、すぐ404になってしまいました。このスレで議論できますか?
物理板にスレを建てたことはあったのですが、過去ログへすぐ行ってしまいました。
ご冗談でしょう?名無しさん [sage] 2019/02/26(火) 18:10:23.26ID:???
そりゃ
脈絡なく核兵器が飛び交って滅ぶ世界も
脈絡なく隕石が落ちてきて滅ぶ世界も
脈絡なく間近で超新星爆発が起こって滅ぶ世界も
...
その他いろいろあるはずだわな。で、それ興味深いか?物理として。
a4 ◆L1L.Ef50zuAv [] 2019/02/26(火) 18:18:03.94:iTAeKIEa

物理として言うなら、2^100000000とか無限に増やしていくと、1つくらいの世界では、
ドラえもんのスモールライトのような技術が出来上がる世界があるのではないか?などと
考えてます。
ご冗談でしょう?名無しさん [sage] 2019/02/26(火) 18:18:53.23ID:???
完全にSFだな
a4 ◆L1L.Ef50zuAv [] 2019/02/26(火) 18:23:36.30:iTAeKIEa

一応、僕はこれに関して論文を書きました。下記に示します。前者は34ページあります
が、量子コンピュータはこんな計算をすることもできない、というのを証明したもの
です。後者は英語で4ページで、量子コンピュータ以外のことも書かれてますが、
大域最適解を探索する数学的手法を載せてあります。

量子アルゴリズムの提案
ttp://http://www.01ken.com/report.pdf
Contradiction on science and Introduction to philanthrophy
ttp://http://www.01ken.com/cosaitp.pdf
ご冗談でしょう?名無しさん [sage] 2019/02/26(火) 18:30:59.68ID:???
怖くて触れんわ
ご冗談でしょう?名無しさん [sage] 2019/02/26(火) 19:03:41.49ID:???
序論からツッコミどころの山で草
ご冗談でしょう?名無しさん [sage] 2019/02/26(火) 20:22:56.71ID:???
日本語でも英語でもいいから文章の書き方を勉強してくれ
論文ってのは人に伝えるためのものなんだ
a4 ◆L1L.Ef50zuAv [] 2019/02/26(火) 20:43:04.68:iTAeKIEa

わかりやすく書いたつもりなのですが、伝わらなかったのなら、申し訳ございません。
僕は東大工学学士で、東大の教授が理解できれば大丈夫なように書きました。大学院へ
は行けなかった(留学はまだ考えてる)ので、論文の書き方に関しては、これから是正
する余地のあるところであるとは考えています。質問があるのならば受け付けます。
ご冗談でしょう?名無しさん [sage] 2019/02/26(火) 21:02:05.11ID:???

肝心の(105)だが、この時間の式って
どうやって出すの?
ご冗談でしょう?名無しさん [sage] 2019/02/26(火) 21:02:56.95ID:???
きっと君以外の誰にも理解できないよ
あとここは質問スレだからそういう活動は自分のブログか何かでやってください
ご冗談でしょう?名無しさん [sage] 2019/02/26(火) 21:12:50.30ID:???

くっくっく氏並みの猛者かな?
量子計算にかかる時間ってどうやって出すのか概略をプリーズ。
a4 ◆L1L.Ef50zuAv [] 2019/02/26(火) 21:15:18.20:iTAeKIEa

このpdfは昔書いたものなので、あまり覚えてないですが、ざっと文献を引くと、
"Quantum Computation and Quantum Information"という本の 262ページに
載ってます。少し長いので全部ここに書けません。ここは肝心では無いですよ。
pdfの16ページの下のほうのアルゴリズムのほうが速いですから。まだ質問あれば
受け付けます。


物理板にスレを建てて活動したいです。物理板に建てたことはあるのですが、
書き込みがあまりないと削除されるアルゴリズムみたいになっていて、上手く
いきません。これに関して何かご意見ありますでしょうか?人工知能のブログの
ようなものなら、こちらでやってます。プログラム技術板のほうです。

a4です。P2P人工知能「T」開発(4)
ttps://mevius.5ch.net/test/read.cgi/tech/1546530651/l50
ご冗談でしょう?名無しさん [sage] 2019/02/26(火) 21:17:42.88ID:???

ざっと見た感じなかなかまともな方だね
時間ができたらゆっくり読みます
a4 ◆L1L.Ef50zuAv [] 2019/02/26(火) 21:25:51.43:iTAeKIEa

巷で作られているのは、使い方が大きく2通りあります。1つ目はShorのアルゴリズム
による素因数分解。これは、整数Nに対して、(log(N))^2の時間でできます。
2つ目はGroverのアルゴリズムや量子焼きなましによる最適解探索で、
2^n通りを関数に入れてその中から計算結果が最大のものを取ろうとすると、
sqrt(2^n)回ほど、関数を計算する必要があり、2^100ほどが最大の並列度だと
考えてます。焼きなましは並列度が上がり、ことは単純ではないですが、
局所最適解を探索するときでも、計算速度は同じくらいです。

これを越えようと思って、非ユニタリ量子計算を思いつきました。それが2つ目の
pdfに載ってるものです。関数を1回だけ計算して、log(2^n)ほどで大域最適解を
探索します。
a4 ◆L1L.Ef50zuAv [] 2019/02/26(火) 21:29:25.71:iTAeKIEa

5chでは誹謗中傷のような人も寄ってくるので、同じような手法で対応しないと
いけないときもありますが、基本的に真面目と言われます。お読みになって
いただけるそうで、ありがとうございます。
ご冗談でしょう?名無しさん [sage] 2019/02/26(火) 21:59:03.04ID:???
ムーニーちゃんしんぷだい
ご冗談でしょう?名無しさん [] 2019/02/26(火) 22:03:55.75:vjaNCGJ/
自分の理解では量子コンピュータは2^nの計算を同時にできるやつでなく、
現実で起こる量子的な確率で結果が求まるやつでは?
たとえば、ネコのHPが0から100(死亡〜健康)まであるとして、
箱に入れて出す処理すると、均一確率ならランダムで0から100が出力される。
ご冗談でしょう?名無しさん [] 2019/02/26(火) 22:15:55.38:vjaNCGJ/
量子コンピュータ - Wikipedia
ショアのアルゴリズム

ショアのアルゴリズムとは、素因数分解問題を高速に解くことができるアルゴリズムのことである。
古典コンピュータでは準指数時間で解くアルゴリズムしか知られていない。
1994年にピーター・ショアによって発見された。ショアは本件で、ネヴァンリンナ賞とゲーデル賞を受賞した。

2001年12月にIBMアルマデン研究所にて7qubitの量子コンピュータで15の素因数分解に成功した(Nature, 12月20日発行号)。

ショアのアルゴリズムは、量子コンピュータが離散フーリエ変換を高速に実行できることによる。
また、アルゴリズム全体は確率的 (BQP) であり、正しい答えが得られるまで、何度も試行する。

手順の概略は以下の2つ。

全ての x に対して、均等な確率となるように初期化する。
そして、それを ax mod N のみ確率を持ち、それらは均等になるように変換する。

ax mod N は周期 r を持つ。この周期が求める位数である。
従って、1で得られた結果を離散フーリエ変換する。
すると、周波数 1/r のところの確率が大きくなるので、観測すると、高い確率で r が得られる。
失敗した場合は、成功するまで繰り返す。
ご冗談でしょう?名無しさん [sage] 2019/02/26(火) 22:18:55.11ID:???

やあ、凄いねあんた。
最近、このスレは自演基地外に占領されてたから。
学士であんな論文書けるんだ。

で、計算にかかる時間ってのは、直接的な時間ではなく
手法の違いによる計算量を比較したものなの?
a4 ◆L1L.Ef50zuAv [] 2019/02/26(火) 22:21:28.28:iTAeKIEa

僕の理解では、猫の生死を0と1として、猫を入れた毒ガスの部屋を100個作って
実験すると、2^100通りの世界がすぐできる。でも、単純に1つの部屋において、
生きている猫の世界から死んでいる猫の世界を観測できるわけじゃなくて、
素因数分解とかなら何故か数学的にはできてしまう、という話だと思っています。
その生きている猫の世界から死んでいる猫の世界を観測できるのがブラックホールに
よる量子の複製を用いた方法なんじゃないかと。聞き慣れない現代人からすると、
危険に思うかもしれませんが、未来の現実なんじゃないかと。だから、早くこの
コンピュータを完成させたいです。過去の人は逆に曼荼羅などで知ってます。
a4 ◆L1L.Ef50zuAv [] 2019/02/26(火) 22:29:33.59:iTAeKIEa

ありがとうございます。

(ユニタリ)量子計算に関しては教科書に書いてあることを書いただけです。
直接的な時間?直接には多世界解釈で思い通りに計算できないんですよ。ユニタリ
という数学的規制を受けるのです。だから、いくつかのアルゴリズムがあります。
Groverのアルゴリズムでも素因数分解ができるとかいう話はあるかもしれません。
ご冗談でしょう?名無しさん [sage] 2019/02/26(火) 22:35:36.99ID:???

あっちのスレ、ざっと見たjけど
あんた中国語もC++もできるんだって?
へえー

ところで、AIとプログラムはどう違うのかって
聞かれたら、あんたならどう答える?
a4 ◆L1L.Ef50zuAv [] 2019/02/26(火) 22:43:43.02:iTAeKIEa

中国語?日常会話ならできます。C++?逆にそれしかできないかもしれません。

AIは、「問題を高速且つ正確に解決するプログラム」のことで、プログラム⊃AI
なんじゃないかと。
a4 ◆L1L.Ef50zuAv [] 2019/02/26(火) 22:50:13.00:iTAeKIEa

難しいですね、この答え。議論の余地のあるところとは思います。僕の作っている
人工知能は、述語論理のようなニューラルコードで、C言語ど同等の記述能力を
持たせるという考え方で、解決しようと思っています。でも、プログラムと何が
違うのか、と言われたら、その「問題を高速且つ正確に解決する」という方向に
遺伝的プログラミングで進化させていこうとしているところです。
ご冗談でしょう?名無しさん [sage] 2019/02/26(火) 22:53:49.15ID:???

ふむ。
「解決」だけでなく、
何かを創造する「ひらめき」ってプログラミングできると思う?

また、AIが究極に進化したら
自我を持つようになると思う?

AIって、人によって指してるものが
ただのプログラムなのか、
人間の脳みたいに創造性や自我のあるものなのか
幅広いからね。
a4 ◆L1L.Ef50zuAv [] 2019/02/26(火) 23:12:10.05:iTAeKIEa

「ひらめき」という概念は幅広いので、簡単には言えませんが、基本的には組み合わせ
の問題でしょう。複雑な数式の積分を、数式として出力するときにmathematicaだったら
一発で出るかもしれませんが、正規表現などでいろんな数式を微分すれば無理やり
数式が計算できるかもしれません。これが基本的な考え方ではないかと。僕は人工生命
の研究をしていたことがあるので、交叉と突然変異から出てきた「子供を生んで、
子供が食料を探して、その子供を食べる」といった創発的なプログラミングは経験
してます。あとは、スパコンなどの処理速度の問題や、人間の文明をライブラリに
どれだけ落とせるか、などで時間の問題でしょう。未来学者のレイ・カーツワイルは
人間のような知能ができるのは2029年頃だと主張してます。

自我があるプログラムに関しては、僕は3つの方法を考えています。
(1)そのまま人間がプログラムする。
(2)EPFLのBlue Brain Projectのように脳神経を全てシミュレーションする。
(3)AIを弱肉強食の世界に置いて、遺伝的プログラミングする。

いかがでしょうか?
ご冗談でしょう?名無しさん [sage] 2019/02/27(水) 00:02:07.55ID:???

なかなか知的レベルの高い人だね。
積分を求めるには、微分の総当りみたいなことをするか。

ちなみに、高校数学の積分は不定積分から定積分を教えているが、
これ本当は大間違いであって、定積分こそが積分であり、不定積分はその途中で出てくるオマケなんだな。
まあ、割とどうでもいい。

「子供を生んで、 子供が食料を探して、その子供を食べる」か。
なんだか奥深くて怖いプログラムだね。
「ひらめき」は、総当りとライブラリの蓄積でなんとかなると、力技かな。

自我については特に聞きたい。
あんたのその方法では、自我はコピー可能だと思うがどうだろう。
そうであればやっかいな問題が出てくる。

自我がコピー可能だとして、AとBがあるとする。
コピーの関係にあるので、A=Bだな。
すると、どちらに「今まで自分だと認識していた自我」が継続していることになるのだろうか?

コピーして、AとBのシンクロを切り離して独立させたとき、
Aから見たBは何を考えているのか分からず、その逆も同じである。
つまり、AとBは互いが他人に見えるのだ。
そのどちらに、今までの自分が継続していることになるのか、
この答えは出せないと思うがどうだろう?

すると、自我はコピーできない、
コピーできるものは本当の自我ではないということになってしまうがいかに。

宇宙で永遠に分からないものが3つあると思う。
・無限大、すなわち宇宙の果て
・無限小、すなわち素粒子未満の世界
・自我

ではおやすみ。
ご冗談でしょう?名無しさん [sage] 2019/02/27(水) 00:03:27.39ID:???
ヒマラヤさんみたいなことを言う人、ヒマラヤさん以外にもいるんですね
a4 ◆L1L.Ef50zuAv [] 2019/02/27(水) 01:04:08.17:+vmeHnR6

コピー?可能ですよ。子供とか作るみたいだったら可愛いかもしれませんが、(2)の
Blue Brainのような方法だと、脳が狂ってGABAのような神経伝達物質を調整しないと
いけないかもしれません。

簡単に言えば魂って何?の問題かな。僕は昔は、人工知能を持った生命を生存競争
させると、個体が、「私は特別な何かを持っている。」「その何かは分割できない。」
などの文章がそのまま書かれた遺伝子情報を持った者が生き残り、今に至る、と
考えていました。

ところが、量子コンピュータに逢ってから、考え方が変わりました。上述の論文にも
なってますが、人間の人生の数列が、「帰納律という法則が成り立つランダム
な数列」だ、と考えると、あなたの言うところの無限小のおかしな物理の意味も
わかります。ほかにも目の前の机の模様などを思い通りに動かしたいとき、僕は
眼鏡をかけているので、眼鏡をとって、近づけたり遠ざけたりしながら、その
模様の確率分布を変えます。それで、思い通りの模様だと判断したら、そこで
観測、ということで、眼鏡をかけると、そのとおりの模様になっている、という
ようなものです。これを利用して、余剰次元に魂の存在(分割できない特別な
物質)を観測できればいいなー、と思っています。そのためには、まず、
量子コンピュータで形而下学的に、下の世界をシミュレーションして、特別な物質を
生命につけます。それで僕がそうしようとしているので、確率が上がり、我々の魂を
観測しに行くと、存在してるのでは?という話です。だから、こうすれば、自我の
同一性が保たれるのではないかと。

おやすみなさい。僕は今日はもうちょっと起きてます。
a4 ◆L1L.Ef50zuAv [] 2019/02/27(水) 01:13:38.74:+vmeHnR6
stackexchangeからメールがありました。コピペします。

"The Physics Stack Exchange community only works because users like you
generously share their knowledge to solve each other's problems. "

嫌われてはないみたいですね。また頃合いを見計らって使ってみます。
ご冗談でしょう?名無しさん [] 2019/02/27(水) 03:19:57.12:DzZ3WfBo
力学と電磁気のおススメの演習書教えて
当方、初学者
ご冗談でしょう?名無しさん [sage] 2019/02/27(水) 16:48:53.40ID:???
ぶどうをレンチンするとプラズマ発生する理由が判明ってやつみたんだけどさ、鶏つくね串の2つのつくねの間が焦げるのってぶどうと同じ理由なのかな?
あと昔は焦げることなかったんだけど最近よく焦げるようになった気がする
ご冗談でしょう?名無しさん [sage] 2019/02/27(水) 17:23:37.30ID:???
真空における点電荷が高密度の場合
プラズマ状態だと思われるのですが、
外部から電場、磁場を与えずかつ電場、磁場が時間変化しないなら、そのプラズマはその場に留まり、自ら生成するのでしょうか?
ご冗談でしょう?名無しさん [sage] 2019/02/27(水) 20:29:07.76ID:???
どこからどこまでか前提なのか自分でも分かってないだろ
ご冗談でしょう?名無しさん [] 2019/02/27(水) 23:12:14.39:TR0ygJKJ
物体の運動エネルギーが1ずつ上がってくとき、何で速さは同様に増えていかないんでしょうか?

静止したとこから速さが1ずつ増えるのと、すでに高速で動いてるとこからさらに1ずつ増えるのではなぜ速さの増え方に違いが生まれるのでしょうか?
ご冗談でしょう?名無しさん [sage] 2019/02/27(水) 23:31:23.20ID:???
物を押すことを考えてみましょう
止まっているものを押すときはちょっと手を触れるだけでいいですね
でも、既に動いている物を押そうとすると、自分も動かないといけませんね
その分余計に仕事がかかるわけです
仕事=力×距離ですからね
物の動きについていこうとする分距離が伸びます
ご冗談でしょう?名無しさん [] 2019/02/28(木) 00:19:07.52:3rYeKUd8
物理と、リッチフロー/幾何化予想/ポアンカレ予想はどのくらい関係してますか。
一般相対性理論をわかっていれば証明をおえますか。
ご冗談でしょう?名無しさん [sage] 2019/02/28(木) 12:50:09.36ID:???
熱的死は地球上に存在するの?
ご冗談でしょう?名無しさん [sage] 2019/02/28(木) 13:57:49.18ID:???
急にニセ質問が増えたな
ご冗談でしょう?名無しさん [] 2019/02/28(木) 21:03:22.19:DQeU7txZ

なぜエネルギーが速さの二乗に比例するのか、その根拠が知りたいんです。それが納得できれば、そこから仕事とかも定義していけるので。
ご冗談でしょう?名無しさん [sage] 2019/02/28(木) 21:08:21.80ID:???
逆ですよ。仕事が力×距離で定義されていて、その仕事分だけ運動エネルギーを得たとすると
必然的に速さの2乗(正確には(1/2)mv^2という良く知られた式)になります。
どんな力学の教科書でもその導出は載ってると思いますが。
ご冗談でしょう?名無しさん [sage] 2019/02/28(木) 21:27:54.49ID:???
ビオサバールの法則の分母の4πr^2の
意味を教えてください
磁場は円周上に回転して発生するのはわかるんですが、球面の表面積とは関係性はありますか?
ご冗談でしょう?名無しさん [] 2019/02/28(木) 21:40:07.35:DQeU7txZ

運動エネルギーをまず定義して、運動の変化の前後の、その差を作用がなした仕事として導く方が理解しやすいので。

Fxで定義される仕事なる量はそもそも力学的に意味不明で、運動エネルギーから導くならFxが非常に便利な量として使われるのは納得できるんですよ。

仕事から運動エネルギーを定義したりするのはただのごまかしにしか見えないです。仕事という概念や量を一番基本的なものにするのがわからないので。
ご冗談でしょう?名無しさん [sage] 2019/02/28(木) 21:51:35.91ID:???

初速vの質点が加速度aで加速されながら距離x進んだ結果、速さがVになったとする。この時、次式が成り立つ。

 V^2=v^2+2ax

両辺に質量mをかけて2で割れ。
ご冗談でしょう?名無しさん [] 2019/02/28(木) 22:07:05.33:DQeU7txZ

それはわかってて、その式からFxを
定義してると自分は思うので。本来なら運動エネルギーの係数は1/2じゃなくてもいいのにそうなってるのは、その式でFxに合わせてるから。

Fxは仕事という名称概念を与える必要はないけど、ポテンシャルや場を定義するのに力×距離からなる量が便利かつ必要だから特別な名称が与えられた補助的な概念で、力学的には本質的なものじゃないと自分は思うんです。

そうすると、一番基本的な量や概念は運動エネルギーなので、その定義などの根拠が知りたいと思ってるんです。
ご冗談でしょう?名無しさん [sage] 2019/02/28(木) 22:10:59.90ID:???
まず本質的の定義を示すべきだと思います。
ご冗談でしょう?名無しさん [sage] 2019/02/28(木) 22:11:49.70ID:???

エネルギーは系のハミルトニアンとして定義される量です
ハミルトニアンとはラグランジアンをルジャンドル変換して得られる量のことで、ラグランジアンとはオイラーラグランジュ方程式にかけると運動方程式を再現するような量のことです
オイラーラグランジュ方程式は最小作用の原理から導かれますね
ラグランジアンのうち、系の慣性項を導出する部分を運動エネルギーと称しますね
ご冗談でしょう?名無しさん [] 2019/02/28(木) 22:23:21.90:3rYeKUd8
F=maはこの辺りだと計算できることになってるが。 エントロピック重力、ブラックホール熱力学、量子情報理論


エントロピック重力理論
ここで話の面白さを明確にするため、ニュートンの逆2乗則を仮定せず、距離に関してどのような強さの重力Fがかかっているかを、知らないとしよう。
ttp://http://mhotta.hatenablog.com/entry/2016/12/25/092822

計算する時空 量子情報科学から見た宇宙
「コンピューターとブラックホールの違いはなんだろう?まるでジョークみたいだが,これは今日の物理学における最も深遠な問題の1つなのだ」
最新の量子情報理論によれば,半導体のチップだけでなく,あらゆる物体が計算している。石ころも,人間も,水爆も,宇宙も──。
物体はそれ自身を構成する基本粒子の位置と速度によって情報を記録し,粒子が相互作用するたびにその情報を書き変える。
近年,情報理論と量子力学が結びついた量子情報科学の登場によって,ブラックホールが情報を放出する仕組みが次々と提唱されている。
ttp://http://www.nikkei-science.com/page/magazine/0502/space.html
ご冗談でしょう?名無しさん [] 2019/02/28(木) 22:31:45.66:DQeU7txZ

運動の原因たる力または作用は存在する、物体の運動を制約するエネルギーが存在してその量が決まっている、でも仕事なる量は物体に備わってもいない、運動を起こす原因でもない、エネルギーに単位が等しい概念に過ぎない。力学的世界観に仕事は必要ないと思います。


解析力学は今勉強中なので、ちょっとよくわからないですね。
ご冗談でしょう?名無しさん [sage] 2019/02/28(木) 22:36:43.16ID:???

わからないんですね(笑)
仕事を表に出さない力学体系が解析力学です
そちらを勉強してから文句は言いましょうね
ご冗談でしょう?名無しさん [sage] 2019/02/28(木) 22:41:57.39ID:???
仕事を使わずにポテンシャルをどう定義するんですかね
ご冗談でしょう?名無しさん [sage] 2019/02/28(木) 22:43:35.10ID:???
F=-∇Uですね
そもそも解析力学は力という対象はなくて、ポテンシャルという対象を基本とする方が多いと思いますけど
ご冗談でしょう?名無しさん [sage] 2019/02/28(木) 22:44:46.76ID:???
解析力学にあまり力は出てきませんね
ご冗談でしょう?名無しさん [sage] 2019/02/28(木) 22:45:48.52ID:???

Fは解析力学でどう定義するんですか?
ご冗談でしょう?名無しさん [sage] 2019/02/28(木) 22:47:10.35ID:???
普通のニュートン力学ではどう定義するんですか?
最初から与えられたものではないですか?
ご冗談でしょう?名無しさん [sage] 2019/02/28(木) 22:48:02.61ID:???
わからないんですね(笑)
ご冗談でしょう?名無しさん [sage] 2019/02/28(木) 23:03:11.89ID:???

>物体の運動を制約するエネルギーが存在してその量が決まっている

エネルギーは定数分だけの自由度がありますよね?
つまり量が決まってるわけではないですよね?
力だけでは駄目なんですか?なぜエネルギーを本質的としたんですか?
ご冗談でしょう?名無しさん [sage] 2019/02/28(木) 23:56:01.06ID:???

運動エネルギーをmv^2で定義しても全部定数倍すればこれに矛盾しないんじゃないかと思ったけど
運動方程式がF=2maになったわ

運動エネルギーの係数1/2を変えようと思ったら運動方程式を書き換える羽目になる
ご冗談でしょう?名無しさん [sage] 2019/03/01(金) 00:32:06.81ID:???
n[mol]の気体の温度をΔT[K]だけ高める低圧変化では、
気体が外部にする仕事はnRΔTで表せることを示せ。
ご冗談でしょう?名無しさん [sage] 2019/03/01(金) 00:40:51.58ID:???
PΔV=nRΔT
ご冗談でしょう?名無しさん [sage] 2019/03/01(金) 00:51:36.15ID:???
P(V+ΔV)=nR(T+ΔT)
PV=nRT
両辺を引いて
PΔV=nRΔT

これであってる?
ご冗談でしょう?名無しさん [] 2019/03/01(金) 05:48:24.90:SM97tpKb

ふむ。
ビオサバールの法則は、
電流素片を軸として磁場が円状に形成されることを示しておる。

で、ビオサバールの法則にパイを含めることで、同法則を
rotB=μ0・i
と変形したときにパイが出てこなくできるわけ。
だからパイをつけたんだよ。もちろん、この式はrotH=iにつながる。

電流によって磁場が円状にできる、つまり円周のパイ由来だ。
球の表面積由来ではない。

くっくっく
ご冗談でしょう?名無しさん [] 2019/03/01(金) 06:03:28.55:SM97tpKb
ちなみに
クーロンの法則も球の表面積由来ではない。
分母に4πR^2があるが、これをデタラメな説明で
球の表面積由来にしているサイトや本ばかりだが、簡単に論破できるからな。

力線を使って球の表面積由来にしておるが、力線密度を使うときに
論理が破綻するんだな、これが。
自分で考えてみろ。

くっくっく
ご冗談でしょう?名無しさん [] 2019/03/01(金) 06:18:19.94:2NK+QlPL
大学の物理学科で、一般相対性理論の一般論は
何年生で何ヶ月くらいやるんですか?
前、独学で勉強しようとして難しそうで断念したんですが。
ご冗談でしょう?名無しさん [sage] 2019/03/01(金) 06:46:00.44ID:???
2年の時に半期で特殊、3年の時に半期で一般やった
ご冗談でしょう?名無しさん [sage] 2019/03/01(金) 07:55:55.26ID:???

東大かな?

京大は4回で一般相対論する
ご冗談でしょう?名無しさん [sage] 2019/03/01(金) 09:16:14.33ID:???

田中さんの重力特論は4回生配当だけど受けてるのほとんど3回生だろ
ご冗談でしょう?名無しさん [sage] 2019/03/01(金) 09:22:23.44ID:???

なるほど、係数が1/2なのは何故かという疑問か。
ならば、答は簡単だ。運動エネルギーは(1/2)mv^2ではない。正しくは、

 mc^2/√(1−v^2/c^2)−mc^2

だ。(1/2)mv^2は、その近似に過ぎない。
ご冗談でしょう?名無しさん [sage] 2019/03/01(金) 09:47:49.24ID:???

相間はいいからキチガイだろ
ご冗談でしょう?名無しさん [sage] 2019/03/01(金) 10:20:07.44ID:???

意味が分からない。
は相対性理論の教科書なら大抵は載っている説明だろ。
ご冗談でしょう?名無しさん [sage] 2019/03/01(金) 10:25:42.30ID:???
mγc^2の導出の仕方によっては、運動エネルギーの係数が1/2であることを使ったりしないのかな?
よくわからんけど
ご冗談でしょう?名無しさん [sage] 2019/03/01(金) 11:20:51.98ID:???
全角英数字って時点で不穏な気配するから仕方ない
ご冗談でしょう?名無しさん [sage] 2019/03/01(金) 13:03:10.91ID:???

エネルギーがスカラーだからさ
力はベクトルだから座標に依存してしまう
相対論になるとエネルギーも運動量ベクトルの成分になって資格を失うがね
ご冗談でしょう?名無しさん [sage] 2019/03/01(金) 13:05:27.26ID:???
座標変換に対して云々がスカラーの定義なら、ニュートン力学の段階で運動エネルギーはスカラーじゃないぞ
ご冗談でしょう?名無しさん [sage] 2019/03/01(金) 18:01:26.02ID:???
ムーニーちゃんしんぷだい
ご冗談でしょう?名無しさん [] 2019/03/01(金) 21:00:05.25:Vi8aCzAS

それは運動量保存則が担うとは思うんですが、運動量保存則の位置付けは微妙な気がしてて。その辺はまだよく整理できてないんで。

ただエネルギーは永久機関の追求などから経験的に要請されてきた経緯があるとは思うんですけど、運動量ベクトルにはそういうのはないし。

宇宙全体の運動量ベクトルが保存されるとは何を意味するのか、わからないんですよね。エネルギーはその点永久機関を否定したり、理論的に未知の粒子の存在を予想させてくれたりと、運動量ベクトルより貢献してきた概念だと思うので。
ご冗談でしょう?名無しさん [sage] 2019/03/01(金) 21:04:23.39ID:???

だから解析力学勉強しろって。エネルギー・運動量・角運動量、...その他の保存則の意味が
統一的に理解できるから
ご冗談でしょう?名無しさん [sage] 2019/03/01(金) 21:37:42.49ID:???

運動量ベクトルの保存は、系の空間並進対称性を意味します

また、エネルギーの保存は、系の時間並進対称性を意味しますね
ご冗談でしょう?名無しさん [sage] 2019/03/01(金) 21:52:22.14ID:???
■ちょっとした物理の質問はここに書いてね237■
ttps://rio2016.5ch.net/test/read.cgi/sci/1551444723/
ご冗談でしょう?名無しさん [] 2019/03/01(金) 22:54:08.32:Vi8aCzAS


解析力学の学習を進めてみます。
ご冗談でしょう?名無しさん [] 2019/03/03(日) 01:16:39.92:E6Tqm3Qs
一般相対性理論は特殊とあわせて一年やるのか。
微分幾何も物理も苦手で断念したけど。
絵や文で説明されても理解に差が出てしまうので。
定義とロジックだけで理解できるはずときっちりやってみる。
ご冗談でしょう?名無しさん [sage] 2019/03/03(日) 01:56:20.64ID:???
ムニちゃーんポポ
ご冗談でしょう?名無しさん [sage] 2019/03/03(日) 13:32:56.51ID:???
場の古典論で相対論を自習した時は、電磁気学の知識不足に気づいて復習した時間の方が長くかかったよ
ご冗談でしょう?名無しさん [sage] 2019/03/13(水) 09:07:53.78ID:???
質問いいですか?
タイムマシンは難しそうですが、スモールライトあたりは可能なんですか??
輸送コスト減ってAmazonが早くなる気がします((o(。>ω<。)o))
ご冗談でしょう?名無しさん [] 2019/03/13(水) 13:42:42.56:y4vm5KiH
コンデンサーの電気容量について質問です。
±Qの電荷を与えたときのQ/Vが電気容量の定義ですよね。
しかし電荷の量が±Qではないコンデンサーがあるのです。

例えば内球の半径a、外球の半径bの2つの充分に薄い同心導体球コンデンサーを考えてください。(内球を接地する)
計算すると内球に+Qの電荷があるとき、外球に-(b/a)×Qの電荷があることになります。
この場合の電気容量はどのように定義しているのでしょうか?
ご冗談でしょう?名無しさん [sage] 2019/03/13(水) 13:45:43.32ID:???
何を勘違いしてんだ?
ご冗談でしょう?名無しさん [sage] 2019/03/13(水) 14:45:09.62ID:???
電気力線の話をするならこのスレで完結させてくれよ
994 [sage] 2019/03/13(水) 18:28:08.25ID:???
質問が分かりにくかったようなので書き方を変えます。
問題集にこんな問題がありました。
この問題の解答が良く分かりません。

内球の半径がa,外球の半径がbの充分に薄い同心導体球よりなるコンデンサーがある。
内球を接地した場合の電気容量を求めよ。
ご冗談でしょう?名無しさん [sage] 2019/03/13(水) 18:40:40.50ID:???
内径からガウスの定理に準じて発生する電界を求めて、bからaまで積分して電位差を求めてQで割って逆数をとるだけだよ。
ご冗談でしょう?名無しさん [sage] 2019/03/13(水) 18:51:12.08ID:???
>計算すると内球に+Qの電荷があるとき、外球に-(b/a)×Qの電荷があることになります。
その計算が間違ってるのでは?
ご冗談でしょう?名無しさん [] 2019/03/13(水) 18:58:57.07:Z968Fe2n
ガウスの「法則」な。
ガウスの「定理」ってのは、面積分と体積分の変換定理だぞ。

くっくっく
1001 [] Over 1000Thread
このスレッドは1000を超えました。
新しいスレッドを立ててください。
life time: 48日 5時間 44分 46秒
1002 [] Over 1000Thread
5ちゃんねるの運営はプレミアム会員の皆さまに支えられています。
運営にご協力お願いいたします。


───────────────────
《プレミアム会員の主な特典》
★ 5ちゃんねる専用ブラウザからの広告除去
★ 5ちゃんねるの過去ログを取得
★ 書き込み規制の緩和
───────────────────

会員登録には個人情報は一切必要ありません。
月300円から匿名でご購入いただけます。

▼ プレミアム会員登録はこちら ▼
ttps://premium.5ch.net/

▼ 浪人ログインはこちら ▼
ttps://login.5ch.net/login.php

勢い5万以上のスレをメールでお知らせするサービス、実施中!!
憧れボディをGETしたい!その夢、ボニックで!

2ch勢いランキング アーカイブ 物理板ランキング

凡例:

レス番

100 (赤) → 2つ以上レスが付いている
100 (紫) → 1つ以上レスが付いている

名前

名無しさん (青) → sage のレス
名無しさん (緑) → age のレス

ID

ID:xxxxxxx (赤) → 発言が3つ以上のID
ID:xxxxxxx (青) → 発言が2つ以上のID

このページは2ch勢いランキングが作成したアーカイブです。削除についてはこちら